NCERT Solution For Class 10 Math All Chapter 2022

Download as pdf or txt
Download as pdf or txt
You are on page 1of 527

NCERT Solutions for Class 10 Maths Unit 1

Real Numbers Class 10


Unit 1 Real Numbers Exercise 1.1, 1.2, 1.3, 1.4 Solutions
Exercise 1.1 : Solutions of Questions on Page Number : 7
Q1 :

Use Euclid's division algorithm to find the HCF of:

Answer :

(i) 135 and 225

Since 225 > 135, we apply the division lemma to 225 and 135 to obtain

225 = 135 x 1 + 90

Since remainder 90 ≠ 0, we apply the division lemma to 135 and 90 to obtain

135 = 90 x 1 + 45

We consider the new divisor 90 and new remainder 45, and apply the division lemma to obtain

90 = 2 x 45 + 0

Since the remainder is zero, the process stops.

Since the divisor at this stage is 45,

Therefore, the HCF of 135 and 225 is 45.

(ii)196 and 38220

Since 38220 > 196, we apply the division lemma to 38220 and 196 to obtain

38220 = 196 x 195 + 0

Since the remainder is zero, the process stops.

Since the divisor at this stage is 196,

Therefore, HCF of 196 and 38220 is 196.

(iii)867 and 255

Since 867 > 255, we apply the division lemma to 867 and 255 to obtain

867 = 255 x 3 + 102

Since remainder 102 ≠ 0, we apply the division lemma to 255 and 102 to obtain

255 = 102 x 2 + 51

We consider the new divisor 102 and new remainder 51, and apply the division lemma to obtain

102 = 51 x 2 + 0

Since the remainder is zero, the process stops.


Since the divisor at this stage is 51,

Therefore, HCF of 867 and 255 is 51.

Q2 :

Show that any positive odd integer is of the form , or , or , where q is some integer.

Answer :

Let a be any positive integer and b = 6. Then, by Euclid's algorithm,

a = 6q + rfor some integer q ≥ 0, and r = 0, 1, 2, 3, 4, 5 because 0 ≤ r < 6.

Therefore, a = 6q or 6q + 1 or 6q + 2 or 6q + 3 or 6q + 4 or 6q + 5

Also, 6q + 1 = 2 x 3q + 1 = 2k1 + 1, wherek1 is a positive integer

6q + 3 = (6q + 2) + 1 = 2 (3q + 1) + 1 = 2k2 + 1, where k2 is an integer

6q + 5 = (6q + 4) + 1 = 2 (3q + 2) + 1 = 2k3 + 1, where k3 is an integer

Clearly, 6q + 1, 6q + 3, 6q + 5 are of the form 2k + 1, where k is an integer.

Therefore, 6q + 1, 6q + 3, 6q + 5 are not exactly divisible by 2. Hence, these expressions of numbers are odd
numbers.

And therefore, any odd integer can be expressed in the form 6q + 1, or 6q + 3,

or 6q + 5

Q3 :

An army contingent of 616 members is to march behind an army band of 32 members in a parade. The two
groups are to march in the same number of columns. What is the maximum number of columns in which
they can march?

Answer :

HCF (616, 32) will give the maximum number of columns in which they can march.

We can use Euclid's algorithm to find the HCF.

616 = 32 x 19 + 8

32 = 8 x 4 + 0

The HCF (616, 32) is 8.

Therefore, they can march in 8 columns each.

Q4 :
Use Euclid's division lemma to show that the square of any positive integer is either of form 3m or 3m + 1 for
some integer m.

[Hint: Let x be any positive integer then it is of the form 3q, 3q + 1 or 3q + 2. Now square each of these and
show that they can be rewritten in the form 3m or 3m + 1.]

Answer :

Let a be any positive integer and b = 3.

Then a = 3q + r for some integer q ≥ 0

And r = 0, 1, 2 because 0 ≤ r < 3

Therefore, a = 3q or 3q + 1 or 3q + 2

Or,

Where k1, k2, and k3 are some positive integers

Hence, it can be said that the square of any positive integer is either of the form 3m or 3m + 1.

Q5 :

Use Euclid's division lemma to show that the cube of any positive integer is of the form 9m, 9m + 1 or 9m +8.

Answer :

Let a be any positive integer and b = 3

a = 3q + r, where q ≥ 0 and 0 ≤ r < 3

Therefore, every number can be represented as these three forms. There are three cases.

Case 1: When a = 3q,

Where m is an integer such that m =

Case 2: When a = 3q + 1,

a3 = (3q +1)3

a3= 27q3 + 27q2 + 9q + 1

a3 = 9(3q3 + 3q2 + q) + 1
a3 = 9m + 1

Where m is an integer such that m = (3q3 + 3q2 + q)

Case 3: When a = 3q + 2,

a3 = (3q +2)3

a3= 27q3 + 54q2 + 36q + 8

a3 = 9(3q3 + 6q2 + 4q) + 8

a3 = 9m + 8

Where m is an integer such that m = (3q3 + 6q2 + 4q)

Therefore, the cube of any positive integer is of the form 9m, 9m + 1,


or 9m + 8.

Next Chapter 2 : Polynomials >>

Exercise 1.2 : Solutions of Questions on Page Number : 11


Q1 :

Express each number as product of its prime factors:

Answer :

Q2 :

Find the LCM and HCF of the following pairs of integers and verify that LCM × HCF = product of the two
numbers.

Answer :
Hence, product of two numbers = HCF × LCM

Hence, product of two numbers = HCF × LCM

Hence, product of two numbers = HCF × LCM

Q3 :

Find the LCM and HCF of the following integers by applying the prime factorisation method.

Answer :
Q4 :

Given that HCF (306, 657) = 9, find LCM (306, 657).

Answer :

Q5 :

Check whether 6n can end with the digit 0 for any natural number n.
Answer :

If any number ends with the digit 0, it should be divisible by 10 or in other words, it will also be divisible by 2 and 5 as
10 = 2 x 5

Prime factorisation of 6n = (2 x 3)n

It can be observed that 5 is not in the prime factorisation of 6n.

Hence, for any value of n, 6n will not be divisible by 5.

Therefore, 6n cannot end with the digit 0 for any natural number n.

Q6 :

Explain why 7 x 11 x 13 + 13 and 7 x 6 x 5 x 4 x 3 x 2 x 1 + 5 are composite numbers.

Answer :

Numbers are of two types - prime and composite. Prime numbers can be divided by 1 and only itself, whereas
composite numbers have factors other than 1 and itself.

It can be observed that

7 x 11 x 13 + 13 = 13 x (7 x 11 + 1) = 13 x (77 + 1)

= 13 x 78

= 13 x 13 x 6

The given expression has 6 and 13 as its factors. Therefore, it is a composite number.

7 x 6 x 5 x 4 x 3 x 2 x 1 + 5 = 5 x (7 x 6 x 4 x 3 x 2 x 1 + 1)

= 5 x (1008 + 1)

= 5 x 1009

1009 cannot be factorised further. Therefore, the given expression has 5 and 1009 as its factors. Hence, it is a
composite number.

Q7 :

There is a circular path around a sports field. Sonia takes 18 minutes to drive one round of the field, while
Ravi takes 12 minutes for the same. Suppose they both start at the same point and at the same time, and go
in the same direction. After how many minutes will they meet again at the starting point?

Answer :

It can be observed that Ravi takes lesser time than Sonia for completing 1 round of the circular path. As they are
going in the same direction, they will meet again at the same time when Ravi will have completed 1 round of that
circular path with respect to Sonia. And the total time taken for completing this 1 round of circular path will be the
LCM of time taken by Sonia and Ravi for completing 1 round of circular path respectively i.e., LCM of 18 minutes and
12 minutes.

18 = 2 x 3 x 3

And, 12 = 2 x 2 x 3

LCM of 12 and 18 = 2 x 2 x 3 x 3 = 36

Therefore, Ravi and Sonia will meet together at the starting pointafter 36 minutes.

Next Chapter 2 : Polynomials >>

Exercise 1.3 : Solutions of Questions on Page Number : 14


Q1 :

Prove that is irrational.

Answer :

Let is a rational number.

Therefore, we can find two integers a, b (b ≠ 0) such that

Let a and b have a common factor other than 1. Then we can divide them by the common factor, and assume
that a and bare co-prime.

Therefore, a2 is divisible by 5 and it can be said that a is divisible by 5.

Let a = 5k, where k is an integer

This means that b2 is divisible by 5 and hence, b is divisible by 5.

This implies that a and b have 5 as a common factor.

And this is a contradiction to the fact that a and b are co-prime.

Hence, cannot be expressed as or it can be said that is irrational.

Q2 :

Prove that is irrational.

Answer :
Let is rational.

Therefore, we can find two integers a, b (b ≠ 0) such that

Since a and b are integers, will also be rational and therefore, is rational.

This contradicts the fact that is irrational. Hence, our assumption that is rational is false.

Therefore, is irrational.

Q3 :

Prove that the following are irrationals:

Answer :

Let is rational.

Therefore, we can find two integers a, b (b ≠ 0) such that

is rational as a and b are integers.

Therefore, is rational which contradicts to the fact that is irrational.


Hence, our assumption is false and is irrational.

Let is rational.

Therefore, we can find two integers a, b (b ≠ 0) such that

for some integers a and b

is rational as a and b are integers.

Therefore, should be rational.

This contradicts the fact that is irrational. Therefore, our assumption that is rational is false. Hence,
is irrational.

Let be rational.

Therefore, we can find two integers a, b (b ≠ 0) such that

Since a and b are integers, is also rational and hence, should be rational. This contradicts the fact that

is irrational. Therefore, our assumption is false and hence,

Next Chapter 2 : Polynomials >>

Exercise 1.4 : Solutions of Questions on Page Number : 17


Q1 :

Without actually performing the long division, state whether the following rational numbers will have a
terminating decimal expansion or a non-terminating repeating decimal expansion:
Answer :

(i)

The denominator is of the form 5m.

Hence, the decimal expansion of is terminating.

(ii)

The denominator is of the form 2m.

Hence, the decimal expansion of is terminating.

(iii)

455 = 5 × 7 × 13

Since the denominator is not in the form 2m × 5n, and it also contains 7 and 13 as its factors, its decimal expansion will
be non-terminating repeating.

(iv)

1600 = 26 × 52

The denominator is of the form 2m ×5n.

Hence, the decimal expansion of is terminating.

(v)
Since the denominator is not in the form 2m × 5n, and it has 7 as its factor, the decimal expansion of is non-
terminating repeating.

(vi)

The denominator is of the form 2m × 5n.

Hence, the decimal expansion of is terminating.

(vii)

Since the denominator is not of the form 2m × 5n, and it also has 7 as its factor, the decimal expansion

of is non-terminating repeating.

(viii)

The denominator is of the form 5n.

Hence, the decimal expansion of is terminating.

(ix)

The denominator is of the form 2m × 5n.

Q2 :

Write down the decimal expansions of those rational numbers in Question 1 above which have terminating
decimal expansions.

Answer :
(viii)
Q3 :

The following real numbers have decimal expansions as given below. In each case, decide whether they are

rational or not. If they are rational, and of the form , what can you say about the prime factor of q?

(i) 43.123456789 (ii) 0.120120012000120000… (iii)

Answer :

(i) 43.123456789

Since this number has a terminating decimal expansion, it is a rational number of the form and q is of the
form

i.e., the prime factors of q will be either 2 or 5 or both.

(ii) 0.120120012000120000 …

The decimal expansion is neither terminating nor recurring. Therefore, the given number is an irrational number.

(iii)

Since the decimal expansion is non-terminating recurring, the given number is a rational number of the form
and q is not of the form i.e., the prime factors of q will also have a factor other than 2 or 5.
 

Polynomials 
 
Exercise 2.1 
Question 1: 
The graphs of y = p(x) are given in following figure, for some polynomials p(x). Find the 
number of zeroes of p(x), in each case. 
(i) 

 
(ii) 

 
(iii) 

 
(iv) 
 
(v) 

 
(v) 

Answer: 

(i) The number of zeroes is 0 as the graph does not cut the x-axis at any point. 

(ii) The number of zeroes is 1 as the graph intersects the x-axis at only 1 point. 

(iii) The number of zeroes is 3 as the graph intersects the x-axis at 3 points. 

(iv) The number of zeroes is 2 as the graph intersects the x-axis at 2 points. 

(v) The number of zeroes is 4 as the graph intersects the x-axis at 4 points. 

(vi) The number of zeroes is 3 as the graph intersects the x-axis at 3 points. 

 
 
 

Exercise 2.2
Question 1: 
Find the zeroes of the following quadratic polynomials and verify the relationship 
between the zeroes and the coefficients. 

Answer: 

The value of is zero when x − 4 = 0 or x + 2 = 0, i.e., when x = 4 or x = −2 

Therefore, the zeroes of are 4 and −2. 

Sum of zeroes =  

Product of zeroes  

The value of 4s2 − 4s + 1 is zero when 2s − 1 = 0, i.e.,  

Therefore, the zeroes of 4s2 − 4s + 1 are and . 

Sum of zeroes =  
Product of zeroes  

The value of 6x2 − 3 − 7x is zero when 3x + 1 = 0 or 2x − 3 = 0, i.e., or  

Therefore, the zeroes of 6x2 − 3 − 7x are . 

Sum of zeroes =  

Product of zeroes =  

The value of 4u2 + 8u is zero when 4u = 0 or u + 2 = 0, i.e., u = 0 or u = −2 

Therefore, the zeroes of 4u2 + 8u are 0 and −2. 

Sum of zeroes =  

Product of zeroes =  

 
The value of t2 − 15 is zero when or , i.e., when 

Therefore, the zeroes of t2 − 15 are and . 

Sum of zeroes =  

Product of zeroes =  

The value of 3x2 − x − 4 is zero when 3x − 4 = 0 or x + 1 = 0, i.e., when or x = −1 

Therefore, the zeroes of 3x2 − x − 4 are and −1. 

Sum of zeroes =  

Product of zeroes  

  

 
 
Question 2: 
Find a quadratic polynomial each with the given numbers as the sum and product of its 
zeroes respectively. 

 
Answer: 

Let the polynomial be , and its zeroes be and . 

Therefore, the quadratic polynomial is 4x2 − x − 4. 

Let the polynomial be , and its zeroes be and . 

Therefore, the quadratic polynomial is 3x2 − x + 1. 

Let the polynomial be , and its zeroes be and . 


 

Therefore, the quadratic polynomial is . 

Let the polynomial be , and its zeroes be and . 

Therefore, the quadratic polynomial is . 

Let the polynomial be , and its zeroes be and . 

Therefore, the quadratic polynomial is . 

Let the polynomial be . 


 

Therefore, the quadratic polynomial is . 

Exercise 2.3 
 
Question 1: 
Divide the polynomial p(x) by the polynomial g(x) and find the quotient and remainder 
in each of the following: 

(i)  

(ii)  

(iii)  

Answer: 

Quotient = x − 3 

Remainder = 7x − 9 
 

Quotient = x2 + x − 3 

Remainder = 8 

Quotient = −x2 − 2 

Remainder = −5x +10 

Question 2: 
Check whether the first polynomial is a factor of the second polynomial by dividing the 
second polynomial by the first polynomial: 
 

Answer: 

=  

Since the remainder is 0, 

Hence, is a factor of . 

 
 

Since the remainder is 0, 

Hence, is a factor of . 

Since the remainder , 

Hence, is not a factor of . 

Question 3: 

Obtain all other zeroes of , if two of its zeroes are 

Answer: 
 

Since the two zeroes are , 

is a factor of . 

Therefore, we divide the given polynomial by . 

We factorize  

Therefore, its zero is given by x + 1 = 0 

x = −1 
As it has the term , therefore, there will be 2 zeroes at x = −1. 

Hence, the zeroes of the given polynomial are , −1 and −1. 

 
 
 
Question 4: 

On dividing by a polynomial g(x), the quotient and remainder were x − 2 


and − 2x + 4, respectively. Find g(x). 

Answer: 

g(x) = ? (Divisor) 

Quotient = (x − 2) 

Remainder = (− 2x + 4) 

Dividend = Divisor × Quotient + Remainder 

g(x) is the quotient when we divide by  


 

Question 5: 
Give examples of polynomial p(x), g(x), q(x) and r(x), which satisfy the division 
algorithm and 
(i) deg p(x) = deg q(x) 
(ii) deg q(x) = deg r(x) 
(iii) deg r(x) = 0 

Answer: 

According to the division algorithm, if p(x) and g(x) are two polynomials with 

g(x) ≠ 0, then we can find polynomials q(x) and r(x) such that 

p(x) = g(x) × q(x) + r(x), 

where r(x) = 0 or degree of r(x) < degree of g(x) 

Degree of a polynomial is the highest power of the variable in the polynomial. 

(i) deg p(x) = deg q(x) 

Degree of quotient will be equal to degree of dividend when divisor is constant ( i.e., 
when any polynomial is divided by a constant). 
Let us assume the division of by 2. 

Here, p(x) =  

g(x) = 2 

q(x) = and r(x) = 0 

Degree of p(x) and q(x) is the same i.e., 2. 

Checking for division algorithm, 

p(x) = g(x) × q(x) + r(x) 

= 2( ) 

=  

Thus, the division algorithm is satisfied. 

(ii) deg q(x) = deg r(x) 

Let us assume the division of x3 + x by x2, 

Here, p(x) = x3 + x 

g(x) = x2 

q(x) = x and r(x) = x 

Clearly, the degree of q(x) and r(x) is the same i.e., 1. 

Checking for division algorithm, 

p(x) = g(x) × q(x) + r(x) 

x3 + x = (x2 ) × x + x 
x3 + x = x3 + x 

Thus, the division algorithm is satisfied. 

(iii)deg r(x) = 0 

Degree of remainder will be 0 when remainder comes to a constant. 

Let us assume the division of x3 + 1by x2. 

Here, p(x) = x3 + 1 

g(x) = x2 

q(x) = x and r(x) = 1 

Clearly, the degree of r(x) is 0. 

Checking for division algorithm, 

p(x) = g(x) × q(x) + r(x) 

x3 + 1 = (x2 ) × x + 1 

x3 + 1 = x3 + 1 

Thus, the division algorithm is satisfied. 

Exercise 2.4
 
Question 1: 
Verify that the numbers given alongside of the cubic polynomials below are their 
zeroes. Also verify the relationship between the zeroes and the coefficients in each 
case: 

 
Answer: 

(i)  

Therefore, , 1, and −2 are the zeroes of the given polynomial. 

Comparing the given polynomial with , we obtain a = 2, b = 1, c = −5, d = 


Therefore, the relationship between the zeroes and the coefficients is verified. 

(ii)  

 
 

Therefore, 2, 1, 1 are the zeroes of the given polynomial. 

Comparing the given polynomial with , we obtain a = 1, b = −4, c = 5, d = 


−2. 

Verification of the relationship between zeroes and coefficient of the given polynomial 

Multiplication of zeroes taking two at a time = (2)(1) + (1)(1) + (2)(1) =2 + 1 + 2 = 5 

Multiplication of zeroes = 2 × 1 × 1 = 2  

Hence, the relationship between the zeroes and the coefficients is verified. 

Question 2: 
Find a cubic polynomial with the sum, sum of the product of its zeroes taken two at a 
time, and the product of its zeroes as 2, − 7, − 14 respectively. 

Answer: 

Let the polynomial be and the zeroes be . 

It is given that 
 

If a = 1, then b = −2, c = −7, d = 14 

Hence, the polynomial is . 

Question 3: 

If the zeroes of polynomial are , find a and b. 

Answer: 

Zeroes are a − b, a + a + b 

Comparing the given polynomial with , we obtain 

p = 1, q = −3, r = 1, t = 1 

The zeroes are . 


 

Hence, a = 1 and b = or . 

Question 4: 

]It two zeroes of the polynomial are , find other 


zeroes. 

Answer: 

Given that 2 + and 2 are zeroes of the given polynomial. 

Therefore, = x2 + 4 − 4x − 3 

= x2 − 4x + 1 is a factor of the given polynomial 

For finding the remaining zeroes of the given polynomial, we will find the quotient by 
dividing by x2 − 4x + 1. 
 

Clearly, =  

It can be observed that is also a factor of the given polynomial. 

And =  

Therefore, the value of the polynomial is also zero when or  

Or x = 7 or −5 

Hence, 7 and −5 are also zeroes of this polynomial. 

 
 
Question 5: 

If the polynomial is divided by another polynomial , 


the remainder comes out to be x + a, find k and a. 

Answer: 

By division algorithm, 

Dividend = Divisor × Quotient + Remainder 


Dividend − Remainder = Divisor × Quotient 

will be perfectly divisible by 


Let us divide by  

It can be observed that will be 0. 

Therefore, = 0 and = 0 

For = 0, 

2 k =10 

And thus, k = 5 

For = 0 

10 − a − 8 × 5 + 25 = 0 

10 − a − 40 + 25 = 0 
− 5 − a = 0 

Therefore, a = −5 

Hence, k = 5 and a = −5 

 
NCERT Solutions for Class 10 Maths Unit 3
Pair of Linear Equations in Two Variables Class 10
Unit 3 Pair of Linear Equations in Two Variables Exercise 3.1, 3.2, 3.3, 3.4, 3.5, 3.6, 3.7 Solutions

If two linear equations have the two same variables, they are called a pair of linear equations in
two variables. Following is the most general form of linear equations:

Here, a1, a2, b1, b2, c1 and c2 are real numbers such that;

A pair of linear equations can be represented and solved by the following methods:

a. Graphical method
b. Algebraic method

Exercise 3.1 : Solutions of Questions on Page Number : 44


Q1 :

Aftab tells his daughter, "Seven years ago, I was seven times as old as you were then. Also, three years from
now, I shall be three times as old as you will be." (Isn't this interesting?) Represent this situation
algebraically and graphically.

Answer :

Let the present age of Aftab be x.

And, present age of his daughter = y

Seven years ago,

Age of Aftab = x - 7

Age of his daughter = y - 7

According to the question,

Three years hence,

Age of Aftab = x + 3

Age of his daughter = y + 3


According to the question,

Therefore, the algebraic representation is

For ,

The solution table is

x -7 0 7
y 5 6 7

For ,

The solution table is

x 6 3 0
y 0 -1 -2
The graphical representation is as follows.

Q2 :
The coach of a cricket team buys 3 bats and 6 balls for Rs 3900. Later, she buys another bat and 3 more balls
of the same kind for Rs 1300. Represent this situation algebraically and geometrically.

Answer :

Let the cost of a bat be Rs x.

And, cost of a ball = Rs y

According to the question, the algebraic representation is

[Math Processing Error]

For ,

The solution table is

x 300 100 -100

y 500 600 700

For x+ 3y = 1300,

x = 1300 - 2y

The solution table is

x 400 700 1000

y 300 200 100

The graphical representation for first line is as follows.


And graph for second line will be,
Q3 :

The cost of 2 kg of apples and 1 kg of grapes on a day was found to be Rs 160. After a month, the cost of 4
kg of apples and 2 kg of grapes is Rs 300. Represent the situation algebraically and geometrically.

Answer :

Let the cost of 1 kg of apples be Rs x.

And, cost of 1 kg of grapes = Rs y

According to the question, the algebraic representation is

For ,
The solution table is

x 50 60 70
y 60 40 20
For 4x + 2y = 300,

The solution table is

x 70 80 75
y 10 - 10 0
The graphical representation is as follows.

Exercise 3.2 : Solutions of Questions on Page Number : 49


Q1 :

Form the pair of linear equations in the following problems, and find their solutions graphically.

(i) 10 students of Class X took part in a Mathematics quiz. If the number of girls is 4 more than the number of
boys, find the number of boys and girls who took part in the quiz.

(ii) 5 pencils and 7 pens together cost Rs 50, whereas 7 pencils and 5 pens together cost Rs 46. Find the cost
of one pencil and that of one pen.

Answer :

(i) Let the number of girls be x and the number of boys be y.


According to the question, the algebraic representation is

x + y = 10

x-y=4

For x + y = 10,

x = 10 - y

x 5 4 6
y 5 6 4
For x - y = 4,

x=4+y

x 5 4 3
y 1 0 -1
Hence, the graphic representation is as follows.

From the figure, it can be observed that these lines intersect each other at point (7, 3).

Therefore, the number of girls and boys in the class are 7 and 3 respectively.

(ii) Let the cost of 1 pencil be Rs x and the cost of 1 pen be Rs y.

According to the question, the algebraic representation is

5x + 7y = 50

7x + 5y = 46

For 5x + 7y = 50,
x 3 10 -4
y 5 0 10
7x + 5y = 46

x 8 3 -2
y -2 5 12
Hence, the graphic representation is as follows.

From the figure, it can be observed that these lines intersect each other at point (3, 5).

Therefore, the cost of a pencil and a pen are Rs 3 and Rs 5 respectively.

Q2 :

On comparing the ratios , find out whether the lines representing the following pairs of
linear equations at a point, are parallel or coincident:

Answer :
(i) 5x - 4y + 8 = 0

7x + 6y - 9 = 0

Comparing these equations with

and , we obtain

Since ,

Hence, the lines representing the given pair of equations have a unique solution and the pair of lines intersects at
exactly one point.

(ii) 9x + 3y + 12 = 0

18x + 6y + 24 = 0

Comparing these equations with

and , we obtain

Since ,

Hence, the lines representing the given pair of equations are coincident and there are infinite possible solutions for
the given pair of equations.

(iii)6x - 3y + 10 = 0

2x - y + 9 = 0

Comparing these equations with


and , we obtain

Since ,

Hence, the lines representing the given pair of equations are parallel to each other and hence, these lines will never
intersect each other at any point or there is no possible solution for the given pair of equations.

Q3 :

On comparing the ratios , find out whether the following pair of linear equations are
consistent, or inconsistent.

Answer :

(i) 3x + 2y = 5

2x - 3y = 7

These linear equations are intersecting each other at one point and thus have only one possible solution. Hence, the
pair of linear equations is consistent.

(ii)2x - 3y = 8
4x - 6y = 9

Since ,

Therefore, these linear equations are parallel to each other and thus have no possible solution. Hence, the pair of
linear equations is inconsistent.

(iii)

Since ,

Therefore, these linear equations are intersecting each other at one point and thus have only one possible solution.
Hence, the pair of linear equations is consistent.

(iv)5x - 3 y = 11

- 10x + 6y = - 22

Since ,

Therefore, these linear equations are coincident pair of lines and thus have infinite number of possible solutions.
Hence, the pair of linear equations is consistent.

(v)

Since

Therefore, these linear equations are coincident pair of lines and thus have infinite number of possible solutions.
Hence, the pair of linear equations is consistent.
Q4 :

Which of the following pairs of linear equations are consistent/ inconsistent? If consistent, obtain the
solution graphically:

Answer :

(i)x + y = 5

2x + 2y = 10

Since ,

Therefore, these linear equations are coincident pair of lines and thus have infinite number of possible solutions.
Hence, the pair of linear equations is consistent.

x+y=5

x=5-y

x 4 3 2
y 1 2 3
And, 2x + 2y = 10

x 4 3 2
y 1 2 3
Hence, the graphic representation is as follows.
From the figure, it can be observed that these lines are overlapping each other. Therefore, infinite solutions are
possible for the given pair of equations.

(ii)x - y = 8

3x - 3y = 16

Since ,

Therefore, these linear equations are parallel to each other and thus have no possible solution. Hence, the pair of
linear equations is inconsistent.

(iii)2x + y - 6 = 0

4x - 2y - 4 = 0

Since ,

Therefore, these linear equations are intersecting each other at one point and thus have only one possible solution.
Hence, the pair of linear equations is consistent.

2x + y - 6 = 0

y = 6 - 2x
x 0 1 2
y 6 4 2
And 4x - 2y - 4 = 0

x 1

Q5 :

Half the perimeter of a rectangular garden, whose length is 4 m more than its width, is 36 m. Find the
dimensions of the garden.

Answer :

Let the width of the garden be x and length be y.

According to the question,

y - x = 4 (1)

y + x = 36 (2)

y-x=4

y=x+4

x 0 8 12
y 4 12 16
y + x = 36

x 0 36 16
y 36 0 20
Hence, the graphic representation is as follows.
From the figure, it can be observed that these lines are intersecting each other at only point i.e., (16, 20). Therefore,
the length and width of the given garden is 20 m and 16 m respectively.

Q6 :

Given the linear equation 2x + 3y - 8 = 0, write another linear equations in two variables such that the
geometrical representation of the pair so formed is:

(i) intersecting lines (ii) parallel lines

(iii) coincident lines

Answer :

(i)Intersecting lines:

For this condition,

The second line such that it is intersecting the given line

is .

(ii) Parallel lines:

For this condition,


Hence, the second line can be

4x + 6y - 8 = 0

(iii)Coincident lines:

For coincident lines,

Hence, the second line can be

6x + 9y - 24 = 0

Q7 :

Draw the graphs of the equations x - y + 1 = 0 and 3x + 2y - 12 = 0. Determine the coordinates of the vertices
of the triangle formed by these lines and the x-axis, and shade the triangular region.

Answer :

x-y+1=0

x=y-1

x 0 1 2
y 1 2 3
3x + 2y - 12 = 0

x 4 2 0
y 0 3 6
Hence, the graphic representation is as follows.
From the figure, it can be observed that these lines are intersecting each other at point (2, 3) and x-axis at ( - 1, 0)
and (4, 0). Therefore, the vertices of the triangle are (2, 3), ( - 1, 0), and (4, 0).

Exercise 3.3 : Solutions of Questions on Page Number : 53


Q1 :

Solve the following pair of linear equations by the substitution method.

Answer :

(i) x + y = 14 (1)

x - y = 4 (2)

From (1), we obtain

x = 14 - y (3)

Substituting this value in equation (2), we obtain


Substituting this in equation (3), we obtain

(ii)

From (1), we obtain

Substituting this value in equation (2), we obtain

Substituting in equation (3), we obtain

s=9

s = 9, t = 6

(iii)3x - y = 3 (1)

9x - 3y = 9 (2)

From (1), we obtain

y = 3x - 3 (3)

Substituting this value in equation (2), we obtain

9=9

This is always true.

Hence, the given pair of equations has infinite possible solutions and the relation between these variables can be
given by

y = 3x - 3
Therefore, one of its possible solutions is x = 1, y = 0.

(iv)

From equation (1), we obtain

Substituting this value in equation (2), we obtain

Substituting this value in equation (3), we obtain

(v)

From equation (1), we obtain

Substituting this value in equation (2), we obtain


Substituting this value in equation (3), we obtain

Q2 :

Solve 2x + 3y = 11 and 2x - 4y = - 24 and hence find the value of 'm' for which y = mx + 3.

Answer :

From equation (1), we obtain

Substituting this value in equation (2), we obtain

Putting this value in equation (3), we obtain

Hence, x = - 2, y = 5

Also,

Q3 :

Solve 2x + 3y = 11 and 2x - 4y = - 24 and hence find the value of 'm' for which y = mx + 3.

Answer :
From equation (1), we obtain

Substituting this value in equation (2), we obtain

Putting this value in equation (3), we obtain

Hence, x = - 2, y = 5

Also,

Q4 :

Answer :

(i) Let the first number be x and the other number be y such that y > x.

According to the given information,

On substituting the value of y from equation (1) into equation (2), we obtain

Substituting this in equation (1), we obtain

y = 39
Hence, the numbers are 13 and 39.

(ii) Let the larger angle be x and smaller angle be y.

We know that the sum of the measures of angles of a supplementary pair is always 180 º.

According to the given information,

From (1), we obtain

x = 180 º - y (3)

Substituting this in equation (2), we obtain

Putting this in equation (3), we obtain

x = 180 º - 81 º

= 99 º

Hence, the angles are 99 º and 81 º.

(iii) Let the cost of a bat and a ball be x and y respectively.

According to the given information,

From (1), we obtain

Substituting this value in equation (2), we obtain


Substituting this in equation (3), we obtain

Hence, the cost of a bat is Rs 500 and that of a ball is Rs 50.

(iv) Let the fixed charge be Rs x and per km charge be Rs y.

According to the given information,

From (1), we obtain

Substituting this in equation (2), we obtain

Putting this in equation (3), we obtain

Hence, fixed charge = Rs 5

And per km charge = Rs 10

Charge for 25 km = x + 25y

= 5 + 250 = Rs 255

(v) Let the fraction be .

According to the given information,


From equation (1), we obtain

Exercise 3.4 : Solutions of Questions on Page Number : 56


Q1 :

Solve the following pair of linear equations by the elimination method and the substitution method:

Answer :

(i) By elimination method

Multiplying equation (1) by 2, we obtain

Subtracting equation (2) from equation (3), we obtain

Substituting the value in equation (1), we obtain


By substitution method

From equation (1), we obtain

(5)

Putting this value in equation (2), we obtain

- 5y = - 6

Substituting the value in equation (5), we obtain

(ii) By elimination method

Multiplying equation (2) by 2, we obtain

Adding equation (1) and (3), we obtain

Substituting in equation (1), we obtain

Hence, x = 2, y = 1

By substitution method

From equation (2), we obtain

(5)

Putting this value in equation (1), we obtain

7y = 7
Substituting the value in equation (5), we obtain

(iii) By elimination method

Q2 :

Form the pair of linear equations in the following problems, and find their solutions (if they exist) by the
elimination method:

(i) If we add 1 to the numerator and subtract 1 from the denominator, a fraction reduces to 1. It becomes if
we only add 1 to the denominator. What is the fraction?

(ii) Five years ago, Nuri was thrice as old as Sonu. Ten years later, Nuri will be twice as old as Sonu. How old
are Nuri and Sonu?

(iii) The sum of the digits of a two-digit number is 9. Also, nine times this number is twice the number
obtained by reversing the order of the digits. Find the number.

(iv) Meena went to bank to withdraw Rs 2000. She asked the cashier to give her Rs 50 and Rs 100 notes only.
Meena got 25 notes in all. Find how many notes of Rs 50 and Rs 100 she received.

(v) A lending library has a fixed charge for the first three days and an additional charge for each day
thereafter. Saritha paid Rs 27 for a book kept for seven days, while Susy paid Rs 21 for the book she kept for
five days. Find the fixed charge and the charge for each extra day.

Answer :

(i)Let the fraction be .

According to the given information,

Subtracting equation (1) from equation (2), we obtain

x = 3 (3)

Substituting this value in equation (1), we obtain


Hence, the fraction is .

(ii)Let present age of Nuri = x

and present age of Sonu = y

According to the given information,

Subtracting equation (1) from equation (2), we obtain

y = 20 (3)

Substituting it in equation (1), we obtain

Hence, age of Nuri = 50 years

And, age of Sonu = 20 years

(iii)Let the unit digit and tens digits of the number be x and y respectively. Then, number = 10y + x

Number after reversing the digits = 10x + y

According to the given information,

x + y = 9 (1)

9(10y + x) = 2(10x + y)

88y - 11x = 0

- x + 8y =0 (2)

Adding equation (1) and (2), we obtain

9y = 9

y = 1 (3)

Substituting the value in equation (1), we obtain

x=8

Hence, the number is 10y + x = 10 × 1 + 8 = 18

(iv)Let the number of Rs 50 notes and Rs 100 notes be x and y respectively.


According to the given information,

Multiplying equation (1) by 50, we obtain

Subtracting equation (3) from equation (2), we obtain

Substituting in equation (1), we have x = 10

Hence, Meena has 10 notes of Rs 50 and 15 notes of Rs 100.

(v)Let the fixed charge for first three days and each day charge thereafter be Rs x and Rs y respectively.

According to the given information,

Subtracting equation (2) from equation (1), we obtain

Substituting in equation (1), we obtain

Hence, fixed charge = Rs 15

And Charge per day = Rs 3

Exercise 3.5 : Solutions of Questions on Page Number : 62


Q1 :

Which of the following pairs of linear equations has unique solution, no solution or infinitely many
solutions? In case there is a unique solution, find it by using cross multiplication method.

Answer :
Therefore, the given sets of lines are parallel to each other. Therefore, they will not intersect each other and thus,
there will not be any solution for these equations.

Therefore, they will intersect each other at a unique point and thus, there will be a unique solution for these
equations.

By cross-multiplication method,

∴ x = 2, y = 1

Therefore, the given sets of lines will be overlapping each other i.e., the lines will be coincident to each other and
thus, there are infinite solutions possible for these equations.
Therefore, they will intersect each other at a unique point and thus, there will be a unique solution for these
equations.

By cross-multiplication,

Q2 :

(i) For which values of a and b will the following pair of linear equations have an

infinite number of solutions?

(ii) For which value of k will the following pair of linear equations have no solution?

Answer :

For infinitely many solutions,


Subtracting (1) from (2), we obtain

Substituting this in equation (2), we obtain

Hence, a = 5 and b = 1 are the values for which the given equations give infinitely many solutions.

For no solution,

Hence, for k = 2, the given equation has no solution.

Q3 :

Solve the following pair of linear equations by the substitution and cross-multiplication methods:
Answer :

From equation (ii), we obtain

Substituting this value in equation (i), we obtain

Substituting this value in equation (ii), we obtain

Hence,

Again, by cross-multiplication method, we obtain

Q4 :

Form the pair of linear equations in the following problems and find their solutions (if they exist) by any
algebraic method:
(i)A part of monthly hostel charges is fixed and the remaining depends on the number of days one has taken
food in the mess. When a student A takes food for 20 days she has to pay Rs 1000 as hostel charges
whereas a student B, who takes food for 26 days, pays Rs 1180 as hostel charges. Find the fixed charges and
the cost of food per day.

(ii)A fraction becomes when 1 is subtracted from the numerator and it becomes when 8 is added to its
denominator. Find the fraction.

(iii)Yash scored 40 marks in a test, getting 3 marks for each right answer and losing 1 mark for each wrong
answer. Had 4 marks been awarded for each correct answer and 2 marks been deducted for each incorrect
answer, then Yash would have scored 50 marks. How many questions were there in the test?

(iv) Places A and B are 100 km apart on a highway. One car starts from A and another from B at the same
time. If the cars travel in the same direction at different speeds, they meet in 5 hours. If they travel towards
each other, they meet in 1 hour. What are the speeds of the two cars?

(v)The area of a rectangle gets reduced by 9 square units, if its length is reduced by 5 units and breadth is
increased by 3 units. If we increase the length by 3 units and the breadth by 2 units, the area increases by 67
square units. Find the dimensions of the rectangle.

Answer :

(i)Let x be the fixed charge of the food and y be the charge for food per day.

According to the given information,

Subtracting equation (1) from equation (2), we obtain

Substituting this value in equation (1), we obtain

Hence, fixed charge = Rs 400

And charge per day = Rs 30

(ii)Let the fraction be .

According to the given information,


Subtracting equation (1) from equation (2), we obtain

Putting this value in equation (1), we obtain

Hence, the fraction is .

(iii)Let the number of right answers and wrong answers be x and y

respectively.

According to the given information,

Subtracting equation (2) from equation (1), we obtain

x = 15 (3)

Substituting this in equation (2), we obtain

Therefore, number of right answers = 15

And number of wrong answers = 5

Total number of questions = 20

(iv)Let the speed of 1st car and 2nd car be u km/h and v km/h.

Respective speed of both cars while they are travelling in same direction = ( ) km/h

Respective speed of both cars while they are travelling in opposite directions i.e., travelling towards each other = (
) km/h

According to the given information,


Adding both the equations, we obtain

Substituting this value in equation (2), we obtain

v = 40 km/h

Hence, speed of one car = 60 km/h and speed of other car = 40 km/h

(v) Let length and breadth of rectangle be x unit and y unit respectively.

Area = xy

According to the question,

By cross-multiplication method, we obtain

Exercise 3.6 : Solutions of Questions on Page Number : 67


Q1 :

Solve the following pairs of equations by reducing them to a pair of linear equations:
Answer :

Let and , then the equations change as follows.

Using cross-multiplication method, we obtain


Putting and in the given equations, we obtain

Multiplying equation (1) by 3, we obtain

6p + 9q = 6 (3)

Adding equation (2) and (3), we obtain

Putting in equation (1), we obtain

Hence,
Substituting in the given equations, we obtain

By cross-multiplication, we obtain

Putting and in the given equation, we obtain

Multiplying equation (1) by 3, we obtain

Q2 :

Formulate the following problems as a pair of equations, and hence find their solutions:

(i) Ritu can row downstream 20 km in 2 hours, and upstream 4 km in 2 hours. Find her speed of rowing in still
water and the speed of the current.
(ii) 2 women and 5 men can together finish an embroidery work in 4 days, while 3 women and 6 men can
finish it in 3 days. Find the time taken by 1 woman alone to finish the work, and also that taken by 1 man
alone.

(iii) Roohi travels 300 km to her home partly by train and partly by bus. She takes 4 hours if she travels 60 km
by train and remaining by bus. If she travels 100 km by train and the remaining by bus, she takes 10 minutes
longer. Find the speed of the train and the bus separately.

Answer :

(i)Let the speed of Ritu in still water and the speed of stream be x km/h

and y km/h respectively.

Speed of Ritu while rowing

Upstream = km/h

Downstream = km/h

According to question,

Adding equation (1) and (2), we obtain

Putting this in equation (1), we obtain

y=4

Hence, Ritu's speed in still water is 6 km/h and the speed of the current is 4 km/h.

(ii)Let the number of days taken by a woman and a man be x and y respectively.

Therefore, work done by a woman in 1 day =

Work done by a man in 1 day =

According to the question,


Putting in these equations, we obtain

By cross-multiplication, we obtain

Hence, number of days taken by a woman = 18

Number of days taken by a man = 36

(iii) Let the speed of train and bus be u km/h and v km/h respectively.

According to the given information,


Putting and in these equations, we obtain

Multiplying equation (3) by 10, we obtain

Subtracting equation (4) from (5), we obtain

Substituting in equation (3), we obtain

Hence, speed of train = 60 km/h

Speed of bus = 80 km/h

Exercise 3.7 : Solutions of Questions on Page Number : 68


Q1 :

The ages of two friends Ani and Biju differ by 3 years. Ani's father Dharam is twice as old as Ani and Biju is
twice as old as his sister Cathy. The ages of Cathy and Dharam differs by 30 years. Find the ages of Ani and
Biju.

Answer :

The difference between the ages of Biju and Ani is 3 years. Either Biju is 3 years older than Ani or Ani is 3 years older
than Biju. However, it is obvious that in both cases, Ani's father's age will be 30 years more than that of Cathy's age.
Let the age of Ani and Biju be x and y years respectively.

Therefore, age of Ani's father, Dharam = 2 × x = 2x years

And age of Biju's sister Cathy years

By using the information given in the question,

Case (I) When Ani is older than Biju by 3 years,

x - y = 3 (i)

4x - y = 60 (ii)

Subtracting (i) from (ii), we obtain

3x = 60 - 3 = 57

Therefore, age of Ani = 19 years

And age of Biju = 19 - 3 = 16 years

Case (II) When Biju is older than Ani,

y - x = 3 (i)

4x - y = 60 (ii)

Adding (i) and (ii), we obtain

3x = 63

x = 21

Therefore, age of Ani = 21 years

And age of Biju = 21 + 3 = 24 years

Q2 :

One says, "Give me a hundred, friend! I shall then become twice as rich as you". The other replies, "If you
give me ten, I shall be six times as rich as you". Tell me what is the amount of their (respective) capital?
[From the Bijaganita of Bhaskara II)

[Hint: x + 100 = 2 (y - 100), y + 10 = 6(x - 10)]

Answer :
Let those friends were having Rs x and y with them.

Using the information given in the question, we obtain

x + 100 = 2(y - 100)

x + 100 = 2y - 200

x - 2y = -300 (i)

And, 6(x - 10) = (y + 10)

6x - 60 = y + 10

6x - y = 70 (ii)

Multiplying equation (ii) by 2, we obtain

12x - 2y = 140 (iii)

Subtracting equation (i) from equation (iii), we obtain

11x = 140 + 300

11x = 440

x = 40

Using this in equation (i), we obtain

40 - 2y = -300

40 + 300 = 2y

2y = 340

y = 170

Therefore, those friends had Rs 40 and Rs 170 with them respectively.

Q3 :

A train covered a certain distance at a uniform speed. If the train would have been 10 km/h faster, it would
have taken 2 hours less than the scheduled time. And if the train were slower by 10 km/h; it would have taken
3 hours more than the scheduled time. Find the distance covered by the train.

Answer :

Let the speed of the train be x km/h and the time taken by train to travel the given distance be t hours and the
distance to travel was d km. We know that,

Or, d = xt (i)

Using the information given in the question, we obtain


By using equation (i), we obtain

- 2x + 10t = 20 (ii)

By using equation (i), we obtain

3x - 10t = 30 (iii)

Adding equations (ii) and (iii), we obtain

x = 50

Using equation (ii), we obtain

( - 2) × (50) + 10t = 20

- 100 + 10t = 20

10t = 120

t = 12 hours

From equation (i), we obtain

Distance to travel = d = xt

= 50 × 12

= 600 km

Hence, the distance covered by the train is 600 km.

Q4 :

The students of a class are made to stand in rows. If 3 students are extra in a row, there would be 1 row less.
If 3 students are less in a row, there would be 2 rows more. Find the number of students in the class.

Answer :

Let the number of rows be x and number of students in a row be y.

Total students of the class

= Number of rows x Number of students in a row

= xy
Using the information given in the question,

Condition 1

Total number of students = (x - 1) (y + 3)

xy = (x - 1) (y + 3) = xy - y + 3x - 3

3x - y - 3 = 0

3x - y = 3 (i)

Condition 2

Total number of students = (x + 2) (y - 3)

xy = xy + 2y - 3x - 6

3x - 2y = -6 (ii)

Subtracting equation (ii) from (i),

(3x - y) - (3x - 2y) = 3 - (-6)

- y + 2y = 3 + 6

y=9

By using equation (i), we obtain

3x - 9 = 3

3x = 9 + 3 = 12

x=4

Number of rows = x = 4

Number of students in a row = y = 9

Number of total students in a class = xy = 4 x 9 = 36

Q5 :

In a ΔABC, ∠ C = 3 ∠ B = 2 (∠ A + ∠ B). Find the three angles.

Answer :

Given that,

∠ C = 3∠ B = 2(∠ A + ∠ B)

3∠ B = 2(∠ A + ∠ B)

3∠ B = 2∠ A + 2∠ B

∠ B = 2∠ A

2 ∠ A - ∠ B = 0 … (i)

We know that the sum of the measures of all angles of a triangle is 180°. Therefore,

∠ A + ∠ B + ∠ C = 180°
∠ A + ∠ B + 3 ∠ B = 180°

∠ A + 4 ∠ B = 180° … (ii)

Multiplying equation (i) by 4, we obtain

8 ∠ A - 4 ∠ B = 0 … (iii)

Adding equations (ii) and (iii), we obtain

9 ∠ A = 180°

∠ A = 20°

From equation (ii), we obtain

20° + 4 ∠ B = 180°

4 ∠ B = 160°

∠ B = 40°

∠C=3∠B

= 3 x 40° = 120°

Therefore, ∠ A, ∠ B, ∠ C are 20°, 40°, and 120° respectively.

Q6 :

Draw the graphs of the equations 5x - y = 5 and 3x - y = 3. Determine the co-ordinates of the vertices of the
triangle formed by these lines and the y axis.

Answer :

5x - y = 5

Or, y = 5x - 5

The solution table will be as follows.

x 0 1 2
y -5 0 5
3x - y = 3

Or, y = 3x - 3

The solution table will be as follows.

x 0 1 2
y -3 0 3
The graphical representation of these lines will be as follows.
It can be observed that the required triangle is ΔABC formed by these lines and y-axis.

The coordinates of vertices are A (1, 0), B (0, - 3), C (0, - 5).

Q7 :

Solve the following pair of linear equations.

(i) px + qy = p - q

qx - py = p + q

(ii) ax + by = c

bx + ay = 1 + c

(iii)

ax + by = a2 + b2

(iv) (a - b) x + (a + b) y = a2 - 2ab - b2

(a + b) (x + y) = a2 + b2

(v) 152x - 378y = - 74

- 378x + 152y = - 604

Answer :

(i)px + qy = p - q … (1)

qx - py = p + q … (2)

Multiplying equation (1) by p and equation (2) by q, we obtain

p2x + pqy = p2 - pq … (3)


q2x - pqy = pq + q2 … (4)

Adding equations (3) and (4), we obtain

p2x + q2 x = p2 + q2

(p2 + q2) x = p2 + q2

From equation (1), we obtain

p (1) + qy = p - q

qy = - q

y=-1

(ii)ax + by = c … (1)

bx + ay = 1 + c … (2)

Multiplying equation (1) by a and equation (2) by b, we obtain

a2x + aby = ac … (3)

b2x + aby = b + bc … (4)

Subtracting equation (4) from equation (3),

(a2 - b2) x = ac - bc - b

From equation (1), we obtain

ax + by = c
(iii)

Or, bx - ay = 0 … (1)

ax + by = a2 + b2 … (2)

Multiplying equation (1) and (2) by b and a respectively, we obtain

b2x - aby = 0 … (3)

a2x + aby = a3 + ab2 … (4)

Adding equations (3) and (4), we obtain

b2x + a2x = a3 + ab2

x (b2 + a2) = a (a2 + b2)

x=a

By using (1), we obtain

b (a) - ay = 0

ab - ay = 0

ay = ab

y=b

(iv) (a - b) x + (a + b) y = a2 - 2ab - b2 … (1)

(a + b) (x + y) = a2 + b2

(a + b) x + (a + b) y = a2 + b2 … (2)

Subtracting equation (2) from (1), we obtain

(a - b) x - (a + b) x = (a2 - 2ab - b2) - (a2 + b2)

(a - b - a - b) x = - 2ab - 2b2

- 2bx = - 2b (a + b)

x=a+b

Using equation (1), we obtain

(a - b) (a + b) + (a + b) y = a2 - 2ab - b2

a2 - b2 + (a + b) y = a2 - 2ab - b2

(a + b) y = - 2ab

(v) 152x - 378y = - 74

76x - 189y = - 37
Q8 :

ABCD is a cyclic quadrilateral finds the angles of the cyclic quadrilateral.

Answer :

We know that the sum of the measures of opposite angles in a cyclic quadrilateral is 180°.

Therefore, ∠ A + ∠ C = 180

4y + 20 - 4x = 180

- 4x + 4y = 160

x - y = - 40 (i)

Also, ∠ B + ∠ D = 180

3y - 5 - 7x + 5 = 180

- 7x + 3y = 180 (ii)

Multiplying equation (i) by 3, we obtain

3x - 3y = - 120 (iii)

Adding equations (ii) and (iii), we obtain

- 7x + 3x = 180 - 120

- 4x = 60

x = -15

By using equation (i), we obtain

x - y = - 40

-15 - y = - 40

y = -15 + 40 = 25

∠ A = 4y + 20 = 4(25) + 20 = 120°

∠ B = 3y - 5 = 3(25) - 5 = 70°

∠ C = - 4x = - 4(- 15) = 60°


∠ D = - 7x + 5 = - 7(-15) + 5 = 110°
NCERT Solutions for Class 10 Maths Unit 4
Quadratic Equations Class 10
Unit 4 Quadratic Equations Exercise 4.1, 4.2, 4.3, 4.3 4.4, 4.4 Solutions
Exercise 4.1 : Solutions of Questions on Page Number : 73
Q1 :

Check whether the following are quadratic equations:

Answer :

It is of the form .

Hence, the given equation is a quadratic equation.

It is of the form .

Hence, the given equation is a quadratic equation.

It is not of the form .

Hence, the given equation is not a quadratic equation.

It is of the form .

Hence, the given equation is a quadratic equation.

It is of the
form .

Hence, the given equation is a quadratic equation.


It is not of the form .

Hence, the given equation is not a quadratic equation.

It is not of the
form .

Hence, the given equation is not a quadratic equation.

It is of
the form .

Hence, the given equation is a quadratic equation.

Q2 :

Represent the following situations in the form of quadratic equations.

(i) The area of a rectangular plot is 528 m2. The length of the plot (in metres) is one more than twice its
breadth. We need to find the length and breadth of the plot.

(ii) The product of two consecutive positive integers is 306. We need to find the integers.

(iii) Rohan's mother is 26 years older than him. The product of their ages (in years) 3 years from now will be
360. We would like to find Rohan's present age.

(iv) A train travels a distance of 480 km at a uniform speed. If the speed had been 8 km/h less, then it would
have taken 3 hours more to cover the same distance. We need to find the speed of the train.

Answer :

(i) Let the breadth of the plot be x m.

Hence, the length of the plot is (2x + 1) m.

Area of a rectangle = Length x Breadth

∴ 528 = x (2x + 1)

(ii) Let the consecutive integers be x and x + 1.

It is given that their product is 306.

(iii) Let Rohan’s age be x.

Hence, his mother’s age = x + 26

3 years hence,

Rohan’s age = x + 3

Mother’s age = x + 26 + 3 = x + 29
It is given that the product of their ages after 3 years is 360.

(iv) Let the speed of train be x km/h.

Time taken to travel 480 km =

In second condition, let the speed of train = km/h

It is also given that the train will take 3 hours to cover the same distance.

Therefore, time taken to travel 480 km = hrs

Speed x Time = Distance

⇒480+3x-3840x-24=480
⇒3x-3840x=24
⇒3x2-24x-3840=0
⇒x2-8x-1280=0

Exercise 4.2 : Solutions of Questions on Page Number : 76


Q1 :

Find the roots of the following quadratic equations by factorisation:

Answer :

Roots of this equation are the values for which =0


∴ = 0 or =0

i.e., x = 5 or x = - 2

Roots of this equation are the values for which =0

∴ = 0 or =0

i.e., x = - 2 or x =

Roots of this equation are the values for which =0

∴ = 0 or =0

i.e., x = or x =

Roots of this equation are the values for which =0


Therefore,

i.e.,

Roots of this equation are the values for which =0

Therefore,

i.e.,

Q2 :

(i) John and Jivanti together have 45 marbles. Both of them lost 5 marbles each, and the product of the
number of marbles they now have is 124. Find out how many marbles they had to start with.

(ii) A cottage industry produces a certain number of toys in a day. The cost of production of each toy (in
rupees) was found to be 55 minus the number of toys produced in a day. On a particular day, the total cost of
production was Rs 750. Find out the number of toys produced on that day.

Answer :

(i) Let the number of John's marbles be x.

Therefore, number of Jivanti's marble = 45 - x

After losing 5 marbles,

Number of John's marbles = x - 5

Number of Jivanti's marbles = 45 - x - 5 = 40 - x

It is given that the product of their marbles is 124.


Either = 0 or x - 9 = 0

i.e., x = 36 or x = 9

If the number of John's marbles = 36,

Then, number of Jivanti's marbles = 45 - 36 = 9

If number of John's marbles = 9,

Then, number of Jivanti's marbles = 45 - 9 = 36

(ii) Let the number of toys produced be x.

∴ Cost of production of each toy = Rs (55 - x)

It is given that, total production of the toys = Rs 750

Either = 0 or x - 30 = 0

i.e., x = 25 or x = 30

Hence, the number of toys will be either 25 or 30.

Q3 :

Find two numbers whose sum is 27 and product is 182.

Answer :

Let the first number be x and the second number is 27 - x.

Therefore, their product = x (27 - x)

It is given that the product of these numbers is 182.


Either = 0 or x - 14 = 0

i.e., x = 13 or x = 14

If first number = 13, then

Other number = 27 - 13 = 14

If first number = 14, then

Other number = 27 - 14 = 13

Therefore, the numbers are 13 and 14.

Q4 :

Find two consecutive positive integers, sum of whose squares is 365.

Answer :

Let the consecutive positive integers be x and x + 1.

Either x + 14 = 0 or x - 13 = 0, i.e., x = - 14 or x = 13

Since the integers are positive, x can only be 13.

∴ x + 1 = 13 + 1 = 14

Therefore, two consecutive positive integers will be 13 and 14.

Q5 :

The altitude of a right triangle is 7 cm less than its base. If the hypotenuse is 13 cm, find the other two sides.

Answer :

Let the base of the right triangle be x cm.

Its altitude = (x - 7) cm
Either x - 12 = 0 or x + 5 = 0, i.e., x = 12 or x = - 5

Since sides are positive, x can only be 12.

Therefore, the base of the given triangle is 12 cm and the altitude of this triangle will be (12 - 7) cm = 5 cm.

Q6 :

A cottage industry produces a certain number of pottery articles in a day. It was observed on a particular day
that the cost of production of each article (in rupees) was 3 more than twice the number of articles produced
on that day. If the total cost of production on that day was Rs 90, find the number of articles produced and
the cost of each article.

Answer :

Let the number of articles produced be x.

Therefore, cost of production of each article = Rs (2x + 3)

It is given that the total production is Rs 90.

Either 2x + 15 = 0 or x - 6 = 0, i.e., x = or x = 6

As the number of articles produced can only be a positive integer, therefore, x can only be 6.

Hence, number of articles produced = 6


Cost of each article = 2 × 6 + 3 = Rs 15

Exercise 4.3 : Solutions of Questions on Page Number : 87


Q1 :

Find the roots of the following quadratic equations, if they exist, by the method of completing the square:

Answer :
Q2 :

Find the roots of the quadratic equations given in Q.1 above by applying the quadratic formula.

Answer :
Exercise 4.3 4.4 : Solutions of Questions on Page Number : 88
Q1 :

Find the nature of the roots of the following quadratic equations.

If the real roots exist, find them;

(I) 2x2 - 3x + 5 = 0

(II)

(III) 2x2 - 6x + 3 = 0

Answer :
We know that for a quadratic equation ax2 + bx + c = 0, discriminant is b2 - 4ac.

(A) If b2 - 4ac > 0 → two distinct real roots

(B) If b2 - 4ac = 0 → two equal real roots

(C) If b2 - 4ac < 0 → no real roots

(I) 2x2 - 3x + 5 = 0

Comparing this equation with ax2 + bx + c = 0, we obtain

a = 2, b = - 3, c = 5

Discriminant = b2 - 4ac = ( - 3)2 - 4 (2) (5) = 9 - 40

= - 31

As b2 - 4ac < 0,

Therefore, no real root is possible for the given equation.

(II)

Comparing this equation with ax2 + bx + c = 0, we obtain

Discriminant

= 48 - 48 = 0

As b2 - 4ac = 0,

Therefore, real roots exist for the given equation and they are equal to each other.

And the roots will be and .

Therefore, the roots are and .

(III) 2x2 - 6x + 3 = 0

Comparing this equation with ax2 + bx + c = 0, we obtain

a = 2, b = - 6, c = 3

Discriminant = b2 - 4ac = ( - 6)2 - 4 (2) (3)

= 36 - 24 = 12

As b2 - 4ac > 0,

Therefore, distinct real roots exist for this equation as follows.


Therefore, the roots are or .

Q2 :

Find the roots of the following equations:

Answer :
Q3 :

The sum of the reciprocals of Rehman's ages, (in years) 3 years ago and 5 years from now is . Find his
present age.

Answer :

Let the present age of Rehman be x years.

Three years ago, his age was (x - 3) years.

Five years hence, his age will be (x + 5) years.

It is given that the sum of the reciprocals of Rehman's ages 3 years ago and 5 years from now is .
However, age cannot be negative.

Therefore, Rehman's present age is 7 years.

Q4 :

In a class test, the sum of Shefali's marks in Mathematics and English is 30. Had she got 2 marks more in
Mathematics and 3 marks less in English, the product of their marks would have been 210. Find her marks in
the two subjects.

Answer :

Let the marks in Maths be x.

Then, the marks in English will be 30 - x.

According to the given question,

If the marks in Maths are 12, then marks in English will be 30 - 12 = 18


If the marks in Maths are 13, then marks in English will be 30 - 13 = 17

Q5 :

The diagonal of a rectangular field is 60 metres more than the shorter side. If the longer side is 30 metres
more than the shorter side, find the sides of the field.

Answer :

Let the shorter side of the rectangle be x m.

Then, larger side of the rectangle = (x + 30) m

However, side cannot be negative. Therefore, the length of the shorter side will be

90 m.

Hence, length of the larger side will be (90 + 30) m = 120 m

Q6 :

The difference of squares of two numbers is 180. The square of the smaller number is 8 times the larger
number. Find the two numbers.

Answer :

Let the larger and smaller number be x and y respectively.

According to the given question,


However, the larger number cannot be negative as 8 times of the larger number will be negative and hence, the
square of the smaller number will be negative which is not possible.

Therefore, the larger number will be 18 only.

Therefore, the numbers are 18 and 12 or 18 and - 12.

Q7 :

A train travels 360 km at a uniform speed. If the speed had been 5 km/h more, it would have taken 1 hour less
for the same journey. Find the speed of the train.

Answer :

Let the speed of the train be x km/hr.

Time taken to cover 360 km hr

According to the given question,


However, speed cannot be negative.

Therefore, the speed of train is 40 km/h

Q8 :

Two water taps together can fill a tank in hours. The tap of larger diameter takes 10 hours less than the
smaller one to fill the tank separately. Find the time in which each tap can separately fill the tank.

Answer :

Let the time taken by the smaller pipe to fill the tank be x hr.

Time taken by the larger pipe = (x - 10) hr

Part of tank filled by smaller pipe in 1 hour =

Part of tank filled by larger pipe in 1 hour =

It is given that the tank can be filled in hours by both the pipes together. Therefore,
Time taken by the smaller pipe cannot be = 3.75 hours. As in this case, the time taken by the larger pipe will be
negative, which is logically not possible.

Therefore, time taken individually by the smaller pipe and the larger pipe will be 25 and 25 - 10 =15 hours
respectively.

Q9 :

An express train takes 1 hour less than a passenger train to travel 132 km between Mysore and Bangalore
(without taking into consideration the time they stop at intermediate stations). If the average speeds of the
express train is 11 km/h more than that of the passenger train, find the average speed of the two trains.

Answer :

Let the average speed of passenger train be x km/h.

Average speed of express train = (x + 11) km/h

It is given that the time taken by the express train to cover 132 km is 1 hour less than the passenger train to cover the
same distance.
Speed cannot be negative.

Therefore, the speed of the passenger train will be 33 km/h and thus, the speed of the express train will be 33 + 11 =
44 km/h.

Q10 :

Sum of the areas of two squares is 468 m2. If the difference of their perimeters is 24 m, find the sides of the
two squares.

Answer :

Let the sides of the two squares be x m and y m. Therefore, their perimeter will be 4x and 4y respectively and their
areas will be x2 and y2 respectively.

It is given that

4x - 4y = 24

x-y=6

x=y+6
However, side of a square cannot be negative.

Hence, the sides of the squares are 12 m and (12 + 6) m = 18 m

Exercise 4.4 : Solutions of Questions on Page Number : 91


Q1 :

Find the values of k for each of the following quadratic equations, so that they have two equal roots.

(I) 2x2 + kx + 3 = 0

(II) kx (x - 2) + 6 = 0

Answer :

We know that if an equation ax2 + bx + c = 0 has two equal roots, its discriminant
(b2 - 4ac) will be 0.

(I) 2x2 + kx + 3 = 0

Comparing equation with ax2 + bx + c = 0, we obtain

a = 2, b = k, c = 3

Discriminant = b2 - 4ac = (k)2 - 4(2) (3)

= k2 - 24

For equal roots,

Discriminant = 0

k2 - 24 = 0

k2 = 24

(II) kx (x - 2) + 6 = 0

or kx2 - 2kx + 6 = 0
Comparing this equation with ax2 + bx + c = 0, we obtain

a = k, b = - 2k, c = 6

Discriminant = b2 - 4ac = ( - 2k)2 - 4 (k) (6)

= 4k2 - 24k

For equal roots,

b2 - 4ac = 0

4k2 - 24k = 0

4k (k - 6) = 0

Either 4k = 0 or k = 6 = 0

k = 0 or k = 6

However, if k = 0, then the equation will not have the terms 'x2' and 'x'.

Therefore, if this equation has two equal roots, k should be 6 only.

Q2 :

Is it possible to design a rectangular mango grove whose length is twice its breadth, and the area is 800 m2?

If so, find its length and breadth.

Answer :

Let the breadth of mango grove be l.

Length of mango grove will be 2l.

Area of mango grove = (2l) (l)

= 2l2

Comparing this equation with al2 + bl + c = 0, we obtain

a = 1 b = 0, c = 400

Discriminant = b2 - 4ac = (0)2 - 4 × (1) × ( - 400) = 1600

Here, b2 - 4ac > 0

Therefore, the equation will have real roots. And hence, the desired rectangular mango grove can be designed.

However, length cannot be negative.

Therefore, breadth of mango grove = 20 m


Length of mango grove = 2 × 20 = 40 m

Q3 :

Is the following situation possible? If so, determine their present ages. The sum of the ages of two friends is
20 years. Four years ago, the product of their ages in years was 48.

Answer :

Let the age of one friend be x years.

Age of the other friend will be (20 - x) years.

4 years ago, age of 1st friend = (x - 4) years

And, age of 2nd friend = (20 - x - 4)

= (16 - x) years

Given that,

(x - 4) (16 - x) = 48

16x - 64 - x2 + 4x = 48

- x2 + 20x - 112 = 0

x2 - 20x + 112 = 0

Comparing this equation with ax2 + bx + c = 0, we obtain

a = 1, b = -20, c = 112

Discriminant = b2 - 4ac = (- 20)2 - 4 (1) (112)

= 400 - 448 = -48

As b2 - 4ac < 0,

Therefore, no real root is possible for this equation and hence, this situation is not possible.

Q4 :

Is it possible to design a rectangular park of perimeter 80 and area 400 m2? If so find its length and breadth.

Answer :

Let the length and breadth of the park be l and b.

Perimeter = 2 (l + b) = 80

l + b = 40

Or, b = 40 - l

Area = l × b = l (40 - l) = 40l - l2


40l - l2 = 400

l2 - 40l + 400 = 0

Comparing this equation with

al2 + bl + c = 0, we obtain

a = 1, b = - 40, c = 400

Discriminate = b2 - 4ac = ( - 40)2 - 4 (1) (400)

= 1600 - 1600 = 0

As b2 - 4ac = 0,

Therefore, this equation has equal real roots. And hence, this situation is possible.

Root of this equation,

Therefore, length of park, l = 20 m

And breadth of park, b = 40 - l = 40 - 20 = 20 m


Arithmetic Progressions 

Exercise 5.1 
Question 1: 

In which of the following situations, does the list of numbers involved make as 
arithmetic progression and why? 

(i) The taxi fare after each km when the fare is Rs 15 for the first km and Rs 8 for each 
additional km. 

(ii) The amount of air present in a cylinder when a vacuum pump removes of the air 
remaining in the cylinder at a time. 

(iii) The cost of digging a well after every metre of digging, when it costs Rs 150 for the 
first metre and rises by Rs 50 for each subsequent metre. 

(iv)The amount of money in the account every year, when Rs 10000 is deposited at 
compound interest at 8% per annum. 

Answer: 

(i) It can be observed that 

Taxi fare for 1st km = 15 

Taxi fare for first 2 km = 15 + 8 = 23 

Taxi fare for first 3 km = 23 + 8 = 31 

Taxi fare for first 4 km = 31 + 8 = 39 

Clearly 15, 23, 31, 39 … forms an A.P. because every term is 8 more than the preceding 
term. 

(ii) Let the initial volume of air in a cylinder be V lit. In each stroke, the vacuum pump 

removes of air remaining in the cylinder at a time. In other words, after every stroke, 

only part of air will remain. 


Therefore, volumes will be  

Clearly, it can be observed that the adjacent terms of this series do not have the same 
difference between them. Therefore, this is not an A.P. 

(iii) Cost of digging for first metre = 150 

Cost of digging for first 2 metres = 150 + 50 = 200 

Cost of digging for first 3 metres = 200 + 50 = 250 

Cost of digging for first 4 metres = 250 + 50 = 300 

Clearly, 150, 200, 250, 300 … forms an A.P. because every term is 50 more than the 
preceding term. 

(iv) We know that if Rs P is deposited at r% compound interest per annum for n years, 

our money will be after n years. 

Therefore, after every year, our money will be 

Clearly, adjacent terms of this series do not have the same difference between them. 
Therefore, this is not an A.P. 

Question 2: 
Write first four terms of the A.P. when the first term a and the common difference d are 
given as follows 

(i) a = 10, d = 10 

(ii) a = − 2, d = 0 

(iii) a = 4, d = − 3 

(iv) a = − 1 d =  

(v) a = − 1.25, d = − 0.25 


Answer: 

(i) a = 10, d = 10 

Let the series be a1, a2, a3, a4, a5 … 

a1 = a = 10 

a2 = a1 + d = 10 + 10 = 20 

a3 = a2 + d = 20 + 10 = 30 

a4 = a3 + d = 30 + 10 = 40 

a5 = a4 + d = 40 + 10 = 50 

Therefore, the series will be 10, 20, 30, 40, 50 … 

First four terms of this A.P. will be 10, 20, 30, and 40. 

(ii) a = −2, d = 0 

Let the series be a1, a2, a3, a4 … 

a1 = a = −2 

a2 = a1 + d = − 2 + 0 = −2 

a3 = a2 + d = − 2 + 0 = −2 

a4 = a3 + d = − 2 + 0 = −2 

Therefore, the series will be −2, −2, −2, −2 … 

First four terms of this A.P. will be −2, −2, −2 and −2. 

(iii) a = 4, d = −3 
Let the series be a1, a2, a3, a4 … 

a1 = a = 4 

a2 = a1 + d = 4 − 3 = 1 

a3 = a2 + d = 1 − 3 = −2 

a4 = a3 + d = − 2 − 3 = −5 

Therefore, the series will be 4, 1, −2 −5 … 

First four terms of this A.P. will be 4, 1, −2 and −5. 

(iv) a = −1, d =  

Let the series be a1, a2, a3, a4 … 

Clearly, the series will be 

…………. 

First four terms of this A.P. will be . 

(v) a = −1.25, d = −0.25 

Let the series be a1, a2, a3, a4 … 


a1 = a = −1.25 

a2 = a1 + d = − 1.25 − 0.25 = −1.50 

a3 = a2 + d = − 1.50 − 0.25 = −1.75 

a4 = a3 + d = − 1.75 − 0.25 = −2.00 

Clearly, the series will be 1.25, −1.50, −1.75, −2.00 …….. 

First four terms of this A.P. will be −1.25, −1.50, −1.75 and −2.00. 

Question 3: 
For the following A.P.s, write the first term and the common difference. 

(i) 3, 1, − 1, − 3 … 

(ii) − 5, − 1, 3, 7 … 

(iii)  

(iv) 0.6, 1.7, 2.8, 3.9 … 

Answer: 

(i) 3, 1, −1, −3 … 

Here, first term, a = 3 

Common difference, d = Second term − First term 

= 1 − 3 = −2 

(ii) −5, −1, 3, 7 … 

Here, first term, a = −5 

Common difference, d = Second term − First term 

= (−1) − (−5) = − 1 + 5 = 4 
(iii)  

Here, first term,  

Common difference, d = Second term − First term 

(iv) 0.6, 1.7, 2.8, 3.9 … 

Here, first term, a = 0.6 

Common difference, d = Second term − First term 

= 1.7 − 0.6 

= 1.1 

Question 4: 
Which of the following are APs? If they form an A.P. find the common difference d and 
write three more terms. 

(i) 2, 4, 8, 16 … 

(ii)  

(iii) − 1.2, − 3.2, − 5.2, − 7.2 … 

(iv) − 10, − 6, − 2, 2 … 

(v)  

(vi) 0.2, 0.22, 0.222, 0.2222 …. 

(vii) 0, − 4, − 8, − 12 … 

(viii)  

(ix) 1, 3, 9, 27 … 

(x) a, 2a, 3a, 4a … 


(xi) a, a2, a3, a4 … (xii)  

(xiii)  

(xiv) 12, 32, 52, 72 … 

(xv) 12, 52, 72, 73 … 

Answer: 

(i) 2, 4, 8, 16 … 

It can be observed that 

a2 − a1 = 4 − 2 = 2 

a3 − a2 = 8 − 4 = 4 

a4 − a3 = 16 − 8 = 8 

i.e., ak+1− ak is not the same every time. Therefore, the given numbers are not forming an 
A.P. 

(ii)  

It can be observed that 

i.e., ak+1− ak is same every time. 

Therefore, and the given numbers are in A.P. 


Three more terms are 

(iii) −1.2, −3.2, −5.2, −7.2 … 

It can be observed that 

a2 − a1 = (−3.2) − (−1.2) = −2 

a3 − a2 = (−5.2) − (−3.2) = −2 

a4 − a3 = (−7.2) − (−5.2) = −2 

i.e., ak+1− ak is same every time. Therefore, d = −2 

The given numbers are in A.P. 

Three more terms are 

a5 = − 7.2 − 2 = −9.2 

a6 = − 9.2 − 2 = −11.2 

a7 = − 11.2 − 2 = −13.2 

(iv) −10, −6, −2, 2 … 

It can be observed that 

a2 − a1 = (−6) − (−10) = 4 

a3 − a2 = (−2) − (−6) = 4 
a4 − a3 = (2) − (−2) = 4 

i.e., ak+1 − ak is same every time. Therefore, d = 4 

The given numbers are in A.P. 

Three more terms are 

a5 = 2 + 4 = 6 

a6 = 6 + 4 = 10 

a7 = 10 + 4 = 14 

(v)  

It can be observed that 

i.e., ak+1 − ak is same every time. Therefore,  

The given numbers are in A.P. 

Three more terms are 

(vi) 0.2, 0.22, 0.222, 0.2222 …. 

It can be observed that 


a2 − a1 = 0.22 − 0.2 = 0.02 

a3 − a2 = 0.222 − 0.22 = 0.002 

a4 − a3 = 0.2222 − 0.222 = 0.0002 

i.e., ak+1 − ak is not the same every time. 

Therefore, the given numbers are not in A.P. 

(vii) 0, −4, −8, −12 … 

It can be observed that 

a2 − a1 = (−4) − 0 = −4 

a3 − a2 = (−8) − (−4) = −4 

a4 − a3 = (−12) − (−8) = −4 

i.e., ak+1 − ak is same every time. Therefore, d = −4 

The given numbers are in A.P. 

Three more terms are 

a5 = − 12 − 4 = −16 

a6 = − 16 − 4 = −20 

a7 = − 20 − 4 = −24 

(viii)  

It can be observed that 


 

i.e., ak+1 − ak is same every time. Therefore, d = 0 

The given numbers are in A.P. 

Three more terms are 

(ix) 1, 3, 9, 27 … 

It can be observed that 

a2 − a1 = 3 − 1 = 2 

a3 − a2 = 9 − 3 = 6 

a4 − a3 = 27 − 9 = 18 

i.e., ak+1 − ak is not the same every time. 

Therefore, the given numbers are not in A.P. 

(x) a, 2a, 3a, 4a … 

It can be observed that 


a2 − a1 = 2a − a = a 

a3 − a2 = 3a − 2a = a 

a4 − a3 = 4a − 3a = a 

i.e., ak+1 − ak is same every time. Therefore, d = a 

The given numbers are in A.P. 

Three more terms are 

a5 = 4a + a = 5a 

a6 = 5a + a = 6a 

a7 = 6a + a = 7a 

(xi) a, a2, a3, a4 … 

It can be observed that 

a2 − a1 = a2 − a = a (a − 1) 

a3 − a2 = a3 − a2 = a2 (a − 1) 

a4 − a3 = a4 − a3 = a3 (a − 1) 

i.e., ak+1 − ak is not the same every time. 

Therefore, the given numbers are not in A.P. 

(xii)  

It can be observed that 


 

i.e., ak+1 − ak is same every time. 

Therefore, the given numbers are in A.P. 

And,  

Three more terms are 

(xiii)  

It can be observed that 

i.e., ak+1 − ak is not the same every time. 

Therefore, the given numbers are not in A.P. 

(xiv) 12, 32, 52, 72 … 

Or, 1, 9, 25, 49 ….. 

It can be observed that 

a2 − a1 = 9 − 1 = 8 
a3 − a2 = 25 − 9 = 16 

a4 − a3 = 49 − 25 = 24 

i.e., ak+1 − ak is not the same every time. 

Therefore, the given numbers are not in A.P. 

(xv) 12, 52, 72, 73 … 

Or 1, 25, 49, 73 … 

It can be observed that 

a2 − a1 = 25 − 1 = 24 

a3 − a2 = 49 − 25 = 24 

a4 − a3 = 73 − 49 = 24 

i.e., ak+1 − ak is same every time. 

Therefore, the given numbers are in A.P. 

And, d = 24 

Three more terms are 

a5 = 73+ 24 = 97 

a6 = 97 + 24 = 121 

a7 = 121 + 24 = 145 

 
Exercise 5.2
Question 1: 

Fill in the blanks in the following table, given that a is the first term, d the common 
difference and an the nth term of the A.P. 
  a  d  n  an 

I  7  3  8  …… 

II  − 18  …..  10  0 

III  …..  − 3  18  − 5 

IV  − 18.9  2.5  …..  3.6 

V  3.5  0  105  ….. 

Answer: 

I. a = 7, d = 3, n = 8, an = ? 

We know that, 

For an A.P. an = a + (n − 1) d 

= 7 + (8 − 1) 3 

= 7 + (7) 3 

= 7 + 21 = 28 

Hence, an = 28 

II. Given that 

a = −18, n = 10, an = 0, d = ? 

We know that, 
an = a + (n − 1) d 

0 = − 18 + (10 − 1) d 

18 = 9d 

Hence, common difference, d = 2 

III. Given that 

d = −3, n = 18, an = −5 

We know that, 

an = a + (n − 1) d 

−5 = a + (18 − 1) (−3) 

−5 = a + (17) (−3) 

−5 = a − 51 

a = 51 − 5 = 46 

Hence, a = 46 

IV. a = −18.9, d = 2.5, an = 3.6, n = ? 

We know that, 

an = a + (n − 1) d 

3.6 = − 18.9 + (n − 1) 2.5 

3.6 + 18.9 = (n − 1) 2.5 


22.5 = (n − 1) 2.5 

Hence, n = 10 

V. a = 3.5, d = 0, n = 105, an = ? 

We know that, 

an = a + (n − 1) d 

an = 3.5 + (105 − 1) 0 

an = 3.5 + 104 × 0 

an = 3.5 

Hence, an = 3.5 

Question 2: 
Choose the correct choice in the following and justify 

I. 30th term of the A.P: 10, 7, 4, …, is 

A. 97 B. 77 C. − 77 D. − 87 

II 11th term of the A.P. is 

A. 28 B. 22 C. − 38 D.  

Answer: 

I. Given that 
A.P. 10, 7, 4, … 

First term, a = 10 

Common difference, d = a2 − a1 = 7 − 10 

= −3 

We know that, an = a + (n − 1) d 

a30 = 10 + (30 − 1) (−3) 

a30 = 10 + (29) (−3) 

a30 = 10 − 87 = −77 

Hence, the correct answer is C. 

II. Given that, A.P.  

First term a = −3 

Common difference, d = a2 − a1 

We know that, 
 

Hence, the answer is B. 

Question 3: 
In the following APs find the missing term in the boxes 

I.  

II.  

III.  

IV.  

V.  

Answer: 

I.  

For this A.P., 

a = 2 

a3 = 26 

We know that, an = a + (n − 1) d 

a3 = 2 + (3 − 1) d 
26 = 2 + 2d 

24 = 2d 

d = 12 

a2 = 2 + (2 − 1) 12 

= 14 

Therefore, 14 is the missing term. 

II.  

For this A.P., 

a2 = 13 and 

a4 = 3 

We know that, an = a + (n − 1) d 

a2 = a + (2 − 1) d 

13 = a + d (I) 

a4 = a + (4 − 1) d 

3 = a + 3d (II) 

On subtracting (I) from (II), we obtain 

−10 = 2d 

d = −5 

From equation (I), we obtain 


13 = a + (−5) 

a = 18 

a3 = 18 + (3 − 1) (−5) 

= 18 + 2 (−5) = 18 − 10 = 8 

Therefore, the missing terms are 18 and 8 respectively. 

III.  

For this A.P., 

We know that, 

Therefore, the missing terms are and 8 respectively. 


IV.  

For this A.P., 

a = −4 and 

a6 = 6 

We know that, 

an = a + (n − 1) d 

a6 = a + (6 − 1) d 

6 = − 4 + 5d 

10 = 5d 

d = 2 

a2 = a + d = − 4 + 2 = −2 

a3 = a + 2d = − 4 + 2 (2) = 0 

a4 = a + 3d = − 4 + 3 (2) = 2 

a5 = a + 4d = − 4 + 4 (2) = 4 

Therefore, the missing terms are −2, 0, 2, and 4 respectively. 

V.  

For this A.P., 

a2 = 38 

a6 = −22 
We know that 

an = a + (n − 1) d 

a2 = a + (2 − 1) d 

38 = a + d (1) 

a6 = a + (6 − 1) d 

−22 = a + 5d (2) 

On subtracting equation (1) from (2), we obtain 

− 22 − 38 = 4d 

−60 = 4d 

d = −15 

a = a2 − d = 38 − (−15) = 53 

a3 = a + 2d = 53 + 2 (−15) = 23 

a4 = a + 3d = 53 + 3 (−15) = 8 

a5 = a + 4d = 53 + 4 (−15) = −7 

Therefore, the missing terms are 53, 23, 8, and −7 respectively. 

Question 4: 
Which term of the A.P. 3, 8, 13, 18, … is 78? 

Answer: 

3, 8, 13, 18, … 

For this A.P., 


a = 3 

d = a2 − a1 = 8 − 3 = 5 

Let nth term of this A.P. be 78. 

an = a + (n − 1) d 

78 = 3 + (n − 1) 5 

75 = (n − 1) 5 

(n − 1) = 15 

n = 16 

Hence, 16th term of this A.P. is 78. 

Question 5: 
Find the number of terms in each of the following A.P. 

I. 7, 13, 19, …, 205 

II.  

Answer: 

I. 7, 13, 19, …, 205 

For this A.P., 

a = 7 

d = a2 − a1 = 13 − 7 = 6 

Let there are n terms in this A.P. 


an = 205 

We know that 

an = a + (n − 1) d 

Therefore, 205 = 7 + (n − 1) 6 

198 = (n − 1) 6 

33 = (n − 1) 

n = 34 

Therefore, this given series has 34 terms in it. 

II.  

For this A.P., 

Let there are n terms in this A.P. 

Therefore, an = −47 and we know that, 


 

Therefore, this given A.P. has 27 terms in it. 

Question 6: 
Check whether − 150 is a term of the A.P. 11, 8, 5, 2, … 

Answer: 

For this A.P., 

a = 11 

d = a2 − a1 = 8 − 11 = −3 

Let −150 be the nth term of this A.P. 

We know that, 

Clearly, n is not an integer. 

Therefore, −150 is not a term of this A.P. 


 

Question 7: 

Find the 31st term of an A.P. whose 11th term is 38 and the 16th term is 73 

Answer: 

Given that, 

a11 = 38 

a16 = 73 

We know that, 

an = a + (n − 1) d 

a11 = a + (11 − 1) d 

38 = a + 10d (1) 

Similarly, 

a16 = a + (16 − 1) d 

73 = a + 15d (2) 

On subtracting (1) from (2), we obtain 

35 = 5d 

d = 7 

From equation (1), 

38 = a + 10 × (7) 

38 − 70 = a 

a = −32 
a31 = a + (31 − 1) d 

= − 32 + 30 (7) 

= − 32 + 210 

= 178 

Hence, 31st term is 178. 

Question 8: 

An A.P. consists of 50 terms of which 3rd term is 12 and the last term is 106. Find the 
29th term 

Answer: 

Given that, 

a3 = 12 

a50 = 106 

We know that, 

an = a + (n − 1) d 

a3 = a + (3 − 1) d 

12 = a + 2d (I) 

Similarly, a50 = a + (50 − 1) d 

106 = a + 49d (II) 

On subtracting (I) from (II), we obtain 

94 = 47d 
d = 2 

From equation (I), we obtain 

12 = a + 2 (2) 

a = 12 − 4 = 8 

a29 = a + (29 − 1) d 

a29 = 8 + (28)2 

a29 = 8 + 56 = 64 

Therefore, 29th term is 64. 

Question 9: 

If the 3rd and the 9th terms of an A.P. are 4 and − 8 respectively. Which term of this A.P. 
is zero. 

Answer: 

Given that, 

a3 = 4 

a9 = −8 

We know that, 

an = a + (n − 1) d 

a3 = a + (3 − 1) d 

4 = a + 2d (I) 

a9 = a + (9 − 1) d 
−8 = a + 8d (II) 

On subtracting equation (I) from (II), we obtain 

−12 = 6d 

d = −2 

From equation (I), we obtain 

4 = a + 2 (−2) 

4 = a − 4 

a = 8 

Let nth term of this A.P. be zero. 

an = a + (n − 1) d 

0 = 8 + (n − 1) (−2) 

0 = 8 − 2n + 2 

2n = 10 

n = 5 

Hence, 5th term of this A.P. is 0. 

Question 10: 

If 17th term of an A.P. exceeds its 10th term by 7. Find the common difference. 

Answer: 

We know that, 

For an A.P., an = a + (n − 1) d 
a17 = a + (17 − 1) d 

a17 = a + 16d 

Similarly, a10 = a + 9d 

It is given that 

a17 − a10 = 7 

(a + 16d) − (a + 9d) = 7 

7d = 7 

d = 1 

Therefore, the common difference is 1. 

Question 11: 

Which term of the A.P. 3, 15, 27, 39, … will be 132 more than its 54th term? 

Answer: 

Given A.P. is 3, 15, 27, 39, … 

a = 3 

d = a2 − a1 = 15 − 3 = 12 

a54 = a + (54 − 1) d 

= 3 + (53) (12) 

= 3 + 636 = 639 

132 + 639 = 771 

We have to find the term of this A.P. which is 771. 


Let nth term be 771. 

an = a + (n − 1) d 

771 = 3 + (n − 1) 12 

768 = (n − 1) 12 

(n − 1) = 64 

n = 65 

Therefore, 65th term was 132 more than 54th term. 

Alternatively, 

Let nth term be 132 more than 54th term. 

Question 12: 

Two APs have the same common difference. The difference between their 100th term 
is 100, what is the difference between their 1000th terms? 

Answer: 

Let the first term of these A.P.s be a1 and a2 respectively and the common difference of 
these A.P.s be d. 

For first A.P., 

a100 = a1 + (100 − 1) d 

= a1 + 99d 

a1000 = a1 + (1000 − 1) d 
a1000 = a1 + 999d 

For second A.P., 

a100 = a2 + (100 − 1) d 

= a2 + 99d 

a1000 = a2 + (1000 − 1) d 

= a2 + 999d 

Given that, difference between 

100th term of these A.P.s = 100 

Therefore, (a1 + 99d) − (a2 + 99d) = 100 

a1 − a2 = 100 (1) 

Difference between 1000th terms of these A.P.s 

(a1 + 999d) − (a2 + 999d) = a1 − a2 

From equation (1), 

This difference, a1 − a2 = 100 

Hence, the difference between 1000th terms of these A.P. will be 100. 

Question 13: 

How many three digit numbers are divisible by 7 

Answer: 

First three-digit number that is divisible by 7 = 105 

Next number = 105 + 7 = 112 


Therefore, 105, 112, 119, … 

All are three digit numbers which are divisible by 7 and thus, all these are terms of an 
A.P. having first term as 105 and common difference as 7. 

The maximum possible three-digit number is 999. When we divide it by 7, the remainder 
will be 5. Clearly, 999 − 5 = 994 is the maximum possible three-digit number that is 
divisible by 7. 

The series is as follows. 

105, 112, 119, …, 994 

Let 994 be the nth term of this A.P. 

a = 105 

d = 7 

an = 994 

n = ? 

an = a + (n − 1) d 

994 = 105 + (n − 1) 7 

889 = (n − 1) 7 

(n − 1) = 127 

n = 128 

Therefore, 128 three-digit numbers are divisible by 7. 

Question 14: 

How many multiples of 4 lie between 10 and 250? 

Answer: 
First multiple of 4 that is greater than 10 is 12. Next will be 16. 

Therefore, 12, 16, 20, 24, … 

All these are divisible by 4 and thus, all these are terms of an A.P. with first term as 12 
and common difference as 4. 

When we divide 250 by 4, the remainder will be 2. Therefore, 250 − 2 = 248 is divisible by 
4. 

The series is as follows. 

12, 16, 20, 24, …, 248 

Let 248 be the nth term of this A.P. 

Therefore, there are 60 multiples of 4 between 10 and 250. 

Question 15: 

For what value of n, are the nth terms of two APs 63, 65, 67, and 3, 10, 17, … equal 

Answer: 

63, 65, 67, … 

a = 63 

d = a2 − a1 = 65 − 63 = 2 
nth term of this A.P. = an = a + (n − 1) d 

an= 63 + (n − 1) 2 = 63 + 2n − 2 

an = 61 + 2n (1) 

3, 10, 17, … 

a = 3 

d = a2 − a1 = 10 − 3 = 7 

nth term of this A.P. = 3 + (n − 1) 7 

an = 3 + 7n − 7 

an = 7n − 4 (2) 

It is given that, nth term of these A.P.s are equal to each other. 

Equating both these equations, we obtain 

61 + 2n = 7n − 4 

61 + 4 = 5n 

5n = 65 

n = 13 

Therefore, 13th terms of both these A.P.s are equal to each other. 

Question 16: 

Determine the A.P. whose third term is 16 and the 7th term exceeds the 5th term by 12. 

Answer: 

=a3 = 16 
a + (3 − 1) d = 16 

a + 2d = 16 (1) 

a7 − a5 = 12 

[a+ (7 − 1) d] − [a + (5 − 1) d]= 12 

(a + 6d) − (a + 4d) = 12 

2d = 12 

d = 6 

From equation (1), we obtain 

a + 2 (6) = 16 

a + 12 = 16 

a = 4 

Therefore, A.P. will be 

4, 10, 16, 22, … 

Question 17: 

Find the 20th term from the last term of the A.P. 3, 8, 13, …, 253 

Answer: 

Given A.P. is 

3, 8, 13, …, 253 

Common difference for this A.P. is 5. 

Therefore, this A.P. can be written in reverse order as 


253, 248, 243, …, 13, 8, 3 

For this A.P., 

a = 253 

d = 248 − 253 = −5 

n = 20 

a20 = a + (20 − 1) d 

a20 = 253 + (19) (−5) 

a20 = 253 − 95 

a = 158 

Therefore, 20th term from the last term is 158. 

Question 18: 

The sum of 4th and 8th terms of an A.P. is 24 and the sum of the 6th and 10th terms is 
44. Find the first three terms of the A.P. 

Answer: 

We know that, 

an = a + (n − 1) d 

a4 = a + (4 − 1) d 

a4 = a + 3d 

Similarly, 

a8 = a + 7d 
a6 = a + 5d 

a10 = a + 9d 

Given that, a4 + a8 = 24 

a + 3d + a + 7d = 24 

2a + 10d = 24 

a + 5d = 12 (1) 

a6 + a10 = 44 

a + 5d + a + 9d = 44 

2a + 14d = 44 

a + 7d = 22 (2) 

On subtracting equation (1) from (2), we obtain 

2d = 22 − 12 

2d = 10 

d = 5 

From equation (1), we obtain 

a + 5d = 12 

a + 5 (5) = 12 

a + 25 = 12 

a = −13 

a2 = a + d = − 13 + 5 = −8 
a3 = a2 + d = − 8 + 5 = −3 

Therefore, the first three terms of this A.P. are −13, −8, and −3. 

Question 19: 

Subba Rao started work in 1995 at an annual salary of Rs 5000 and received an 
increment of Rs 200 each year. In which year did his income reach Rs 7000? 

Answer: 

It can be observed that the incomes that Subba Rao obtained in various years are in A.P. 
as every year, his salary is increased by Rs 200. 

Therefore, the salaries of each year after 1995 are 

5000, 5200, 5400, … 

Here, a = 5000 

d = 200 

Let after nth year, his salary be Rs 7000. 

Therefore, an = a + (n − 1) d 

7000 = 5000 + (n − 1) 200 

200(n − 1) = 2000 

(n − 1) = 10 

n = 11 

Therefore, in 11th year, his salary will be Rs 7000. 

Question 20: 

Ramkali saved Rs 5 in the first week of a year and then increased her weekly saving by 
Rs 1.75. If in the nth week, her week, her weekly savings become Rs 20.75, find n. 
Answer: 

Given that, 

a = 5 

d = 1.75 

an = 20.75 

n = ? 

an = a + (n − 1) d 

n − 1 = 9 

n = 10 

Hence, n is 10. 

 
Exercise 5.3 
 

Question 1: 

Find the sum of the following APs. 

(i) 2, 7, 12 ,…., to 10 terms. 

(ii) − 37, − 33, − 29 ,…, to 12 terms 

(iii) 0.6, 1.7, 2.8 ,…….., to 100 terms 

(iv) ,………, to 11 terms 

Answer: 

(i)2, 7, 12 ,…, to 10 terms 

For this A.P., 

a = 2 

d = a2 − a1 = 7 − 2 = 5 

n = 10 

We know that, 

(ii)−37, −33, −29 ,…, to 12 terms 

For this A.P., 

a = −37 
d = a2 − a1 = (−33) − (−37) 

= − 33 + 37 = 4 

n = 12 

We know that, 

(iii) 0.6, 1.7, 2.8 ,…, to 100 terms 

For this A.P., 

a = 0.6 

d = a2 − a1 = 1.7 − 0.6 = 1.1 

n = 100 

We know that, 

(iv) . …….. , to 11 terms 


For this A.P., 

n = 11 

We know that, 

Question 2: 
Find the sums given below 

(i) 7 + + 14 + ………… + 84 

(ii) 34 + 32 + 30 + ……….. + 10 

(iii) − 5 + (− 8) + (− 11) + ………… + (− 230) 

Answer: 

(i)7 + + 14 + …………+ 84 

For this A.P., 

a = 7 
l = 84 

Let 84 be the nth term of this A.P. 

l = a + (n − 1)d 

22 = n − 1 

n = 23 

We know that, 

(ii)34 + 32 + 30 + ……….. + 10 

For this A.P., 

a = 34 

d = a2 − a1 = 32 − 34 = −2 

l = 10 
Let 10 be the nth term of this A.P. 

l = a + (n − 1) d 

10 = 34 + (n − 1) (−2) 

−24 = (n − 1) (−2) 

12 = n − 1 

n = 13 

(iii)(−5) + (−8) + (−11) + ………… + (−230) 

For this A.P., 

a = −5 

l = −230 

d = a2 − a1 = (−8) − (−5) 

= − 8 + 5 = −3 

Let −230 be the nth term of this A.P. 

l = a + (n − 1)d 

−230 = − 5 + (n − 1) (−3) 
−225 = (n − 1) (−3) 

(n − 1) = 75 

n = 76 

And,  

Question 3: 
In an AP 

(i) Given a = 5, d = 3, an = 50, find n and Sn. 

(ii) Given a = 7, a13 = 35, find d and S13. 

(iii) Given a12 = 37, d = 3, find a and S12. 

(iv) Given a3 = 15, S10 = 125, find d and a10. 

(v) Given d = 5, S9 = 75, find a and a9. 

(vi) Given a = 2, d = 8, Sn = 90, find n and an. 

(vii) Given a = 8, an = 62, Sn = 210, find n and d. 

(viii) Given an = 4, d = 2, Sn = − 14, find n and a. 

(ix) Given a = 3, n = 8, S = 192, find d. 

(x)Given l = 28, S = 144 and there are total 9 terms. Find a. 

Answer: 

(i) Given that, a = 5, d = 3, an = 50 

As an = a + (n − 1)d, 

∴ 50 = 5 + (n − 1)3 
45 = (n − 1)3 

15 = n − 1 

n = 16 

(ii) Given that, a = 7, a13 = 35 

As an = a + (n − 1) d, 

∴ a13 = a + (13 − 1) d 

35 = 7 + 12 d 

35 − 7 = 12d 

28 = 12d 

(iii)Given that, a12 = 37, d = 3 


As an = a + (n − 1)d, 

a12 = a + (12 − 1)3 

37 = a + 33 

a = 4 

(iv) Given that, a3 = 15, S10 = 125 

As an = a + (n − 1)d, 

a3 = a + (3 − 1)d 

15 = a + 2d (i) 

On multiplying equation (1) by 2, we obtain 

30 = 2a + 4d (iii) 

On subtracting equation (iii) from (ii), we obtain 

−5 = 5d 
d = −1 

From equation (i), 

15 = a + 2(−1) 

15 = a − 2 

a = 17 

a10 = a + (10 − 1)d 

a10 = 17 + (9) (−1) 

a10 = 17 − 9 = 8 

(v)Given that, d = 5, S9 = 75 

As , 

25 = 3(a + 20) 

25 = 3a + 60 

3a = 25 − 60 

an = a + (n − 1)d 

a9 = a + (9 − 1) (5) 
 

(vi) Given that, a = 2, d = 8, Sn = 90 

As , 

90 = n [2 + (n − 1)4] 

90 = n [2 + 4n − 4] 

90 = n (4n − 2) = 4n2 − 2n 

4n2 − 2n − 90 = 0 

4n2 − 20n + 18n − 90 = 0 

4n (n − 5) + 18 (n − 5) = 0 

(n − 5) (4n + 18) = 0 

Either n − 5 = 0 or 4n + 18 = 0 

n = 5 or  

However, n can neither be negative nor fractional. 

Therefore, n = 5 
an = a + (n − 1)d 

a5 = 2 + (5 − 1)8 

= 2 + (4) (8) 

= 2 + 32 = 34 

(vii) Given that, a = 8, an = 62, Sn = 210 

n = 6 

an = a + (n − 1)d 

62 = 8 + (6 − 1)d 

62 − 8 = 5d 

54 = 5d 

(viii) Given that, an = 4, d = 2, Sn = −14 

an = a + (n − 1)d 

4 = a + (n − 1)2 

4 = a + 2n − 2 
a + 2n = 6 

a = 6 − 2n (i) 

−28 = n (a + 4) 

−28 = n (6 − 2n + 4) {From equation (i)} 

−28 = n (− 2n + 10) 

−28 = − 2n2 + 10n 

2n2 − 10n − 28 = 0 

n2 − 5n −14 = 0 

n2 − 7n + 2n − 14 = 0 

n (n − 7) + 2(n − 7) = 0 

(n − 7) (n + 2) = 0 

Either n − 7 = 0 or n + 2 = 0 

n = 7 or n = −2 

However, n can neither be negative nor fractional. 

Therefore, n = 7 

From equation (i), we obtain 

a = 6 − 2n 
a = 6 − 2(7) 

= 6 − 14 

= −8 

(ix)Given that, a = 3, n = 8, S = 192 

192 = 4 [6 + 7d] 

48 = 6 + 7d 

42 = 7d 

d = 6 

(x)Given that, l = 28, S = 144 and there are total of 9 terms. 

(16) × (2) = a + 28 

32 = a + 28 

a = 4 

Question 4: 
How many terms of the AP. 9, 17, 25 … must be taken to give a sum of 636? 

Answer: 
Let there be n terms of this A.P. 

For this A.P., a = 9 

d = a2 − a1 = 17 − 9 = 8 

636 = n [9 + 4n − 4] 

636 = n (4n + 5) 

4n2 + 5n − 636 = 0 

4n2 + 53n − 48n − 636 = 0 

n (4n + 53) − 12 (4n + 53) = 0 

(4n + 53) (n − 12) = 0 

Either 4n + 53 = 0 or n − 12 = 0 

or n = 12 

n cannot be . As the number of terms can neither be negative nor fractional, 


therefore, n = 12 only. 

Question 5: 

The first term of an AP is 5, the last term is 45 and the sum is 400. Find the number of 
terms and the common difference. 

Answer: 
Given that, 

a = 5 

l = 45 

Sn = 400 

n = 16 

l = a + (n − 1) d 

45 = 5 + (16 − 1) d 

40 = 15d 

Question 6: 
The first and the last term of an AP are 17 and 350 respectively. If the common 
difference is 9, how many terms are there and what is their sum? 

Answer: 

Given that, 

a = 17 

l = 350 

d = 9 
Let there be n terms in the A.P. 

l = a + (n − 1) d 

350 = 17 + (n − 1)9 

333 = (n − 1)9 

(n − 1) = 37 

n = 38 

Thus, this A.P. contains 38 terms and the sum of the terms of this A.P. is 6973. 

Question 7: 

Find the sum of first 22 terms of an AP in which d = 7 and 22nd term is 149. 

Answer: 

d = 7 

a22 = 149 

S22 = ? 

an = a + (n − 1)d 

a22 = a + (22 − 1)d 

149 = a + 21 × 7 

149 = a + 147 

a = 2 
 

Question 8: 
Find the sum of first 51 terms of an AP whose second and third terms are 14 and 18 
respectively. 

Answer: 

Given that, 

a2 = 14 

a3 = 18 

d = a3 − a2 = 18 − 14 = 4 

a2 = a + d 

14 = a + 4 

a = 10 

= 5610 

Question 9: 
If the sum of first 7 terms of an AP is 49 and that of 17 terms is 289, find the sum of 
first n terms. 
Answer: 

Given that, 

S7 = 49 

S17 = 289 

7 = (a + 3d) 

a + 3d = 7 (i) 

Similarly,  

17 = (a + 8d) 

a + 8d = 17 (ii) 

Subtracting equation (i) from equation (ii), 

5d = 10 

d = 2 

From equation (i), 

a + 3(2) = 7 
a + 6 = 7 

a = 1 

= n2 

  

Question 10: 

Show that a1, a2 … , an , … form an AP where an is defined as below 

(i) an = 3 + 4n 

(ii) an = 9 − 5n 

Also find the sum of the first 15 terms in each case. 

Answer: 

(i) an = 3 + 4n 

a1 = 3 + 4(1) = 7 

a2 = 3 + 4(2) = 3 + 8 = 11 

a3 = 3 + 4(3) = 3 + 12 = 15 

a4 = 3 + 4(4) = 3 + 16 = 19 

It can be observed that 


a2 − a1 = 11 − 7 = 4 

a3 − a2 = 15 − 11 = 4 

a4 − a3 = 19 − 15 = 4 

i.e., ak + 1 − ak is same every time. Therefore, this is an AP with common difference as 4 


and first term as 7. 

= 15 × 35 

= 525 

(ii) an = 9 − 5n 

a1 = 9 − 5 × 1 = 9 − 5 = 4 

a2 = 9 − 5 × 2 = 9 − 10 = −1 

a3 = 9 − 5 × 3 = 9 − 15 = −6 

a4 = 9 − 5 × 4 = 9 − 20 = −11 

It can be observed that 

a2 − a1 = − 1 − 4 = −5 

a3 − a2 = − 6 − (−1) = −5 
a4 − a3 = − 11 − (−6) = −5 

i.e., ak + 1 − ak is same every time. Therefore, this is an A.P. with common difference as 
−5 and first term as 4. 

= −465 

  

Question 11: 

If the sum of the first n terms of an AP is 4n − n2, what is the first term (that is S1)? 
What is the sum of first two terms? What is the second term? Similarly find the 3rd, 
the10th and the nth terms. 

Answer: 

Given that, 

Sn = 4n − n2 

First term, a = S1 = 4(1) − (1)2 = 4 − 1 = 3 

Sum of first two terms = S2 

= 4(2) − (2)2 = 8 − 4 = 4 

Second term, a2 = S2 − S1 = 4 − 3 = 1 
d = a2 − a = 1 − 3 = −2 

an = a + (n − 1)d 

= 3 + (n − 1) (−2) 

= 3 − 2n + 2 

= 5 − 2n 

Therefore, a3 = 5 − 2(3) = 5 − 6 = −1 

a10 = 5 − 2(10) = 5 − 20 = −15 

Hence, the sum of first two terms is 4. The second term is 1. 3rd, 10th, and nth terms are 
−1, −15, and 5 − 2n respectively. 

Question 12: 
Find the sum of first 40 positive integers divisible by 6. 

Answer: 

The positive integers that are divisible by 6 are 

6, 12, 18, 24 … 

It can be observed that these are making an A.P. whose first term is 6 and common 
difference is 6. 

a = 6 

d = 6 

S40 =? 
 

= 20[12 + (39) (6)] 

= 20(12 + 234) 

= 20 × 246 

= 4920 

Question 13: 
Find the sum of first 15 multiples of 8. 

Answer: 

The multiples of 8 are 

8, 16, 24, 32… 

These are in an A.P., having first term as 8 and common difference as 8. 

Therefore, a = 8 

d = 8 

S15 =? 

 
= 960 

Question 14: 
Find the sum of the odd numbers between 0 and 50. 

Answer: 

The odd numbers between 0 and 50 are 

1, 3, 5, 7, 9 … 49 

Therefore, it can be observed that these odd numbers are in an A.P. 

a = 1 

d = 2 

l = 49 

l = a + (n − 1) d 

49 = 1 + (n − 1)2 

48 = 2(n − 1) 

n − 1 = 24 

n = 25 

= 625 

Question 15: 
A contract on construction job specifies a penalty for delay of completion beyond a 
certain date as follows: Rs. 200 for the first day, Rs. 250 for the second day, Rs. 300 for 
the third day, etc., the penalty for each succeeding day being Rs. 50 more than for the 
preceding day. How much money the contractor has to pay as penalty, if he has delayed 
the work by 30 days. 

Answer: 

It can be observed that these penalties are in an A.P. having first term as 200 and 
common difference as 50. 

a = 200 

d = 50 

Penalty that has to be paid if he has delayed the work by 30 days = S30 

= 15 [400 + 1450] 

= 15 (1850) 

= 27750 

Therefore, the contractor has to pay Rs 27750 as penalty. 

Question 16: 
A sum of Rs 700 is to be used to give seven cash prizes to students of a school for their 
overall academic performance. If each prize is Rs 20 less than its preceding prize, find 
the value of each of the prizes. 

Answer: 

Let the cost of 1st prize be P. 

Cost of 2nd prize = P − 20 

And cost of 3rd prize = P − 40 


It can be observed that the cost of these prizes are in an A.P. having common difference 
as −20 and first term as P. 

a = P 

d = −20 

Given that, S7 = 700 

a + 3(−20) = 100 

a − 60 = 100 

a = 160 

Therefore, the value of each of the prizes was Rs 160, Rs 140, Rs 120, Rs 100, Rs 80, Rs 
60, and Rs 40. 

Question 17: 

In a school, students thought of planting trees in and around the school to reduce air 
pollution. It was decided that the number of trees, that each section of each class will 
plant, will be the same as the class, in which they are studying, e.g., a section of class I 
will plant 1 tree, a section of class II will plant 2 trees and so on till class XII. There are 
three sections of each class. How many trees will be planted by the students? 

Answer: 

It can be observed that the number of trees planted by the students is in an AP. 

1, 2, 3, 4, 5………………..12 

First term, a = 1 

Common difference, d = 2 − 1 = 1 
 

= 6 (2 + 11) 

= 6 (13) 

= 78 

Therefore, number of trees planted by 1 section of the classes = 78 

Number of trees planted by 3 sections of the classes = 3 × 78 = 234 

Therefore, 234 trees will be planted by the students. 

Question 18: 
A spiral is made up of successive semicircles, with centres alternately at A and B, 
starting with centre at A of radii 0.5, 1.0 cm, 1.5 cm, 2.0 cm, ……… as shown in figure. 
What is the total length of such a spiral made up of thirteen consecutive semicircles? 

Answer: 

Semi-perimeter of circle = πr 

I1 = π(0.5)  
I2 = π(1) = π cm 

I3 = π(1.5) =  

Therefore, I1, I2, I3 ,i.e. the lengths of the semi-circles are in an A.P., 

S13 =? 

We know that the sum of n terms of an a A.P. is given by 

= 143 

Therefore, the length of such spiral of thirteen consecutive semi-circles will be 143 cm. 

Question 19: 
200 logs are stacked in the following manner: 20 logs in the bottom row, 19 in the next 
row, 18 in the row next to it and so on. In how many rows are the 200 logs placed and 
how many logs are in the top row? 

Answer: 

It can be observed that the numbers of logs in rows are in an A.P. 

20, 19, 18… 

For this A.P., 

a = 20 

d = a2 − a1 = 19 − 20 = −1 

Let a total of 200 logs be placed in n rows. 

Sn = 200 

400 = n (40 − n + 1) 

400 = n (41 − n) 

400 = 41n − n2 

n2 − 41n + 400 = 0 

n2 − 16n − 25n + 400 = 0 


n (n − 16) −25 (n − 16) = 0 

(n − 16) (n − 25) = 0 

Either (n − 16) = 0 or n − 25 = 0 

n = 16 or n = 25 

an = a + (n − 1)d 

a16 = 20 + (16 − 1) (−1) 

a16 = 20 − 15 

a16 = 5 

Similarly, 

a25 = 20 + (25 − 1) (−1) 

a25 = 20 − 24 

= −4 

Clearly, the number of logs in 16th row is 5. However, the number of logs in 25th row is 
negative, which is not possible. 

Therefore, 200 logs can be placed in 16 rows and the number of logs in the 16th row is 5. 

Question 20: 
In a potato race, a bucket is placed at the starting point, which is 5 m from the first 
potato and other potatoes are placed 3 m apart in a straight line. There are ten potatoes 
in the line. 

 
A competitor starts from the bucket, picks up the nearest potato, runs back with it, 
drops it in the bucket, runs back to pick up the next potato, runs to the bucket to drop it 
in, and she continues in the same way until all the potatoes are in the bucket. What is 
the total distance the competitor has to run? 

[Hint: to pick up the first potato and the second potato, the total distance (in metres) 
run by a competitor is 2 × 5 + 2 ×(5 + 3)] 

Answer: 

The distances of potatoes are as follows. 

5, 8, 11, 14… 

It can be observed that these distances are in A.P. 

a = 5 

d = 8 − 5 = 3 

= 5[10 + 9 × 3] 

= 5(10 + 27) = 5(37) 

= 185 

As every time she has to run back to the bucket, therefore, the total distance that the 
competitor has to run will be two times of it. 

Therefore, total distance that the competitor will run = 2 × 185 

= 370 m 
Alternatively, 

The distances of potatoes from the bucket are 5, 8, 11, 14… 

Distance run by the competitor for collecting these potatoes are two times of the 
distance at which the potatoes have been kept. Therefore, distances to be run are 

10, 16, 22, 28, 34,………. 

a = 10 

d = 16 − 10 = 6 

S10 =? 

= 5[20 + 54] 

= 5 (74) 

= 370 

Therefore, the competitor will run a total distance of 370 m. 

 
Exercise 5.4
Question 1: 

Which term of the A.P. 121, 117, 113 … is its first negative term? 

[Hint: Find n for an < 0] 

Answer: 

Given A.P. is 121, 117, 113 … 

a = 121 

d = 117 − 121 = −4 

an = a + (n − 1) d 

= 121 + (n − 1) (−4) 

= 121 − 4n + 4 

= 125 − 4n 

We have to find the first negative term of this A.P. 

Therefore, 32nd term will be the first negative term of this A.P. 

Question 2: 

The sum of the third and the seventh terms of an A.P is 6 and their product is 8. Find 
the sum of first sixteen terms of the A.P. 

Answer: 
We know that, 

an = a + (n − 1) d 

a3 = a + (3 − 1) d 

a3 = a + 2d 

Similarly, a7 = a + 6d 

Given that, a3 + a7 = 6 

(a + 2d) + (a + 6d) = 6 

2a + 8d = 6 

a + 4d = 3 

a = 3 − 4d (i) 

Also, it is given that (a3) × (a7) = 8 

(a + 2d) × (a + 6d) = 8 

From equation (i), 

From equation (i), 


 

 
 

Question 3: 
A ladder has rungs 25 cm apart. (See figure). The rungs decrease uniformly in length 

from 45 cm at the bottom to 25 cm at the top. If the top and bottom rungs are m 
apart, what is the length of the wood required for the rungs? 

[Hint: number of rungs ] 

Answer: 

It is given that the rungs are 25 cm apart and the top and bottom rungs are m apart. 

∴ Total number of rungs  

Now, as the lengths of the rungs decrease uniformly, they will be in an A.P. 
The length of the wood required for the rungs equals the sum of all the terms of this A.P. 

First term, a = 45 

Last term, l = 25 

n = 11 

Therefore, the length of the wood required for the rungs is 385 cm. 

Question 4: 
The houses of a row are number consecutively from 1 to 49. Show that there is a value 
of x such that the sum of numbers of the houses preceding the house numbered x is 
equal to the sum of the number of houses following it. 

Find this value of x. 

[Hint Sx − 1 = S49 − Sx] 

Answer: 

The number of houses was 

1, 2, 3 … 49 

It can be observed that the number of houses are in an A.P. having a as 1 and d also as 
1. 

Let us assume that the number of xth house was like this. 

We know that, 

Sum of n terms in an A.P.  

Sum of number of houses preceding xth house = Sx − 1 


 

Sum of number of houses following xth house = S49 − Sx 

It is given that these sums are equal to each other. 

However, the house numbers are positive integers. 

The value of x will be 35 only. 

Therefore, house number 35 is such that the sum of the numbers of houses preceding 
the house numbered 35 is equal to the sum of the numbers of the houses following it. 

Question 5: 
A small terrace at a football ground comprises of 15 steps each of which is 50 m long 
and built of solid concrete. 
Each step has a rise of m and a tread of m (See figure) calculate the total volume 
of concrete required to build the terrace. 

Answer 

From the figure, it can be observed that 

1st step is m wide, 

2nd step is 1 m wide, 

3rd step is m wide. 


Therefore, the width of each step is increasing by m each time whereas their height 

m and length 50 m remains the same. 

Therefore, the widths of these steps are 

Volume of concrete in 1st step  

Volume of concrete in 2nd step  

Volume of concrete in 3rd step  

It can be observed that the volumes of concrete in these steps are in an A.P. 

 
Volume of concrete required to build the terrace is 750 m3. 

 
NCERT Solutions for Class 10 Maths Unit 6
Triangles Class 10
Unit 6 Triangles Exercise 6.1, 6.2, 6.3, 6.4, 6.5, 6.6 Solutions
Exercise 6.1 : Solutions of Questions on Page Number : 122
Q1 :

Fill in the blanks using correct word given in the brackets:-

(i) All circles are __________. (congruent, similar)

(ii) All squares are __________. (similar, congruent)

(iii) All __________ triangles are similar. (isosceles, equilateral)

(iv) Two polygons of the same number of sides are similar, if (a) their corresponding angles are __________
and (b) their corresponding sides are __________. (equal, proportional)

Answer :

(i) Similar

(ii) Similar

(iii) Equilateral

(iv) (a) Equal

(b) Proportional

Q2 :

Give two different examples of pair of

(i) Similar figures

(ii)Non-similar figures

Answer :

(i) Two equilateral triangles with sides 1 cm and 2 cm

Two squares with sides 1 cm and 2 cm


(ii) Trapezium and square

Triangle and parallelogram

Q3 :

State whether the following quadrilaterals are similar or not:

Answer :

Quadrilateral PQRS and ABCD are not similar as their corresponding sides are proportional, i.e. 1:2, but their
corresponding angles are not equal.

Exercise 6.2 : Solutions of Questions on Page Number : 128


Q1 :

In figure.6.17. (i) and (ii), DE || BC. Find EC in (i) and AD in (ii).

(i)
(ii)

Answer :

(i)

Let EC = x cm

It is given that DE || BC.

By using basic proportionality theorem, we obtain

(ii)
Let AD = x cm

It is given that DE || BC.

By using basic proportionality theorem, we obtain

Q2 :

E and F are points on the sides PQ and PR respectively of a ΔPQR. For each of the following cases, state
whether EF || QR.

(i) PE = 3.9 cm, EQ = 3 cm, PF = 3.6 cm and FR = 2.4 cm

(ii) PE = 4 cm, QE = 4.5 cm, PF = 8 cm and RF = 9 cm

(iii)PQ = 1.28 cm, PR = 2.56 cm, PE = 0.18 cm and PF = 0.63 cm

Answer :

(i)

Given that, PE = 3.9 cm, EQ = 3 cm, PF = 3.6 cm, FR = 2.4 cm


(ii)

PE = 4 cm, QE = 4.5 cm, PF = 8 cm, RF = 9 cm

(iii)
PQ = 1.28 cm, PR = 2.56 cm, PE = 0.18 cm, PF = 0.36 cm

Q3 :

In the following figure, if LM || CB and LN || CD, prove that

Answer :

In the given figure, LM || CB

By using basic proportionality theorem, we obtain


Q4 :

In the following figure, DE || AC and DF || AE. Prove that

Answer :

In ΔABC, DE || AC
Q5 :

In the following figure, DE || OQ and DF || OR, show that EF || QR.

Answer :

In Δ POQ, DE || OQ
Q6 :

In the following figure, A, B and C are points on OP, OQ and OR respectively such that AB || PQ and AC || PR.
Show that BC || QR.

Answer :
In Δ POQ, AB || PQ

Q7 :

Using Basic proportionality theorem, prove that a line drawn through the mid-points of one side of a triangle
parallel to another side bisects the third side. (Recall that you have proved it in Class IX).
Answer :

Consider the given figure in which PQ is a line segment drawn through the mid-point P of line AB, such
that

Or, Q is the mid-point of AC.

Q8 :

Using Converse of basic proportionality theorem, prove that the line joining the mid-points of any two sides
of a triangle is parallel to the third side. (Recall that you have done it in Class IX).

Answer :

Consider the given figure in which PQ is a line segment joining the mid-points P and Q of line AB and AC
respectively.

i.e., AP = PB and AQ = QC

It can be observed that


Hence, by using basic proportionality theorem, we obtain

Q9 :

ABCD is a trapezium in which AB || DC and its diagonals intersect each other at the point O. Show

that

Answer :

Draw a line EF through point O, such that

In ΔADC,

By using basic proportionality theorem, we obtain

In ΔABD,

So, by using basic proportionality theorem, we obtain

From equations (1) and (2), we obtain


Q10 :

The diagonals of a quadrilateral ABCD intersect each other at the point O such that Show that
ABCD is a trapezium.

Answer :

Let us consider the following figure for the given question.

Draw a line OE || AB

In ΔABD, OE || AB

By using basic proportionality theorem, we obtain

However, it is given that


⇒ EO || DC [By the converse of basic proportionality theorem]

⇒ AB || OE || DC

⇒ AB || CD

∴ ABCD is a trapezium.

Exercise 6.3 : Solutions of Questions on Page Number : 138


Q1 :

State which pairs of triangles in the following figure are similar? Write the similarity criterion used by you for
answering the question and also write the pairs of similar triangles in the symbolic form:

(i)

(ii)

(iii)

(iv)
(v)

(vi)

Answer :

(i) ∠A = ∠P = 60°

∠B = ∠Q = 80°

∠C = ∠R = 40°

Therefore, ΔABC ∼ ΔPQR [By AAA similarity criterion]

(ii)

(iii)The given triangles are not similar as the corresponding sides are not proportional.

(iv) In ∆MNL and ∆QPR, we observe that,


MNQP = MLQR = 12

Q2 :

In the following figure, ΔODC ∝¼ ΔOBA, ∠ BOC = 125° and ∠ CDO = 70°. Find ∠ DOC, ∠ DCO and ∠ OAB
Answer :

DOB is a straight line.

∴ ∠ DOC + ∠ COB = 180°

⇒ ∠ DOC = 180° - 125°

= 55°

In ΔDOC,

∠ DCO + ∠ CDO + ∠ DOC = 180°

(Sum of the measures of the angles of a triangle is 180º.)

⇒ ∠ DCO + 70º + 55º = 180°

⇒ ∠ DCO = 55°

It is given that ΔODC ∠¼ ΔOBA.

∴ ∠ OAB = ∠ OCD [Corresponding angles are equal in similar triangles.]

⇒ ∠ OAB = 55°

Q3 :

Diagonals AC and BD of a trapezium ABCD with AB || DC intersect each other at the point O. Using a

similarity criterion for two triangles, show that

Answer :

In ΔDOC and ΔBOA,


∠CDO = ∠ABO [Alternate interior angles as AB || CD]

∠DCO = ∠BAO [Alternate interior angles as AB || CD]

∠DOC = ∠BOA [Vertically opposite angles]

∴ ΔDOC ∼ ΔBOA [AAA similarity criterion]

Q4 :

In the following figure, Show that

Answer :

In ΔPQR, ∠PQR = ∠PRQ

∴ PQ = PR (i)

Given,
Q5 :

S and T are point on sides PR and QR of ΔPQR such that ∠ P = ∠ RTS. Show that ΔRPQ ∠¼ ΔRTS.

Answer :

In ΔRPQ and ΔRST,

∠ RTS = ∠ QPS (Given)

∠ R = ∠ R (Common angle)

∴ ΔRPQ ∝¼ ΔRTS (By AA similarity criterion)

Q6 :

In the following figure, if ΔABE ≅ ΔACD, show that ΔADE ∝¼ ΔABC.


Answer :

It is given that ΔABE ≅ ΔACD.

∴ AB = AC [By CPCT] (1)

And, AD = AE [By CPCT] (2)

In ΔADE and ΔABC,

[Dividing equation (2) by (1)]

∠A = ∠A [Common angle]

∴ ΔADE ∼ ΔABC [By SAS similarity criterion]

Q7 :

In the following figure, altitudes AD and CE of ΔABC intersect each other at the point P. Show that:

(i) ΔAEP ∝¼ ΔCDP

(ii) ΔABD ∝¼ ΔCBE

(iii) ΔAEP ∝¼ ΔADB

(v) ΔPDC ∝¼ ΔBEC

Answer :

(i)
In ΔAEP and ΔCDP,

∠ AEP = ∠ CDP (Each 90°)

∠ APE = ∠ CPD (Vertically opposite angles)

Hence, by using AA similarity criterion,

ΔAEP ∝¼ ΔCDP

(ii)

In ΔABD and ΔCBE,

∠ ADB = ∠ CEB (Each 90°)

∠ ABD = ∠ CBE (Common)

Hence, by using AA similarity criterion,

ΔABD ∝¼ ΔCBE

(iii)

In ΔAEP and ΔADB,

∠ AEP = ∠ ADB (Each 90°)

∠ PAE = ∠ DAB (Common)

Hence, by using AA similarity criterion,

ΔAEP ∝¼ ΔADB

(iv)
In ΔPDC and ΔBEC,

∠ PDC = ∠ BEC (Each 90°)

∠ PCD = ∠ BCE (Common angle)

Hence, by using AA similarity criterion,

ΔPDC ∝¼ ΔBEC

Q8 :

E is a point on the side AD produced of a parallelogram ABCD and BE intersects CD at F. Show that ΔABE
∠¼ ΔCFB

Answer :

In ΔABE and ΔCFB,

∠ A = ∠ C (Opposite angles of a parallelogram)

∠ AEB = ∠ CBF (Alternate interior angles as AE || BC)

∴ ΔABE ∝¼ ΔCFB (By AA similarity criterion)

Q9 :

In the following figure, ABC and AMP are two right triangles, right angled at B and M respectively, prove that:
(i) ΔABC ∼ ΔAMP

(ii)

Answer :

In ΔABC and ΔAMP,

∠ABC = ∠AMP (Each 90°)

∠A = ∠A (Common)

∴ ΔABC ∼ ΔAMP (By AA similarity criterion)

Q10 :

CD and GH are respectively the bisectors of ∠ACB and ∠EGF such that D and H lie on sides AB and FE of
ΔABC and ΔEFG respectively. If ΔABC ∼ ΔFEG, Show that:

(i)

(ii) ΔDCB ∼ ΔHGE

(iii) ΔDCA ∼ ΔHGF

Answer :

It is given that ΔABC ∼ ΔFEG.

∴ ∠A = ∠F, ∠B = ∠E, and ∠ACB = ∠FGE


∠ACB = ∠FGE

∴ ∠ACD = ∠FGH (Angle bisector)

And, ∠DCB = ∠HGE (Angle bisector)

In ΔACD and ΔFGH,

∠A = ∠F (Proved above)

∠ACD = ∠FGH (Proved above)

∴ ΔACD ∼ ΔFGH (By AA similarity criterion)

In ΔDCB and ΔHGE,

∠DCB = ∠HGE (Proved above)

∠B = ∠E (Proved above)

∴ ΔDCB ∼ ΔHGE (By AA similarity criterion)

In ΔDCA and ΔHGF,

∠ACD = ∠FGH (Proved above)

∠A = ∠F (Proved above)

∴ ΔDCA ∼ ΔHGF (By AA similarity criterion)

Q11 :

In the following figure, E is a point on side CB produced of an isosceles triangle ABC with AB = AC. If AD ⊥
BC and EF ⊥ AC, prove that ΔABD ∝¼ ΔECF

Answer :

It is given that ABC is an isosceles triangle.

∴ AB = AC

⇒ ∠ ABD = ∠ ECF

In ΔABD and ΔECF,


∠ ADB = ∠ EFC (Each 90°)

∠ BAD = ∠ CEF (Proved above)

∴ ΔABD ∠¼ ΔECF (By using AA similarity criterion)

Q12 :

Sides AB and BC and median AD of a triangle ABC are respectively proportional to sides PQ and QR and
median PM of ΔPQR (see the given figure). Show that ΔABC ∠¼ ΔPQR.

Answer :

Median divides the opposite side.

Given that,

In ΔABD and ΔPQM,

(Proved above)

∴ ΔABD ∼ ΔPQM (By SSS similarity criterion)

⇒ ∠ABD = ∠PQM (Corresponding angles of similar triangles)

In ΔABC and ΔPQR,

∠ABD = ∠PQM (Proved above)


∴ ΔABC ∼ ΔPQR (By SAS similarity criterion)

Q13 :

D is a point on the side BC of a triangle ABC such that ∠ADC = ∠BAC. Show that

Answer :

In ΔADC and ΔBAC,

∠ADC = ∠BAC (Given)

∠ACD = ∠BCA (Common angle)

∴ ΔADC ∼ ΔBAC (By AA similarity criterion)

We know that corresponding sides of similar triangles are in proportion.

Q14 :

Sides AB and AC and median AD of a triangle ABC are respectively proportional to sides PQ and PR and
median PM of another triangle PQR. Show that

Answer :
Given that,

Let us extend AD and PM up to point E and L respectively, such that AD = DE and PM = ML. Then, join B to E, C to
E, Q to L, and R to L.

We know that medians divide opposite sides.

Therefore, BD = DC and QM = MR

Also, AD = DE (By construction)

And, PM = ML (By construction)

In quadrilateral ABEC, diagonals AE and BC bisect each other at point D.

Therefore, quadrilateral ABEC is a parallelogram.

∴ AC = BE and AB = EC (Opposite sides of a parallelogram are equal)

Similarly, we can prove that quadrilateral PQLR is a parallelogram and PR = QL, PQ = LR

It was given that


∴ ΔABE ∼ ΔPQL (By SSS similarity criterion)

We know that corresponding angles of similar triangles are equal.

∴ ∠BAE = ∠QPL … (1)

Similarly, it can be proved that ΔAEC ∼ ΔPLR and

∠CAE = ∠RPL … (2)

Adding equation (1) and (2), we obtain

∠BAE + ∠CAE = ∠QPL + ∠RPL

⇒ ∠CAB = ∠RPQ … (3)

In ΔABC and ΔPQR,

(Given)

∠CAB = ∠RPQ [Using equation (3)]

∴ ΔABC ∼ ΔPQR (By SAS similarity criterion)

Q15 :

A vertical pole of a length 6 m casts a shadow 4m long on the ground and at the same time a tower casts a
shadow 28 m long. Find the height of the tower.

Answer :

Let AB and CD be a tower and a pole respectively.

Let the shadow of BE and DF be the shadow of AB and CD respectively.

At the same time, the light rays from the sun will fall on the tower and the pole at the same angle.

Therefore, ∠DCF = ∠BAE

And, ∠DFC = ∠BEA

∠CDF = ∠ABE (Tower and pole are vertical to the ground)

∴ ΔABE ∼ ΔCDF (AAA similarity criterion)


Therefore, the height of the tower will be 42 metres.

Q16 :

If AD and PM are medians of triangles ABC and PQR, respectively

where

Answer :

It is given that ΔABC ∼ ΔPQR

We know that the corresponding sides of similar triangles are in proportion.

∴ … (1)

Also, ∠A = ∠P, ∠B = ∠Q, ∠C = ∠R … (2)

Since AD and PM are medians, they will divide their opposite sides.

∴ … (3)

From equations (1) and (3), we obtain

… (4)

In ΔABD and ΔPQM,

∠B = ∠Q [Using equation (2)]


[Using equation (4)]

∴ ΔABD ∼ ΔPQM (By SAS similarity criterion)

Exercise 6.4 : Solutions of Questions on Page Number : 143


Q1 :

Let and their areas be, respectively, 64 cm2 and 121 cm2. If EF = 15.4 cm, find BC.

Answer :

Q2 :

Diagonals of a trapezium ABCD with AB || DC intersect each other at the point O. If AB = 2CD, find the ratio of
the areas of triangles AOB and COD.

Answer :
Since AB || CD,

∴ ∠OAB = ∠OCD and ∠OBA = ∠ODC (Alternate interior angles)

In ΔAOB and ΔCOD,

∠AOB = ∠COD (Vertically opposite angles)

∠OAB = ∠OCD (Alternate interior angles)

∠OBA = ∠ODC (Alternate interior angles)

∴ ΔAOB ∼ ΔCOD (By AAA similarity criterion)

Q3 :

In the following figure, ABC and DBC are two triangles on the same base BC. If AD intersects BC at O, show

that

Answer :

Let us draw two perpendiculars AP and DM on line BC.


We know that area of a triangle =

In ΔAPO and ΔDMO,

∠APO = ∠DMO (Each = 90°)

∠AOP = ∠DOM (Vertically opposite angles)

∴ ΔAPO ∼ ΔDMO (By AA similarity criterion)

Q4 :

If the areas of two similar triangles are equal, prove that they are congruent.

Answer :

Let us assume two similar triangles as ΔABC ∼ ΔPQR.


Q5 :

D, E and F are respectively the mid-points of sides AB, BC and CA of ΔABC. Find the ratio of the area of
ΔDEF and ΔABC.

Answer :

D and E are the mid-points of ΔABC.


Q6 :

Prove that the ratio of the areas of two similar triangles is equal to the square

of the ratio of their corresponding medians.

Answer :

Let us assume two similar triangles as ΔABC ∼ ΔPQR. Let AD and PS be the medians of these triangles.

ΔABC ∼ ΔPQR


…(1)

∠A = ∠P, ∠B = ∠Q, ∠C = ∠R … (2)

Since AD and PS are medians,

∴ BD = DC =

And, QS = SR =

Equation (1) becomes

… (3)

In ΔABD and ΔPQS,

∠B = ∠Q [Using equation (2)]

And, [Using equation (3)]

∴ ΔABD ∼ ΔPQS (SAS similarity criterion)

Therefore, it can be said that

… (4)

From equations (1) and (4), we may find that

And hence,

Q7 :

Prove that the area of an equilateral triangle described on one side of a square is equal to half the area of the
equilateral triangle described on one of its diagonals.
Answer :

Let ABCD be a square of side a.

Therefore, its diagonal

Two desired equilateral triangles are formed as ΔABE and ΔDBF.

Side of an equilateral triangle, ΔABE, described on one of its sides = a

Side of an equilateral triangle, ΔDBF, described on one of its diagonals

We know that equilateral triangles have all its angles as 60 º and all its sides of the same length. Therefore, all
equilateral triangles are similar to each other. Hence, the ratio between the areas of these triangles will be equal to
the square of the ratio between the sides of these triangles.

Q8 :

ABC and BDE are two equilateral triangles such that D is the mid-point of BC. Ratio of the area of triangles
ABC and BDE is

(A) 2 : 1

(B) 1 : 2

(C) 4 : 1

(D) 1 : 4

Answer :
We know that equilateral triangles have all its angles as 60 º and all its sides of the same length. Therefore, all
equilateral triangles are similar to each other. Hence, the ratio between the areas of these triangles will be equal to
the square of the ratio between the sides of these triangles.

Let side of ΔABC = x

Therefore, side of

Hence, the correct answer is (C).

Q9 :

Sides of two similar triangles are in the ratio 4 : 9. Areas of these triangles are in the ratio

(A) 2 : 3

(B) 4 : 9

(C) 81 : 16

(D) 16 : 81

Answer :

If two triangles are similar to each other, then the ratio of the areas of these triangles will be equal to the square of the
ratio of the corresponding sides of these triangles.

It is given that the sides are in the ratio 4:9.

Therefore, ratio between areas of these triangles =

Hence, the correct answer is (D).


Exercise 6.5 : Solutions of Questions on Page Number : 150
Q1 :

Sides of triangles are given below. Determine which of them are right triangles? In case of a right triangle,
write the length of its hypotenuse.

(i) 7 cm, 24 cm, 25 cm

(ii) 3 cm, 8 cm, 6 cm

(iii) 50 cm, 80 cm, 100 cm

(iv) 13 cm, 12 cm, 5 cm

Answer :

(i) It is given that the sides of the triangle are 7 cm, 24 cm, and 25 cm.

Squaring the lengths of these sides, we will obtain 49, 576, and 625.

49 + 576 = 625

Or,

The sides of the given triangle are satisfying Pythagoras theorem.

Therefore, it is a right triangle.

We know that the longest side of a right triangle is the hypotenuse.

Therefore, the length of the hypotenuse of this triangle is 25 cm.

(ii) It is given that the sides of the triangle are 3 cm, 8 cm, and 6 cm.

Squaring the lengths of these sides, we will obtain 9, 64, and 36.

However, 9 + 36 ≠ 64

Or, 32 + 62 ≠ 82

Clearly, the sum of the squares of the lengths of two sides is not equal to the square of the length of the third side.

Therefore, the given triangle is not satisfying Pythagoras theorem.

Hence, it is not a right triangle.

(iii)Given that sides are 50 cm, 80 cm, and 100 cm.

Squaring the lengths of these sides, we will obtain 2500, 6400, and 10000.

However, 2500 + 6400 ≠ 10000

Or, 502 + 802 ≠ 1002

Clearly, the sum of the squares of the lengths of two sides is not equal to the square of the length of the third side.

Therefore, the given triangle is not satisfying Pythagoras theorem.

Hence, it is not a right triangle.

(iv)Given that sides are 13 cm, 12 cm, and 5 cm.


Squaring the lengths of these sides, we will obtain 169, 144, and 25.

Clearly, 144 +25 = 169

Or,

The sides of the given triangle are satisfying Pythagoras theorem.

Therefore, it is a right triangle.

We know that the longest side of a right triangle is the hypotenuse.

Therefore, the length of the hypotenuse of this triangle is 13 cm.

Q2 :

PQR is a triangle right angled at P and M is a point on QR such that PM ⊥ QR. Show that PM2 = QM x MR.

Answer :
Q3 :

ABC is an isosceles triangle right angled at C. prove that AB2 = 2 AC2.

Answer :

Given that ΔABC is an isosceles triangle.

∴ AC = CB

Applying Pythagoras theorem in ΔABC (i.e., right-angled at point C), we obtain


Q4 :

ABC is an isosceles triangle with AC = BC. If AB2 = 2 AC2, prove that ABC is a right triangle.

Answer :

Given that,

Q5 :

ABC is an equilateral triangle of side 2a. Find each of its altitudes.

Answer :

Let AD be the altitude in the given equilateral triangle, ΔABC.

We know that altitude bisects the opposite side.

∴ BD = DC = a
In an equilateral triangle, all the altitudes are equal in length.

Therefore, the length of each altitude will be .

Q6 :

Prove that the sum of the squares of the sides of rhombus is equal to the sum of the squares of its
diagonals.

Answer :

In ΔAOB, ΔBOC, ΔCOD, ΔAOD,

Applying Pythagoras theorem, we obtain


Q7 :

In the following figure, O is a point in the interior of a triangle ABC, OD ⊥ BC, OE ⊥ AC and OF ⊥ AB. Show
that

(i) OA2 + OB2 + OC2 - OD2 - OE2 - OF2 = AF2 + BD2 + CE2

(ii) AF2 + BD2 + CE2 = AE2 + CD2 + BF2

Answer :

Join OA, OB, and OC.


(i) Applying Pythagoras theorem in ΔAOF, we obtain

Similarly, in ΔBOD,

Similarly, in ΔCOE,

(ii) From the above result,

Q8 :

A ladder 10 m long reaches a window 8 m above the ground. Find the distance of the foot of the ladder from
base of the wall.

Answer :

Let OA be the wall and AB be the ladder.


Therefore, by Pythagoras theorem,

Therefore, the distance of the foot of the ladder from the base of the wall is

6 m.

Q9 :

A guy wire attached to a vertical pole of height 18 m is 24 m long and has a stake attached to the other end.
How far from the base of the pole should the stake be driven so that the wire will be taut?

Answer :

Let OB be the pole and AB be the wire.

By Pythagoras theorem,

Therefore, the distance from the base is m.

Q10 :

An aeroplane leaves an airport and flies due north at a speed of 1,000 km per hour. At the same time, another
aeroplane leaves the same airport and flies due west at a speed of 1,200 km per hour. How far apart will be

the two planes after hours?


Answer :

Distance travelled by the plane flying towards north in

Similarly, distance travelled by the plane flying towards west in

Let these distances be represented by OA and OB respectively.

Applying Pythagoras theorem,

Distance between these planes after , AB =

Therefore, the distance between these planes will be km after .

Q11 :

Two poles of heights 6 m and 11 m stand on a plane ground. If the distance between the feet of the poles is
12 m, find the distance between their tops.

Answer :
Let CD and AB be the poles of height 11 m and 6 m.

Therefore, CP = 11 - 6 = 5 m

From the figure, it can be observed that AP = 12m

Applying Pythagoras theorem for ΔAPC, we obtain

Therefore, the distance between their tops is 13 m.

Q12 :

D and E are points on the sides CA and CB respectively of a triangle ABC right angled at C. Prove that AE2+
BD2 = AB2 + DE2

Answer :

Applying Pythagoras theorem in ΔACE, we obtain


Q13 :

The perpendicular from A on side BC of a ΔABC intersect BC at D such that DB = 3 CD. Prove that 2 AB2 = 2
AC2 + BC2

Answer :

Applying Pythagoras theorem for ΔACD, we obtain

Applying Pythagoras theorem in ΔABD, we obtain


Q14 :

In an equilateral triangle ABC, D is a point on side BC such that BD = BC. Prove that 9 AD2 = 7 AB2.

Answer :

Let the side of the equilateral triangle be a, and AE be the altitude of ΔABC.

∴ BE = EC = =

And, AE =

Given that, BD = BC
∴ BD =

DE = BE - BD =

Applying Pythagoras theorem in ΔADE, we obtain

AD2 = AE2 + DE2

⇒ 9 AD2 = 7 AB2

Q15 :

In an equilateral triangle, prove that three times the square of one side is equal to four times the square of
one of its altitudes.

Answer :

Let the side of the equilateral triangle be a, and AE be the altitude of ΔABC.

∴ BE = EC = =

Applying Pythagoras theorem in ΔABE, we obtain

AB2 = AE2 + BE2


4AE2 = 3a2

⇒ 4 × (Square of altitude) = 3 × (Square of one side)

Q16 :

Tick the correct answer and justify: In ΔABC, AB = cm, AC = 12 cm and BC = 6 cm.

The angle B is:

(A) 120° (B) 60°

(C) 90° (D) 45°

Answer :

Given that, AB = cm, AC = 12 cm, and BC = 6 cm

It can be observed that

AB2 = 108

AC2 = 144

And, BC2 = 36

AB2 +BC2 = AC2

The given triangle, ΔABC, is satisfying Pythagoras theorem.

Therefore, the triangle is a right triangle, right-angled at B.

∴ ∠B = 90°

Hence, the correct answer is (C).


Exercise 6.6 : Solutions of Questions on Page Number : 152
Q1 :

In the given figure, PS is the bisector of ∠QPR of ΔPQR. Prove that .

Answer :

Let us draw a line segment RT parallel to SP which intersects extended line segment QP at point T.

Given that, PS is the angle bisector of ∠QPR.

∠QPS = ∠SPR … (1)

By construction,

∠SPR = ∠PRT (As PS || TR) … (2)

∠QPS = ∠QTR (As PS || TR) … (3)

Using these equations, we obtain

∠PRT = ∠QTR

∴ PT = PR

By construction,

PS || TR
By using basic proportionality theorem for ΔQTR,
QSSR=QPPT
⇒QSSR

Q2 :

In the given figure, D is a point on hypotenuse AC of ΔABC, DM ⊥ BC and DN ⊥ AB, Prove that:

(i) DM2 = DN.MC

(ii) DN2 = DM.AN

Answer :

(i)Let us join DB.

We have, DN || CB, DM || AB, and ∠B = 90°

∴ DMBN is a rectangle.

∴ DN = MB and DM = NB

The condition to be proved is the case when D is the foot of the perpendicular drawn from B to AC.

∴ ∠CDB = 90°

⇒ ∠2 + ∠3 = 90° … (1)

In ΔCDM,

∠1 + ∠2 + ∠DMC = 180°

⇒ ∠1 + ∠2 = 90° … (2)
In ΔDMB,

∠3 + ∠DMB + ∠4 = 180°

⇒ ∠3 + ∠4 = 90° … (3)

From equation (1) and (2), we obtain

∠1 = ∠3

From equation (1) and (3), we obtain

∠2 = ∠4

In ΔDCM and ΔBDM,

∠1 = ∠3 (Proved above)

∠2 = ∠4 (Proved above)

∴ ΔDCM ∼ ΔBDM (AA similarity criterion)

⇒ DM2 = DN × MC

(ii) In right triangle DBN,

∠5 + ∠7 = 90° … (4)

In right triangle DAN,

∠6 + ∠8 = 90° … (5)

D is the foot of the perpendicular drawn from B to AC.

∴ ∠ADB = 90°

⇒ ∠5 + ∠6 = 90° … (6)

From equation (4) and (6), we obtain

∠6 = ∠7

From equation (5) and (6), we obtain

∠8 = ∠5

In ΔDNA and ΔBND,

∠6 = ∠7 (Proved above)

∠8 = ∠5 (Proved above)

∴ ΔDNA ∼ ΔBND (AA similarity criterion)

⇒ DN2 = AN × NB
⇒ DN2 = AN × DM (As NB = DM)

Q3 :

In the given figure, ABC is a triangle in which ∠ ABC> 90° and AD ⊥ CB produced. Prove that AC2 = AB2 +
BC2 + 2BC.BD.

Answer :

Applying Pythagoras theorem in ΔADB, we obtain

AB2 = AD2 + DB2 … (1)

Applying Pythagoras theorem in ΔACD, we obtain

AC2 = AD2 + DC2

AC2 = AD2 + (DB + BC)2

AC2 = AD2 + DB2 + BC2 + 2DB x BC

AC2 = AB2 + BC2 + 2DB x BC [Using equation (1)]

Q4 :

In the given figure, ABC is a triangle in which ∠ ABC < 90° and AD ⊥ BC. Prove that AC2 = AB2 + BC2 - 2BC.BD.

Answer :

Applying Pythagoras theorem in ΔADB, we obtain

AD2 + DB2 = AB2

⇒ AD2 = AB2 - DB2 … (1)


Applying Pythagoras theorem in ΔADC, we obtain

AD2 + DC2 = AC2

AB2 - BD2 + DC2 = AC2 [Using equation (1)]

AB2 - BD2 + (BC - BD)2 = AC2

AC2 = AB2 - BD2 + BC2 + BD2 -2BC x BD

= AB2 + BC2 - 2BC x BD

Q5 :

In the given figure, AD is a median of a triangle ABC and AM ⊥ BC. Prove that:

(i)

(ii)

(iii)

Answer :

(i) Applying Pythagoras theorem in ΔAMD, we obtain

AM2 + MD2 = AD2 … (1)

Applying Pythagoras theorem in ΔAMC, we obtain

AM2 + MC2 = AC2

AM2 + (MD + DC)2 = AC2

(AM2 + MD2) + DC2 + 2MD.DC = AC2

AD2 + DC2 + 2MD.DC = AC2 [Using equation (1)]

Using the result, , we obtain


(ii) Applying Pythagoras theorem in ΔABM, we obtain

AB2 = AM2 + MB2

= (AD2 - DM2) + MB2

= (AD2 - DM2) + (BD - MD)2

= AD2 - DM2 + BD2 + MD2 - 2BD × MD

= AD2 + BD2 - 2BD × MD

(iii)Applying Pythagoras theorem in ΔABM, we obtain

AM2 + MB2 = AB2 … (1)

Applying Pythagoras theorem in ΔAMC, we obtain

AM2 + MC2 = AC2 … (2)

Adding equations (1) and (2), we obtain

2AM2 + MB2 + MC2 = AB2 + AC2

2AM2 + (BD - DM)2 + (MD + DC)2 = AB2 + AC2

2AM2+BD2 + DM2 - 2BD.DM + MD2 + DC2 + 2MD.DC = AB2 + AC2

2AM2 + 2MD2 + BD2 + DC2 + 2MD ( - BD + DC) = AB2 + AC2


Question 6: 
Prove that the sum of the squares of the diagonals of parallelogram is equal to the 
sum of the squares of its sides. 

Answer: 

Let ABCD be a parallelogram. 

Let us draw perpendicular DE on extended side AB, and AF on side DC. 

Applying Pythagoras theorem in ΔDEA, we obtain 

DE2 + EA2 = DA2 … (i) 

Applying Pythagoras theorem in ΔDEB, we obtain 

DE2 + EB2 = DB2 

DE2 + (EA + AB)2 = DB2 

(DE2 + EA2) + AB2 + 2EA × AB = DB2 

DA2 + AB2 + 2EA × AB = DB2 … (ii) 

Applying Pythagoras theorem in ΔADF, we obtain 

AD2 = AF2 + FD2 

Applying Pythagoras theorem in ΔAFC, we obtain 


AC2 = AF2 + FC2 

= AF2 + (DC − FD)2 

= AF2 + DC2 + FD2 − 2DC × FD 

= (AF2 + FD2) + DC2 − 2DC × FD 

AC2 = AD2 + DC2 − 2DC × FD … (iii) 

Since ABCD is a parallelogram, 

AB = CD … (iv) 

And, BC = AD … (v) 

In ΔDEA and ΔADF, 

∠DEA = ∠AFD (Both 90°) 

∠EAD = ∠ADF (EA || DF) 

AD = AD (Common) 

∴ ΔEAD ΔFDA (AAS congruence criterion) 

EA = DF … (vi) 

Adding equations (i) and (iii), we obtain 

DA2 + AB2 + 2EA × AB + AD2 + DC2 − 2DC × FD = DB2 + AC2 

DA2 + AB2 + AD2 + DC2 + 2EA × AB − 2DC × FD = DB2 + AC2 

BC2 + AB2 + AD2 + DC2 + 2EA × AB − 2AB × EA = DB2 + AC2 

[Using equations (iv) and (vi)] 

AB2 + BC2 + CD2 + DA2 = AC2 + BD2 

Question 7: 
In the given figure, two chords AB and CD intersect each other at the point P. prove 
that: 
(i) ΔAPC ∼ ΔDPB 
(ii) AP.BP = CP.DP 

Answer: 

Let us join CB. 

(i) In ΔAPC and ΔDPB, 

∠APC = ∠DPB (Vertically opposite angles) 

∠CAP = ∠BDP (Angles in the same segment for chord CB) 

ΔAPC ∼ ΔDPB (By AA similarity criterion) 

(ii) We have already proved that 


ΔAPC ∼ ΔDPB 

We know that the corresponding sides of similar triangles are proportional. 

∴ AP. PB = PC. DP 

Question 8: 
In the given figure, two chords AB and CD of a circle intersect each other at the 
point P (when produced) outside the circle. Prove that 
(i) ΔPAC ∼ ΔPDB 
(ii) PA.PB = PC.PD 

Answer: 

(i) In ΔPAC and ΔPDB, 

∠P = ∠P (Common) 

∠PAC = ∠PDB (Exterior angle of a cyclic quadrilateral is ∠PCA = ∠PBD equal to the 
opposite interior angle) 

∴ ΔPAC ∼ ΔPDB 

(ii)We know that the corresponding sides of similar triangles are proportional. 
 

∴ PA.PB = PC.PD 

Question 9: 

In the given figure, D is a point on side BC of ΔABC such that . Prove that 
AD is the bisector of ∠BAC. 

Answer: 

Let us extend BA to P such that AP = AC. Join PC. 

It is given that, 
 

By using the converse of basic proportionality theorem, we obtain 

AD || PC 

∠BAD = ∠APC (Corresponding angles) … (1) 

And, ∠DAC = ∠ACP (Alternate interior angles) … (2) 

By construction, we have 

AP = AC 

∠APC = ∠ACP … (3) 

On comparing equations (1), (2), and (3), we obtain 

∠BAD = ∠APC 

AD is the bisector of the angle BAC. 

Question 10: 
Nazima is fly fishing in a stream. The tip of her fishing rod is 1.8 m above the 
surface of the water and the fly at the end of the string rests on the water 3.6 m 
away and 2.4 m from a point directly under the tip of the rod. Assuming that her 
string (from the tip of her rod to the fly) is taut, ho much string does she have out 
(see Fig. 6.64)? If she pulls in the string at the rate of 5 cm per second, what will be 
the horizontal distance of the fly from her after 12 seconds? 

 
 

Answer: 

Let AB be the height of the tip of the fishing rod from the water surface. Let BC be 
the horizontal distance of the fly from the tip of the fishing rod. 

Then, AC is the length of the string. 

AC can be found by applying Pythagoras theorem in ΔABC. 

AC2 = AB2 + BC2 

AB2 = (1.8 m)2 + (2.4 m)2 

AB2 = (3.24 + 5.76) m2 

AB2 = 9.00 m2 


 

Thus, the length of the string out is 3 m. 

She pulls the string at the rate of 5 cm per second. 

Therefore, string pulled in 12 seconds = 12 × 5 = 60 cm = 0.6 m 

Let the fly be at point D after 12 seconds. 

Length of string out after 12 seconds is AD. 

AD = AC − String pulled by Nazima in 12 seconds 

= (3.00 − 0.6) m 

= 2.4 m 

In ΔADB, 

AB2 + BD2 = AD2 

(1.8 m)2 + BD2 = (2.4 m)2 

BD2 = (5.76 − 3.24) m2 = 2.52 m2 

BD = 1.587 m 

Horizontal distance of fly = BD + 1.2 m 

= (1.587 + 1.2) m 
= 2.787 m 

= 2.79 m 

 
 

Coordinate Geometry 
Exercise 7.1
 
Question 1: 
Find the distance between the following pairs of points: 

(i) (2, 3), (4, 1) (ii) (−5, 7), (−1, 3) (iii) (a, b), (− a, − b) 

Answer: 

(i) Distance between the two points is given by 

(ii) Distance between is given by 

(iii) Distance between is given by 

Question 2: 
Find the distance between the points (0, 0) and (36, 15). Can you now find the distance 
between the two towns A and B discussed in Section 7.2. 
Answer: 

Distance between points  

Yes, we can find the distance between the given towns A and B. 

Assume town A at origin point (0, 0). 

Therefore, town B will be at point (36, 15) with respect to town A. 

And hence, as calculated above, the distance between town A and B will be 

39 km. 

Question 3: 
Determine if the points (1, 5), (2, 3) and (− 2, − 11) are collinear. 

Answer: 

Let the points (1, 5), (2, 3), and (−2, −11) be representing the vertices A, B, and C of the 
given triangle respectively. 

Let  

 
Therefore, the points (1, 5), (2, 3), and (−2, −11) are not collinear. 

Question 4: 
Check whether (5, − 2), (6, 4) and (7, − 2) are the vertices of an isosceles triangle. 

Answer: 

Let the points (5, −2), (6, 4), and (7, −2) are representing the vertices A, B, and C of the 
given triangle respectively. 

As two sides are equal in length, therefore, ABCis an isosceles triangle. 

Question 5: 
In a classroom, 4 friends are seated at the points A, B, C and D as shown in the 
following figure. Champa and Chameli walk into the class and after observing for a few 
minutes Champa asks Chameli, “Don’t you think ABCD is a square?” Chameli disagrees. 

Using distance formula, find which of them is correct. 

Answer: 
It can be observed that A (3, 4), B (6, 7), C (9, 4), and D (6, 1) are the positions of these 4 
friends. 

 
CD=9-62+4-12=32+32=9+9=18=32

It can be observed that all sides of this quadrilateral ABCD are of the same length and 
also the diagonals are of the same length. 

Therefore, ABCD is a square and hence, Champa was correct 

Question 6: 
Name the type of quadrilateral formed, if any, by the following points, and give reasons 
for your answer: 

(i) (− 1, − 2), (1, 0), (− 1, 2), (− 3, 0) 

(ii) (− 3, 5), (3, 1), (0, 3), (− 1, − 4) 

(iii) (4, 5), (7, 6), (4, 3), (1, 2) 


Answer: 

(i) Let the points (−1, −2), (1, 0), (−1, 2), and (−3, 0) be representing the vertices A, B, C, 
and D of the given quadrilateral respectively. 

It can be observed that all sides of this quadrilateral are of the same length and also, the 
diagonals are of the same length. Therefore, the given points are the vertices of a 
square. 

(ii)Let the points (− 3, 5), (3, 1), (0, 3), and (−1, −4) be representing the vertices A, B, C, 
and D of the given quadrilateral respectively. 

It can be observed that all sides of this quadrilateral are of different lengths. Therefore, it 
can be said that it is only a general quadrilateral, and not specific such as square, 
rectangle, etc. 
(iii)Let the points (4, 5), (7, 6), (4, 3), and (1, 2) be representing the vertices A, B, C, and D 
of the given quadrilateral respectively. 

It can be observed that opposite sides of this quadrilateral are of the same length. 
However, the diagonals are of different lengths. Therefore, the given points are the 
vertices of a parallelogram. 

Question 7: 
Find the point on the x-axis which is equidistant from (2, − 5) and (− 2, 9). 

Answer: 

We have to find a point on x-axis. Therefore, its y-coordinate will be 0. 

Let the point on x-axis be . 

By the given condition, these distances are equal in measure. 


 

Therefore, the point is (− 7, 0). 

Question 8: 
Find the values of y for which the distance between the points P (2, − 3) and Q (10, y) is 
10 units. 

Answer: 

It is given that the distance between (2, −3) and (10, y) is 10. 

Question 9: 
If Q (0, 1) is equidistant from P (5, − 3) and R (x, 6), find the values of x. Also find the 
distance QR and PR. 

Answer: 
 

Therefore, point R is (4, 6) or (−4, 6). 

When point R is (4, 6), 

When point R is (−4, 6), 

Question 10: 
Find a relation between x and y such that the point (x, y) is equidistant from the point 
(3, 6) and (− 3, 4). 

Answer: 

Point (x, y) is equidistant from (3, 6) and (−3, 4). 


 

 
Exercise 7.2 
Question 1: 
Find the coordinates of the point which divides the join of (− 1, 7) and (4, − 3) in the 
ratio 2:3. 

Answer: 

Let P(x, y) be the required point. Using the section formula, we obtain 

Therefore, the point is (1, 3). 

Question 2: 
Find the coordinates of the points of trisection of the line segment joining (4, − 1) and 
(− 2, − 3). 

Answer: 

Let P (x1, y1) and Q (x2, y2) are the points of trisection of the line segment joining the 

given points i.e., AP = PQ = QB 

Therefore, point P divides AB internally in the ratio 1:2. 


 

Point Q divides AB internally in the ratio 2:1. 

Question 3: 
To conduct Sports Day activities, in your rectangular shaped school ground ABCD, lines 
have been drawn with chalk powder at a distance of 1 m each. 100 flower pots have 
been placed at a distance of 1 m from each other along AD, as shown in the following 

figure. Niharika runs the distance AD on the 2nd line and posts a green flag. Preet 

runs the distance AD on the eighth line and posts a red flag. What is the distance 
between both the flags? If Rashmi has to post a blue flag exactly halfway between the 
line segment joining the two flags, where should she post her flag? 

 
Answer: 

It can be observed that Niharika posted the green flag at of the distance AD i.e., 

m from the starting point of 2nd line. Therefore, the coordinates of this 

point G is (2, 25). 

Similarly, Preet posted red flag at of the distance AD i.e., m from the 

starting point of 8th line. Therefore, the coordinates of this point R are (8, 20). 

Distance between these flags by using distance formula = GR 

=  

The point at which Rashmi should post her blue flag is the mid-point of the line joining 
these points. Let this point be A (x, y). 

Therefore, Rashmi should post her blue flag at 22.5m on 5th line. 

Question 4: 
Find the ratio in which the line segment joining the points (− 3, 10) and (6, − 8) is 
divided by (− 1, 6). 

Answer: 
Let the ratio in which the line segment joining (−3, 10) and (6, −8) is divided by point (−1, 
6) be k : 1. 

Question 5: 
Find the ratio in which the line segment joining A (1, − 5) and B (− 4, 5) is divided by the 
x-axis. Also find the coordinates of the point of division. 

Answer: 

Let the ratio in which the line segment joining A (1, −5) and B (−4, 5) is divided by 

x-axisbe . 

Therefore, the coordinates of the point of division is . 

We know that y-coordinate of any point on x-axis is 0. 

Therefore, x-axis divides it in the ratio 1:1. 

Division point =  

Question 6: 
If (1, 2), (4, y), (x, 6) and (3, 5) are the vertices of a parallelogram taken in order, find x 
and y. 

Answer: 

Let (1, 2), (4, y), (x, 6), and (3, 5) are the coordinates of A, B, C, D vertices of a 
parallelogram ABCD. Intersection point O of diagonal AC and BD also divides these 
diagonals. 

Therefore, O is the mid-point of AC and BD. 

If O is the mid-point of AC, then the coordinates of O are 

If O is the mid-point of BD, then the coordinates of O are 

Since both the coordinates are of the same point O, 

Question 7: 
Find the coordinates of a point A, where AB is the diameter of circle whose centre is (2, 
− 3) and B is (1, 4) 

Answer: 

Let the coordinates of point A be (x, y). 

Mid-point of AB is (2, −3), which is the center of the circle. 

Question 8: 
If A and B are (− 2, − 2) and (2, − 4), respectively, find the coordinates of P such that 

and P lies on the line segment AB. 

Answer: 

The coordinates of point A and B are (−2, −2) and (2, −4) respectively. 

Since , 

Therefore, AP: PB = 3:4 


Point P divides the line segment AB in the ratio 3:4. 

Question 9: 
Find the coordinates of the points which divide the line segment joining A (− 2, 2) and B 
(2, 8) into four equal parts. 

Answer: 

From the figure, it can be observed that points P, Q, R are dividing the line segment in a 
ratio 1:3, 1:1, 3:1 respectively. 

Question 10: 
Find the area of a rhombus if its vertices are (3, 0), (4, 5), (− 1, 4) and (− 2, −1) taken in 

order. [Hint: Area of a rhombus = (product of its diagonals)] 

Answer: 

Let (3, 0), (4, 5), (−1, 4) and (−2, −1) are the vertices A, B, C, D of a rhombus ABCD. 

 
Exercise 7.3 
Question 1: 
Find the area of the triangle whose vertices are: 

(i) (2, 3), (− 1, 0), (2, − 4) (ii) (− 5, − 1), (3, − 5), (5, 2) 

Answer: 

(i) Area of a triangle is given by 

(ii)  

Question 2: 
In each of the following find the value of ‘k’, for which the points are collinear. 

(i) (7, − 2), (5, 1), (3, − k) (ii) (8, 1), (k, − 4), (2, − 5) 

Answer: 

(i) For collinear points, area of triangle formed by them is zero. 


Therefore, for points (7, −2) (5, 1), and (3, k), area = 0 

(ii) For collinear points, area of triangle formed by them is zero. 

Therefore, for points (8, 1), (k, −4), and (2, −5), area = 0 

Question 3: 
Find the area of the triangle formed by joining the mid-points of the sides of the 
triangle whose vertices are (0, − 1), (2, 1) and (0, 3). Find the ratio of this area to the 
area of the given triangle. 

Answer: 

Let the vertices of the triangle be A (0, −1), B (2, 1), C (0, 3). 

Let D, E, F be the mid-points of the sides of this triangle. Coordinates of D, E, and F are 
given by 
 

Question 4: 
Find the area of the quadrilateral whose vertices, taken in order, are (− 4, − 2), (− 3, − 5), 
(3, − 2) and (2, 3) 

Answer: 

Let the vertices of the quadrilateral be A (−4, −2), B (−3, −5), C (3, −2), and D (2, 3). Join 
AC to form two triangles ΔABC and ΔACD. 
 

Question 5: 
You have studied in Class IX that a median of a triangle divides it into two triangles of 
equal areas. Verify this result for ΔABC whose vertices are A (4, − 6), B (3, − 2) and C 
(5, 2) 

Answer: 

Let the vertices of the triangle be A (4, −6), B (3, −2), and C (5, 2). 

Let D be the mid-point of side BC of ΔABC. Therefore, AD is the median in ΔABC. 

 
 

However, area cannot be negative. Therefore, area of ΔABD is 3 square units. 

However, area cannot be negative. Therefore, area of ΔADC is 3 square units. 

Clearly, median AD has divided ΔABC in two triangles of equal areas. 

 
Exercise 7.4 
 

Question 1: 
Determine the ratio in which the line 2x + y − 4 = 0 divides the line segment joining the 
points A(2, − 2) and B(3, 7) 

Answer: 

Let the given line divide the line segment joining the points A(2, −2) and B(3, 7) in a ratio 
k : 1. 

Coordinates of the point of division  

This point also lies on 2x + y − 4 = 0 

Therefore, the ratio in which the line 2x + y − 4 = 0 divides the line segment joining the 
points A(2, −2) and B(3, 7) is 2:9. 

Question 2: 
Find a relation between x and y if the points (x, y), (1, 2) and (7, 0) are collinear. 

Answer: 

If the given points are collinear, then the area of triangle formed by these points will be 0. 
 

This is the required relation between x and y. 

Question 3: 
Find the centre of a circle passing through the points (6, − 6), (3, − 7) and (3, 3). 

Answer: 

Let O (x, y) be the centre of the circle. And let the points (6, −6), (3, −7), and (3, 3) be 
representing the points A, B, and C on the circumference of the circle. 
 

On adding equation (1) and (2), we obtain 

10y = −20 

y = −2 

From equation (1), we obtain 

3x − 2 = 7 

3x = 9 

x = 3 

Therefore, the centre of the circle is (3, −2). 

Question 4: 
The two opposite vertices of a square are (− 1, 2) and (3, 2). Find the coordinates of the 
other two vertices. 
Answer: 

Let ABCD be a square having (−1, 2) and (3, 2) as vertices A and C respectively. Let (x, y), 

(x1, y1) be the coordinate of vertex B and D respectively. 

We know that the sides of a square are equal to each other. 

∴ AB = BC 

We know that in a square, all interior angles are of 90°. 

In ΔABC, 

AB2 + BC2 = AC2 

⇒4+y 2
+ 4 − 4y + 4 + y2 − 4y + 4 =16 
⇒ 2y 2
+ 16 − 8 y =16 

⇒ 2y 2
− 8 y = 0 

⇒ y (y − 4) = 0 
⇒ y = 0 or 4 
We know that in a square, the diagonals are of equal length and bisect each other at 90°. 
Let O be the mid-point of AC. Therefore, it will also be the mid-point of BD. 

⇒y+y 1 = 4 

If y = 0, 

y1 = 4 

If y = 4, 

y1 = 0 

Therefore, the required coordinates are (1, 0) and (1, 4). 

Question 5: 
The class X students of a secondary school in Krishinagar have been allotted a 
rectangular plot of land for their gardening activity. Saplings of Gulmohar are planted 
on the boundary at a distance of 1 m from each other. There is a triangular grassy lawn 
in the plot as shown in the following figure. The students are to sow seeds of flowering 
plants on the remaining area of the plot. 

(i) Taking A as origin, find the coordinates of the vertices of the triangle. 

(ii) What will be the coordinates of the vertices of Δ PQR if C is the origin? 

Also calculate the areas of the triangles in these cases. What do you observe? 

Answer: 

(i) Taking A as origin, we will take AD as x-axis and AB as y-axis. It can be observed that 
the coordinates of point P, Q, and R are (4, 6), (3, 2), and (6, 5) respectively. 

(ii) Taking C as origin, CB as x-axis, and CD as y-axis, the coordinates of vertices P, Q, and 
R are (12, 2), (13, 6), and (10, 3) respectively. 
 

It can be observed that the area of the triangle is same in both the cases. 

Question 6: 
The vertices of a ΔABC are A (4, 6), B (1, 5) and C (7, 2). A line is drawn to intersect 

sides AB and AC at D and E respectively, such that . Calculate the area of 
the ΔADE and compare it with the area of ΔABC. (Recall Converse of basic 
proportionality theorem and Theorem 6.6 related to 

ratio of areas of two similar triangles) 

Answer: 

Given that,  
 

Therefore, D and E are two points on side AB and AC respectively such that they divide 
side AB and AC in a ratio of 1:3. 

Clearly, the ratio between the areas of ΔADE and ΔABC is 1:16. 

Alternatively, 

We know that if a line segment in a triangle divides its two sides in the same ratio, then 
the line segment is parallel to the third side of the triangle. These two triangles so 
formed (here ΔADE and ΔABC) will be similar to each other. 
Hence, the ratio between the areas of these two triangles will be the square of the ratio 
between the sides of these two triangles. 

Therefore, ratio between the areas of ΔADE and ΔABC =  

Question 7: 
Let A (4, 2), B (6, 5) and C (1, 4) be the vertices of ΔABC. 

(i) The median from A meets BC at D. Find the coordinates of point D. 

(ii) Find the coordinates of the point P on AD such that AP: PD = 2:1 

(iii) Find the coordinates of point Q and R on medians BE and CF respectively such that 
BQ: QE = 2:1 and CR: RF = 2:1. 

(iv) What do you observe? 

(v) If A(x1, y1), B(x2, y2), and C(x3, y3) are the vertices of ΔABC, find the coordinates of 

the centroid of the triangle. 

Answer: 

(i) Median AD of the triangle will divide the side BC in two equal parts. 

Therefore, D is the mid-point of side BC. 

 
(ii) Point P divides the side AD in a ratio 2:1. 

(iii) Median BE of the triangle will divide the side AC in two equal parts. 

Therefore, E is the mid-point of side AC. 

Point Q divides the side BE in a ratio 2:1. 

Median CF of the triangle will divide the side AB in two equal parts. Therefore, F is the 
mid-point of side AB. 

Point R divides the side CF in a ratio 2:1. 

(iv) It can be observed that the coordinates of point P, Q, R are the same. 
Therefore, all these are representing the same point on the plane i.e., the centroid of the 
triangle. 

(v) Consider a triangle, ΔABC, having its vertices as A(x1, y1), B(x2, y2), and C(x3, 

y3). 

Median AD of the triangle will divide the side BC in two equal parts. Therefore, D is the 
mid-point of side BC. 

Let the centroid of this triangle be O. 

Point O divides the side AD in a ratio 2:1. 

Question 8: 
ABCD is a rectangle formed by the points A (− 1, − 1), B (− 1, 4), C (5, 4) and D (5, − 1). 
P, Q, R and S are the mid-points of AB, BC, CD, and DA respectively. Is the quadrilateral 
PQRS is a square? a rectangle? or a rhombus? Justify your answer. 

Answer: 
 

It can be observed that all sides of the given quadrilateral are of the same measure. 
However, the diagonals are of different lengths. Therefore, PQRS is a rhombus. 

 
NCERT Solutions for Class 10 Maths Unit 8
Introduction to Trigonometry Class 10
Unit 8 Introduction to Trigonometry Exercise 8.1, 8.2, 8.3, 8.4 Solutions

Exercise 8.1 : Solutions of Questions on Page Number : 181


Q1 :

In ΔABC right angled at B, AB = 24 cm, BC = 7 m. Determine

(i) sin A, cos A

(ii) sin C, cos C

Answer :

Applying Pythagoras theorem for ΔABC, we obtain

AC2 = AB2 + BC2

= (24 cm)2 + (7 cm)2

= (576 + 49) cm2

= 625 cm2

∴ AC = cm = 25 cm

(i) sin A =

cos A =

(ii)
sin C =

cos C =

Q2 :

In the given figure find tan P - cot R

Answer :

Applying Pythagoras theorem for ΔPQR, we obtain

PR2 = PQ2 + QR2

(13 cm)2 = (12 cm)2 + QR2

169 cm2 = 144 cm2 + QR2

25 cm2 = QR2

QR = 5 cm
tan P - cot R =

Q3 :

If sin A = , calculate cos A and tan A.

Answer :

Let ΔABC be a right-angled triangle, right-angled at point B.

Given that,
Let BC be 3k. Therefore, AC will be 4k, where k is a positive integer.

Applying Pythagoras theorem in ΔABC, we obtain

AC2 = AB2 + BC2

(4k)2 = AB2 + (3k)2

16k 2 - 9k 2 = AB2

7k 2 = AB2

AB =

Q4 :

Given 15 cot A = 8. Find sin A and sec A

Answer :

Consider a right-angled triangle, right-angled at B.

It is given that,
cot A =

Let AB be 8k.Therefore, BC will be 15k, where k is a positive integer.

Applying Pythagoras theorem in ΔABC, we obtain

AC2 = AB2 + BC2

= (8k)2 + (15k)2

= 64k2 + 225k2

= 289k2

AC = 17k

Q5 :

Given sec θ = , calculate all other trigonometric ratios.

Answer :

Consider a right-angle triangle ΔABC, right-angled at point B.


If AC is 13k, AB will be 12k, where k is a positive integer.

Applying Pythagoras theorem in ΔABC, we obtain

(AC)2 = (AB)2 + (BC)2

(13k)2 = (12k)2 + (BC)2

169k2 = 144k2 + BC2

25k2 = BC2

BC = 5k

Q6 :

If ∠ A and ∠ B are acute angles such that cos A = cos B, then show that

∠ A = ∠ B.

Answer :

Let us consider a triangle ABC in which CD ⊥ AB.

It is given that

cos A = cos B
… (1)

We have to prove ∠A = ∠B. To prove this, let us extend AC to P such that BC = CP.

From equation (1), we obtain

By using the converse of B.P.T,

CD||BP

⇒∠ACD = ∠CPB (Corresponding angles) … (3)

And, ∠BCD = ∠CBP (Alternate interior angles) … (4)

By construction, we have BC = CP.

∴ ∠CBP = ∠CPB (Angle opposite to equal sides of a triangle) … (5)

From equations (3), (4), and (5), we obtain

∠ACD = ∠BCD … (6)

In ΔCAD and ΔCBD,

∠ACD = ∠BCD [Using equation (6)]

∠CDA = ∠CDB [Both 90°]

Therefore, the remaining angles should be equal.

∴∠CAD = ∠CBD

⇒ ∠A = ∠B

Alternatively,

Let us consider a triangle ABC in which CD ⊥ AB.


It is given that,

cos A = cos B

Let

⇒ AD = k BD … (1)

And, AC = k BC … (2)

Using Pythagoras theorem for triangles CAD and CBD, we obtain

CD2 = AC2 - AD2 … (3)

And, CD2 = BC2 - BD2 … (4)

From equations (3) and (4), we obtain

AC2 - AD2 = BC2 - BD2

⇒ (k BC)2 - (k BD)2 = BC2 - BD2

⇒ k2 (BC2 - BD2) = BC2 - BD2

⇒ k2 = 1

⇒k=1

Putting this value in equation (2), we obtain

AC = BC

⇒ ∠A = ∠B(Angles opposite to equal sides of a triangle)

Q7 :

If cot θ = , evaluate

(i) (ii) cot2 θ


Answer :

Let us consider a right triangle ABC, right-angled at point B.

If BC is 7k, then AB will be 8k, where k is a positive integer.

Applying Pythagoras theorem in ΔABC, we obtain

AC2 = AB2 + BC2

= (8k)2 + (7k)2

= 64k2 + 49k2

= 113k2

AC =

(i)
(ii) cot2 θ = (cot θ)2 = =

Q8 :

If 3 cot A = 4, Check whether

Answer :

It is given that 3cot A = 4

Or, cot A =

Consider a right triangle ABC, right-angled at point B.

If AB is 4k, then BC will be 3k, where k is a positive integer.

In ΔABC,

(AC)2 = (AB)2 + (BC)2

= (4k)2 + (3k)2
= 16k2 + 9k2

= 25k2

AC = 5k

cos2 A - sin2 A =

Q9 :

In ΔABC, right angled at B. If , find the value of

(i) sin A cos C + cos A sin C

(ii) cos A cos C - sin A sin C


Answer :

If BC is k, then AB will be , where k is a positive integer.

In ΔABC,

AC2 = AB2 + BC2

= 3k2 + k2 = 4k2

∴ AC = 2k

(i) sin A cos C + cos A sin C

(ii) cos A cos C - sin A sin C


Q10 :

In ΔPQR, right angled at Q, PR + QR = 25 cm and PQ = 5 cm. Determine the values of sin P, cos P and tan P.

Answer :

Given that, PR + QR = 25

PQ = 5

Let PR be x.

Therefore, QR = 25 - x

Applying Pythagoras theorem in ΔPQR, we obtain

PR2 = PQ2 + QR2

x2 = (5)2 + (25 - x)2

x2 = 25 + 625 + x2 - 50x

50x = 650

x = 13

Therefore, PR = 13 cm

QR = (25 - 13) cm = 12 cm
Q11 :

State whether the following are true or false. Justify your answer.

(i) The value of tan A is always less than 1.

(ii) sec A = for some value of angle A.

(iii) cos A is the abbreviation used for the cosecant of angle A.

(iv) cot A is the product of cot and A

(v) sin θ = , for some angle θ

Answer :

(i) Consider a ΔABC, right-angled at B.

But >1

∴tan A > 1

So, tan A < 1 is not always true.

Hence, the given statement is false.

(ii)
Let AC be 12k, AB will be 5k, where k is a positive integer.

Applying Pythagoras theorem in ΔABC, we obtain

AC2 = AB2 + BC2

(12k)2 = (5k)2 + BC2

144k2 = 25k2 + BC2

BC2 = 119k2

BC = 10.9k

It can be observed that for given two sides AC = 12k and AB = 5k,

BC should be such that,

AC - AB < BC < AC + AB

12k - 5k < BC < 12k + 5k

7k < BC < 17 k

However, BC = 10.9k. Clearly, such a triangle is possible and hence, such value of sec A is possible.

Hence, the given statement is true.

(iii) Abbreviation used for cosecant of angle A is cosec A. And cos A is the abbreviation used for cosine of angle A.

Hence, the given statement is false.

(iv) cot A is not the product of cot and A. It is the cotangent of ∠A.

Hence, the given statement is false.

(v) sin θ =

We know that in a right-angled triangle,

In a right-angled triangle, hypotenuse is always greater than the remaining two sides. Therefore, such value of sin θ
is not possible.
Hence, the given statement is false

Exercise 8.2 : Solutions of Questions on Page Number : 187


Q1 :

Evaluate the following

(i) sin60° cos30° + sin30° cos 60°

(ii) 2tan245° + cos230° - sin260°

(iii)

(iv)

(v)

Answer :

(i) sin60° cos30° + sin30° cos 60°

(ii) 2tan245° + cos230° - sin260°

(iii)
(iv)

(v)
Q2 :

Choose the correct option and justify your choice.

(i)

(A). sin60°

(B). cos60°

(C). tan60°

(D). sin30°

(ii)

(A). tan90°

(B). 1

(C). sin45°

(D). 0

(iii) sin2A = 2sinA is true when A =

(A). 0°

(B). 30°

(C). 45°

(D). 60°

(iv)
(A). cos60°

(B). sin60°

(C). tan60°

(D). sin30°

Answer :

(i)

Out of the given alternatives, only

Hence, (A) is correct.

(ii)

Hence, (D) is correct.

(iii)Out of the given alternatives, only A = 0° is correct.

As sin 2A = sin 0° = 0

2 sinA = 2sin 0° = 2(0) = 0

Hence, (A) is correct.

(iv)
Out of the given alternatives, only tan 60°

Hence, (C) is correct.

Q3 :

If and ;

0° < A + B ≤90°, A > B find A and B.

Answer :

⇒ A + B = 60 … (1)

⇒ tan (A - B) = tan30

⇒ A - B = 30 … (2)

On adding both equations, we obtain

2A = 90

⇒ A = 45

From equation (1), we obtain

45 + B = 60

B = 15

Therefore, ∠A = 45° and ∠B = 15°

Q4 :

State whether the following are true or false. Justify your answer.

(i) sin(A + B) = sin A + sin B

(ii) The value of sinθincreases as θincreases

(iii) The value of cos θincreases as θincreases

(iv) sinθ= cos θ for all values of θ

(v) cot A is not defined for A = 0°


Answer :

(i) sin(A + B) = sin A + sin B

Let A = 30° and B = 60°

sin (A + B) = sin (30° + 60°)

= sin 90°

=1

sin A + sin B = sin 30° + sin 60°

Clearly, sin (A + B) ≠sin A + sin B

Hence, the given statement is false.

(ii) The value of sin θ increases as θ increases in the interval of 0° < θ < 90° as

sin 0° = 0

sin 90° = 1

Hence, the given statement is true.

(iii) cos 0° = 1

cos90° = 0

It can be observed that the value of cos θ does not increase in the interval of 0° < θ < 90°.

Hence, the given statement is false.

(iv) sin θ = cos θ for all values of θ.

This is true when θ = 45°


As

It is not true for all other values of θ.

As and ,

Hence, the given statement is false.

(v) cot A is not defined for A = 0°

As ,

= undefined

Hence, the given statement is true.

Exercise 8.3 : Solutions of Questions on Page Number : 189


Q1 :

Evaluate

(I)

(II)

(III) cos 48° - sin 42°

(IV)cosec 31° - sec 59°

Answer :

(I)

(II)
(III)cos 48° - sin 42° = cos (90° - 42°) - sin 42°

= sin 42° - sin 42°

=0

(IV) cosec 31° - sec 59° = cosec (90° - 59°) - sec 59°

= sec 59° - sec 59°

=0

Q2 :

Show that

(I) tan 48° tan 23° tan 42° tan 67° = 1

(II)cos 38° cos 52° - sin 38° sin 52° = 0

Answer :

(I) tan 48° tan 23° tan 42° tan 67°

= tan (90° - 42°) tan (90° - 67°) tan 42° tan 67°

= cot 42° cot 67° tan 42° tan 67°

= (cot 42° tan 42°) (cot 67° tan 67°)

= (1) (1)

=1

(II) cos 38° cos 52° - sin 38° sin 52°

= cos (90° - 52°) cos (90°-38°) - sin 38° sin 52°

= sin 52° sin 38° - sin 38° sin 52°

=0

Q3 :

If tan 2A = cot (A- 18°), where 2A is an acute angle, find the value of A.

Answer :

Given that,

tan 2A = cot (A- 18°)

cot (90° - 2A) = cot (A -18°)


90° - 2A = A- 18°

108° = 3A

A = 36°

Q4 :

If tan A = cot B, prove that A + B = 90°

Answer :

Given that,

tan A = cot B

tan A = tan (90° - B)

A = 90° - B

A + B = 90°

Q5 :

If sec 4A = cosec (A- 20°), where 4A is an acute angle, find the value of A.

Answer :

Given that,

sec 4A = cosec (A - 20°)

cosec (90° - 4A) = cosec (A - 20°)

90° - 4A= A- 20°

110° = 5A

A = 22°

Q6 :

If A, Band C are interior angles of a triangle ABC then show that

Answer :

We know that for a triangle ABC,

∠ A + ∠B + ∠C = 180°
∠B + ∠C= 180° - ∠A

Q7 :

Express sin 67° + cos 75° in terms of trigonometric ratios of angles between 0° and 45°.

Answer :

sin 67° + cos 75°

= sin (90° - 23°) + cos (90° - 15°)

= cos 23° + sin 15°

Exercise 8.4 : Solutions of Questions on Page Number : 193


Q1 :

Express the trigonometric ratios sin A, sec A and tan A in terms of cot A.

Answer :

We know that,

will always be positive as we are adding two positive quantities.

Therefore,

We know that,
However,

Therefore,

Also,

Q2 :

Write all the other trigonometric ratios of ∠ A in terms of sec A.

Answer :

We know that,

Also, sin2 A + cos2 A = 1

sin2 A = 1 - cos2 A

tan2A + 1 = sec2A

tan2A = sec2A - 1
Q3 :

Evaluate

(i)

(ii) sin25° cos65° + cos25° sin65°

Answer :

(i)

(As sin2A + cos2A = 1)

=1

(ii) sin25° cos65° + cos25° sin65°

= sin225° + cos225°
= 1 (As sin2A + cos2A = 1)

Q4 :

Choose the correct option. Justify your choice.

(i) 9 sec2 A - 9 tan2 A =

(A) 1

(B) 9

(C) 8

(D) 0

(ii) (1 + tan θ + sec θ) (1 + cot θ - cosec θ)

(A) 0

(B) 1

(C) 2

(D) - 1

(iii) (secA + tanA) (1 - sinA) =

(A) secA

(B) sinA

(C) cosecA

(D) cosA

(iv)

(A) sec2 A

(B) - 1

(C) cot2 A

(D) tan2 A

Answer :

(i) 9 sec2A - 9 tan2A

= 9 (sec2A - tan2A)

= 9 (1) [As sec2 A - tan2 A = 1]

=9

Hence, alternative (B) is correct.

(ii)
(1 + tan θ + sec θ) (1 + cot θ - cosec θ)

Hence, alternative (C) is correct.

(iii) (secA + tanA) (1 - sinA)

= cosA

Hence, alternative (D) is correct.

(iv)

Hence, alternative (D) is correct.


Q5 :

Prove the following identities, where the angles involved are acute angles for which the expressions are
defined.

Answer :

(i)

(ii)

(iii)
 

(iii)  

 
= secθ cosec θ + 

= R.H.S. 

(iv)  

= R.H.S 

(v)  

Using the identity cosec2 = 1 + cot2 , 

L.H.S =  
 

= cosec A + cot A 

= R.H.S 

(vi)  

 
(vii)  

(viii)  

(ix)  

 
 

Hence, L.H.S = R.H.S 

(x)  

 
NCERT Solutions for Class 10 Maths Unit 9
Some Applications of Trigonometry Class 10
Unit 9 Some Applications of Trigonometry Exercise 9.1 Solutions

Exercise 9.1 : Solutions of Questions on Page Number : 203


Q1 :

A circus artist is climbing a 20 m long rope, which is tightly stretched and tied from the top of a vertical pole
to the ground. Find the height of the pole, if the angle made by the rope with the ground level is 30 °.

Answer :

It can be observed from the figure that AB is the pole.

In ΔABC,

Therefore, the height of the pole is 10 m.

Q2 :

A tree breaks due to storm and the broken part bends so that the top of the tree touches the ground making
an angle 30 ° with it. The distance between the foot of the tree to the point where the top touches the ground
is 8 m. Find the height of the tree.

Answer :
Let AC was the original tree. Due to storm, it was broken into two parts. The broken part is making 30° with
the ground.

In ,

Height of tree = + BC

Hence, the height of the tree is .

Q3 :
A contractor plans to install two slides for the children to play in a park. For the children below the age of 5
years, she prefers to have a slide whose top is at a height of 1.5 m, and is inclined at an angle of 30 ° to the
ground, where as for the elder children she wants to have a steep side at a height of 3 m, and inclined at an
angle of 60 ° to the ground. What should be the length of the slide in each case?

Answer :

It can be observed that AC and PR are the slides for younger and elder children respectively.

In ΔABC,

In ΔPQR,

Therefore, the lengths of these slides are 3 m and .

Q4 :
The angle of elevation of the top of a tower from a point on the ground, which is 30 m away from the foot of
the tower is 30°. Find the height of the tower.

Answer :

Let AB be the tower and the angle of elevation from point C (on ground) is

30°.

In ΔABC,

Therefore, the height of the tower is .

Q5 :

A kite is flying at a height of 60 m above the ground. The string attached to the kite is temporarily tied to a
point on the ground. The inclination of the string with the ground is 60°. Find the length of the string,
assuming that there is no slack in the string.

Answer :

Let K be the kite and the string is tied to point P on the ground.

In ΔKLP,
Hence, the length of the string is .

Q6 :

A 1.5 m tall boy is standing at some distance from a 30 m tall building. The angle of elevation from his eyes
to the top of the building increases from 30° to 60° as he walks towards the building. Find the distance he
walked towards the building.

Answer :

Let the boy was standing at point S initially. He walked towards the building and reached at point T.

It can be observed that

PR = PQ - RQ

= (30 - 1.5) m = 28.5 m =

In ΔPAR,

In ΔPRB,
ST = AB

Hence, he walked towards the building.

Q7 :

From a point on the ground, the angles of elevation of the bottom and the top of a transmission tower fixed
at the top of a 20 m high building are 45° and 60° respectively. Find the height of the tower.

Answer :

Let BC be the building, AB be the transmission tower, and D be the point on the ground from where the elevation
angles are to be measured.

In ΔBCD,

In ΔACD,
Therefore, the height of the transmission tower is m.

Q8 :

A statue, 1.6 m tall, stands on a top of pedestal, from a point on the ground, the angle of elevation of the top
of statue is 60° and from the same point the angle of elevation of the top of the pedestal is 45 °. Find the
height of the pedestal.

Answer :

Let AB be the statue, BC be the pedestal, and D be the point on the ground from where the elevation angles are to be
measured.

In ΔBCD,

In ΔACD,
Therefore, the height of the pedestal is 0.8 m.

Q9 :

The angle of elevation of the top of a building from the foot of the tower is

30° and the angle of elevation of the top of the tower from the foot of the building is 60°. If the tower is 50 m
high, find the height of the building.

Answer :

Let AB be the building and CD be the tower.

In ΔCDB,
In ΔABD,

Therefore, the height of the building is .

Q10 :

Two poles of equal heights are standing opposite each other an either side of the road, which is 80 m wide.
From a point between them on the road, the angles of elevation of the top of the poles are 60° and 30º,
respectively. Find the height of poles and the distance of the point from the poles.

Answer :

Let AB and CD be the poles and O is the point from where the elevation angles are measured.

In ΔABO,

In ΔCDO,
Since the poles are of equal heights,

CD = AB

DO = BD - BO = (80 - 20) m = 60 m

Therefore, the height of poles is and the point is 20 m and 60 m far from these poles.

Q11 :

A TV tower stands vertically on a bank of a canal. From a point on the other bank directly opposite the tower
the angle of elevation of the top of the tower is 60°. From another point 20 m away from this point on the line
joining this point to the foot of the tower, the angle of elevation of the top of the tower is 30°. Find the height
of the tower and the width of the canal.
Answer :

In ΔABC,

In ΔABD,

Therefore, the height of the tower is m and the width of the canal is

10 m.
Q12 :

From the top of a 7 m high building, the angle of elevation of the top of a cable tower is 60° and the angle of
depression of its foot is 45°. Determine the height of the tower.

Answer :

Let AB be a building and CD be a cable tower.

In ΔABD,

In ΔACE,

AE = BD = 7 m

Therefore, the height of the cable tower is .


Q13 :

As observed from the top of a 75 m high lighthouse from the sea-level, the angles of depression of two ships
are 30° and 45°. If one ship is exactly behind the other on the same side of the lighthouse, find the distance
between the two ships.

Answer :

Let AB be the lighthouse and the two ships be at point C and D respectively.

In ΔABC,

In ΔABD,

Therefore, the distance between the two ships is m.

Q14 :
A 1.2 m tall girl spots a balloon moving with the wind in a horizontal line at a height of 88.2 m from the
ground. The angle of elevation of the balloon from the eyes of the girl at any instant is 60°. After some time,
the angle of elevation reduces to 30°. Find the distance travelled by the balloon during the interval.

Answer :

Let the initial position A of balloon change to B after some time and CD be the girl.

In ΔACE,

In ΔBCG,
Distance travelled by balloon = EG = CG - CE

Q15 :

A straight highway leads to the foot of a tower. A man standing at the top of the tower observes a car as an
angle of depression of 30°, which is approaching the foot of the tower with a uniform speed. Six seconds
later, the angle of depression of the car is found to be 60°. Find the time taken by the car to reach the foot of
the tower from this point.

Answer :

Let AB be the tower.

Initial position of the car is C, which changes to D after six seconds.

In ΔADB,

In ΔABC,
Time taken by the car to travel distance DC = 6 seconds

Time taken by the car to travel distance DB

Q16 :

The angles of elevation of the top of a tower from two points at a distance of 4 m and 9 m. from the base of
the tower and in the same straight line with it are complementary. Prove that the height of the tower is 6 m.

Answer :

Let AQ be the tower and R, S are the points 4m, 9m away from the base of the tower respectively.

The angles are complementary. Therefore, if one angle is θ, the other will be 90 - θ.

In ΔAQR,
In ΔAQS,

On multiplying equations (i) and (ii), we obtain

However, height cannot be negative.

Therefore, the height of the tower is 6 m.


NCERT Solutions for Class 10 Maths Unit 10
Circles Class 10
Unit 10 Circles Exercise 10.1, 10.2 Solutions

Exercise 10.1 : Solutions of Questions on Page Number : 204


Q1 :

How many tangents can a circle have?

Answer :

A circle can have infinite tangents.

Q2 :

Fill in the blanks:

(i) A tangent to a circle intersects it in _______ point (s).

(ii) A line intersecting a circle in two points is called a __________.

(iii) A circle can have __________ parallel tangents at the most.

(iv) The common point of a tangent to a circle and the circle is called ____.

Answer :

(i) One

(ii) Secant

(iii) Two

(iv) Point of contact

Q3 :

A tangent PQ at a point P of a circle of radius 5 cm meets a line through the centre O at a point Q so that OQ
= 12 cm. Length PQ is :

(A) 12 cm. (B) 13 cm (C) 8.5 cm (D) cm

Answer :

We know that the line drawn from the centre of the circle to the tangent is perpendicular to the tangent.

OP PQ
By applying Pythagoras theorem in ΔOPQ,

OP2 + PQ2 = OQ2

52 + PQ2 =122

PQ2 =144 - 25

PQ = cm.

Hence, the correct answer is (D).

Q4 :

Draw a circle and two lines parallel to a given line such that one is a tangent and the other, a secant to the
circle.

Answer :

It can be observed that AB and CD are two parallel lines. Line AB is intersecting the circle at exactly two points, P
and Q. Therefore, line AB is the secant of this circle. Since line CD is intersecting the circle at exactly one point, R,
line CD is the tangent to the circle.

Exercise 10.2 : Solutions of Questions on Page Number : 213


Q1 :

From a point Q, the length of the tangent to a circle is 24 cm and the distance of Q from the centre is 25 cm.
The radius of the circle is

(A) 7 cm (B) 12 cm
(C) 15 cm (D) 24.5 cm

Answer :

Let O be the centre of the circle.

Given that,

OQ = 25cm and PQ = 24 cm

As the radius is perpendicular to the tangent at the point of contact,

Therefore, OP ⊥ PQ

Applying Pythagoras theorem in ΔOPQ, we obtain

OP2 + PQ2 = OQ2

OP2 + 242 = 252

OP2 = 625 - 576

OP2 = 49

OP = 7

Therefore, the radius of the circle is 7 cm.

Hence, alternative (A) is correct.

Q2 :

In the given figure, if TP and TQ are the two tangents to a circle with centre O so that ∠POQ = 110 , then
∠PTQ is equal to

(A) 60 (B) 70

(C) 80 (D) 90
Answer :

It is given that TP and TQ are tangents.

Therefore, radius drawn to these tangents will be perpendicular to the tangents.

Thus, OP ⊥ TP and OQ ⊥ TQ

∠OPT = 90 º

∠OQT = 90 º

In quadrilateral POQT,

Sum of all interior angles = 360

∠OPT + ∠POQ +∠OQT + ∠PTQ = 360

⇒ 90 + 110 º + 90 + PTQ = 360

⇒ PTQ = 70

Hence, alternative (B) is correct.

Q3 :

If tangents PA and PB from a point P to a circle with centre O are inclined to each other an angle of 80 ,
then ∠POA is equal to

(A) 50 (B) 60

(C) 70 (D) 80

Answer :

It is given that PA and PB are tangents.

Therefore, the radius drawn to these tangents will be perpendicular to the tangents.

Thus, OA ⊥ PA and OB ⊥ PB

∠OBP = 90 º

∠OAP = 90 º

In AOBP,

Sum of all interior angles = 360


∠OAP + ∠APB +∠PBO + ∠BOA = 360

90 + 80 +90 º + BOA = 360

∠BOA = 100

In ΔOPB and ΔOPA,

AP = BP (Tangents from a point)

OA = OB (Radii of the circle)

OP = OP (Common side)

Therefore, ΔOPB ≅ ΔOPA (SSS congruence criterion)

A ↔ B, P ↔ P, O ↔ O

And thus, ∠POB = ∠POA

Hence, alternative (A) is correct.

Q4 :

Prove that the tangents drawn at the ends of a diameter of a circle are parallel.

Answer :

Let AB be a diameter of the circle. Two tangents PQ and RS are drawn at points A and B respectively.

Radius drawn to these tangents will be perpendicular to the tangents.

Thus, OA ⊥ RS and OB ⊥ PQ

∠ OAR = 90º

∠ OAS = 90º

∠ OBP = 90º

∠ OBQ = 90º

It can be observed that


∠ OAR = ∠ OBQ (Alternate interior angles)

∠ OAS = ∠ OBP (Alternate interior angles)

Since alternate interior angles are equal, lines PQ and RS will be parallel.

Q5 :

Prove that the perpendicular at the point of contact to the tangent to a circle passes through the centre.

Answer :

Let us consider a circle with centre O. Let AB be a tangent which touches the circle at P.

We have to prove that the line perpendicular to AB at P passes through centre O. We shall prove this by contradiction
method.

Let us assume that the perpendicular to AB at P does not pass through centre O. Let it pass through another point O'.
Join OP and O'P.

As perpendicular to AB at P passes through O', therefore,

∠ O'PB = 90° ... (1)

O is the centre of the circle and P is the point of contact. We know the line joining the centre and the point of contact
to the tangent of the circle are perpendicular to each other.

∴ ∠ OPB = 90° ... (2)

Comparing equations (1) and (2), we obtain

∠ O'PB = ∠ OPB ... (3)

From the figure, it can be observed that,

∠ O'PB < ∠ OPB ... (4)

Therefore, ∠ O'PB = ∠ OPB is not possible. It is only possible, when the line O'P coincides with OP.

Therefore, the perpendicular to AB through P passes through centre O.


Q6 :

The length of a tangent from a point A at distance 5 cm from the centre of the circle is 4 cm. Find the radius
of the circle.

Answer :

Let us consider a circle centered at point O.

AB is a tangent drawn on this circle from point A.

Given that,

OA = 5cm and AB = 4 cm

In ΔABO,

OB ⊥ AB (Radius ⊥ tangent at the point of contact)

Applying Pythagoras theorem in ΔABO, we obtain

AB2 + BO2 = OA2

42 + BO2 = 52

16 + BO2 = 25

BO2 = 9

BO = 3

Hence, the radius of the circle is 3 cm.

Q7 :

Two concentric circles are of radii 5 cm and 3 cm. Find the length of the chord of the larger circle which
touches the smaller circle.

Answer :
Let the two concentric circles be centered at point O. And let PQ be the chord of the larger circle which touches the
smaller circle at point A. Therefore, PQ is tangent to the smaller circle.

OA ⊥ PQ (As OA is the radius of the circle)

Applying Pythagoras theorem in ΔOAP, we obtain

OA2 + AP2 = OP2

32 + AP2 = 52

9 + AP2 = 25

AP2 = 16

AP = 4

In ΔOPQ,

Since OA ⊥ PQ,

AP = AQ (Perpendicular from the center of the circle bisects the chord)

PQ = 2AP = 2 × 4 = 8

Therefore, the length of the chord of the larger circle is 8 cm.

Q8 :

A quadrilateral ABCD is drawn to circumscribe a circle (see given figure) Prove that AB + CD = AD + BC

Answer :

It can be observed that

DR = DS (Tangents on the circle from point D) … (1)


CR = CQ (Tangents on the circle from point C) … (2)

BP = BQ (Tangents on the circle from point B) … (3)

AP = AS (Tangents on the circle from point A) … (4)

Adding all these equations, we obtain

DR + CR + BP + AP = DS + CQ + BQ + AS

(DR + CR) + (BP + AP) = (DS + AS) + (CQ + BQ)

CD + AB = AD + BC

Q9 :

In the given figure, XY and X'Y' are two parallel tangents to a circle with centre O and another tangent AB
with point of contact C intersecting XY at A and X'Y' at B. Prove that ∠AOB=90 .

Answer :

Let us join point O to C.

In ΔOPA and ΔOCA,

OP = OC (Radii of the same circle)

AP = AC (Tangents from point A)

AO = AO (Common side)

ΔOPA ΔOCA (SSS congruence criterion)

Therefore, P ↔ C, A ↔ A, O ↔ O

∠POA = ∠COA … (i)

Similarly, ΔOQB ΔOCB


∠QOB = ∠COB … (ii)

Since POQ is a diameter of the circle, it is a straight line.

Therefore, ∠POA + ∠COA + ∠COB + ∠QOB = 180 º

From equations (i) and (ii), it can be observed that

2∠COA + 2 ∠COB = 180 º

∠COA + ∠COB = 90 º

∠AOB = 90°

Q10 :

Prove that the angle between the two tangents drawn from an external point to a circle is supplementary to
the angle subtended by the line-segment joining the points of contact at the centre.

Answer :

Let us consider a circle centered at point O. Let P be an external point from which two tangents PA and PB are drawn
to the circle which are touching the circle at point A and B respectively and AB is the line segment, joining point of
contacts A and B together such that it subtends ∠ AOB at center O of the circle.

It can be observed that

OA (radius) ⊥ PA (tangent)

Therefore, ∠ OAP = 90°

Similarly, OB (radius) ⊥ PB (tangent)

∠ OBP = 90°

In quadrilateral OAPB,

Sum of all interior angles = 360º

∠ OAP +∠ APB+∠ PBO +∠ BOA = 360º

90º + ∠ APB + 90º + ∠ BOA = 360º

∠ APB + ∠ BOA = 180º

Hence, it can be observed that the angle between the two tangents drawn from an external point to a circle is
supplementary to the angle subtended by the line-segment joining the points of contact at the centre.
Q11 :

Prove that the parallelogram circumscribing a circle is a rhombus.

Answer :

Since ABCD is a parallelogram,

AB = CD ...(1)

BC = AD ...(2)

It can be observed that

DR = DS (Tangents on the circle from point D)

CR = CQ (Tangents on the circle from point C)

BP = BQ (Tangents on the circle from point B)

AP = AS (Tangents on the circle from point A)

Adding all these equations, we obtain

DR + CR + BP + AP = DS + CQ + BQ + AS

(DR + CR) + (BP + AP) = (DS + AS) + (CQ + BQ)

CD + AB = AD + BC

On putting the values of equations (1) and (2) in this equation, we obtain

2AB = 2BC

AB = BC ...(3)

Comparing equations (1), (2), and (3), we obtain

AB = BC = CD = DA

Hence, ABCD is a rhombus.

Q12 :

A triangle ABC is drawn to circumscribe a circle of radius 4 cm such that the segments BD and DC into
which BC is divided by the point of contact D are of lengths 8 cm and 6 cm respectively (see given figure).
Find the sides AB and AC.
Answer :

Let the given circle touch the sides AB and AC of the triangle at point E and F respectively and the length of the line
segment AF be x.

In ABC,

CF = CD = 6cm (Tangents on the circle from point C)

BE = BD = 8cm (Tangents on the circle from point B)

AE = AF = x (Tangents on the circle from point A)

AB = AE + EB = x + 8

BC = BD + DC = 8 + 6 = 14

CA = CF + FA = 6 + x

2s = AB + BC + CA

= x + 8 + 14 + 6 + x

= 28 + 2x

s = 14 + x
Area of ΔOBC =

Area of ΔOCA =

Area of ΔOAB =

Area of ΔABC = Area of ΔOBC + Area of ΔOCA + Area of ΔOAB

Either x+14 = 0 or x - 7 =0

Therefore, x = - 14and 7

However, x = - 14 is not possible as the length of the sides will be negative.

Therefore, x = 7

Hence, AB = x + 8 = 7 + 8 = 15 cm

CA = 6 + x = 6 + 7 = 13 cm

Q13 :

Prove that opposite sides of a quadrilateral circumscribing a circle subtend supplementary angles at the
centre of the circle.

Answer :
Let ABCD be a quadrilateral circumscribing a circle centered at O such that it touches the circle at point P, Q, R, S.
Let us join the vertices of the quadrilateral ABCD to the center of the circle.

Consider ΔOAP and ΔOAS,

AP = AS (Tangents from the same point)

OP = OS (Radii of the same circle)

OA = OA (Common side)

ΔOAP ≅ ΔOAS (SSS congruence criterion)

Therefore, A â” ” A, P â” ” S, O â” ” O

And thus, ∠ POA = ∠ AOS

∠1=∠8

Similarly,

∠2=∠3

∠4=∠5

∠6=∠7

∠ 1 + ∠ 2 + ∠ 3 + ∠ 4 + ∠ 5 + ∠ 6 + ∠ 7 + ∠ 8 = 360º

(∠ 1 + ∠ 8) + (∠ 2 + ∠ 3) + (∠ 4 + ∠ 5) + (∠ 6 + ∠ 7) = 360º

2∠ 1 + 2∠ 2 + 2∠ 5 + 2∠ 6 = 360º

2(∠ 1 + ∠ 2) + 2(∠ 5 + ∠ 6) = 360º

(∠ 1 + ∠ 2) + (∠ 5 + ∠ 6) = 180º

∠ AOB + ∠ COD = 180º

Similarly, we can prove that ∠ BOC + ∠ DOA = 180º

Hence, opposite sides of a quadrilateral circumscribing a circle subtend supplementary angles at the centre of the
circle.
NCERT Solutions for Class 10 Maths Unit 11
Constructions Class 10
Unit 11 Constructions Exercise 11.1, 11.2 Solutions

Exercise 11.1 : Solutions of Questions on Page Number : 219


Q1 :

Draw a line segment of length 7.6 cm and divide it in the ratio 5:8. Measure the two parts. Give the
justification of the construction.

Answer :

A line segment of length 7.6 cm can be divided in the ratio of 5:8 as follows.

Step 1 Draw line segment AB of 7.6 cm and draw a ray AX making an acute angle with line segment AB.

Step 2 Locate 13 (= 5 + 8) points, A1, A2, A3, A4 …….. A13, on AX such that AA1 = A1A2 = A2A3 and so on.

Step 3 Join BA13.

Step 4 Through the point A5, draw a line parallel to BA13 (by making an angle equal to ∠AA13B) at A5 intersecting AB at
point C.

C is the point dividing line segment AB of 7.6 cm in the required ratio of 5:8.

The lengths of AC and CB can be measured. It comes out to 2.9 cm and 4.7 cm respectively.

Justification

The construction can be justified by proving that

By construction, we have A5C || A13B. By applying Basic proportionality theorem for the triangle AA13B, we obtain
… (1)

From the figure, it can be observed that AA5 and A5A13 contain 5 and 8 equal divisions of line segments respectively.

… (2)

On comparing equations (1) and (2), we obtain

This justifies the construction.

Q2 :

Construct a triangle of sides 4 cm, 5cm and 6cm and then a triangle similar to it whose sides are of the
corresponding sides of the first triangle.

Give the justification of the construction.

Answer :

Step 1

Draw a line segment AB = 4 cm. Taking point A as centre, draw an arc of 5 cm radius. Similarly, taking point B as its
centre, draw an arc of 6 cm radius. These arcs will intersect each other at point C. Now, AC = 5 cm and BC = 6 cm
and ΔABC is the required triangle.

Step 2

Draw a ray AX making an acute angle with line AB on the opposite side of vertex C.

Step 3

Locate 3 points A1, A2, A3 (as 3 is greater between 2 and 3) on line AX such that AA1 = A1A2 = A2A3.

Step 4

Join BA3 and draw a line through A2 parallel to BA3 to intersect AB at point B'.

Step 5

Draw a line through B' parallel to the line BC to intersect AC at C'.

ΔAB'C' is the required triangle.


Justification

The construction can be justified by proving that

By construction, we have B'C' || BC

∴ ∠A = ∠ABC (Corresponding angles)

In ΔAB'C' and ΔABC,

∠ = ∠ABC (Proved above)

∠ = ∠BAC (Common)

∴Δ Ã¢Ë†Â¼ ΔABC (AA similarity criterion)

… (1)

In ΔAA2B' and ΔAA3B,

∠A2AB' = ∠A3AB (Common)

∠AA2B' = ∠AA3B (Corresponding angles)

∴ ΔAA2B' ∼ ΔAA3B (AA similarity criterion)

From equations (1) and (2), we obtain


This justifies the construction.

Q3 :

Construct a triangle with sides 5 cm, 6 cm and 7 cm and then another triangle whose sides are of the
corresponding sides of the first triangle.

Give the justification of the construction.

Answer :

Step 1

Draw a line segment AB of 5 cm. Taking A and B as centre, draw arcs of 6 cm and 7 cm radius respectively. Let
these arcs intersect each other at point C. ΔABC is the required triangle having length of sides as 5 cm, 6 cm, and 7
cm respectively.

Step 2

Draw a ray AX making acute angle with line AB on the opposite side of vertex C.

Step 3

Locate 7 points, A1, A2, A3, A4 A5, A6, A7 (as 7 is greater between 5and 7), on line AX such that AA1 = A1A2 = A2A3 =
A3A4 = A4A5 = A5A6 = A6A7.

Step 4

Join BA5 and draw a line through A7 parallel to BA5 to intersect extended line segment AB at point B'.

Step 5

Draw a line through B' parallel to BC intersecting the extended line segment AC at C'. ΔAB'C' is the required triangle.
Justification

The construction can be justified by proving that

In ΔABC and ΔAB'C',

∠ABC = ∠AB'C' (Corresponding angles)

∠BAC = ∠B'AC' (Common)

∴ ΔABC ∼ ΔAB'C' (AA similarity criterion)

… (1)

In ΔAA5B and ΔAA7B',

∠A5AB = ∠A7AB' (Common)

∠AA5B = ∠AA7B' (Corresponding angles)

∴ ΔAA5B ∼ ΔAA7B' (AA similarity criterion)

On comparing equations (1) and (2), we obtain


This justifies the construction.

Q4 :

Construct an isosceles triangle whose base is 8 cm and altitude 4 cm and then another triangle whose side

are times the corresponding sides of the isosceles triangle.

Give the justification of the construction.

Answer :

Let us assume that ΔABC is an isosceles triangle having CA and CB of equal lengths, base AB of 8 cm, and AD is
the altitude of 4 cm.

A ΔAB'C' whose sides are times of ΔABC can be drawn as follows.

Step 1

Draw a line segment AB of 8 cm. Draw arcs of same radius on both sides of the line segment while taking point A and
B as its centre. Let these arcs intersect each other at O and O'. Join OO'. Let OO' intersect AB at D.

Step 2

Taking D as centre, draw an arc of 4 cm radius which cuts the extended line segment OO' at point C. An isosceles
ΔABC is formed, having CD (altitude) as 4 cm and AB (base) as 8 cm.

Step 3

Draw a ray AX making an acute angle with line segment AB on the opposite side of vertex C.

Step 4

Locate 3 points (as 3 is greater between 3 and 2) A1, A2, and A3 on AX such that AA1 = A1A2 = A2A3.

Step 5

Join BA2 and draw a line through A3 parallel to BA2 to intersect extended line segment AB at point B'.

Step 6

Draw a line through B' parallel to BC intersecting the extended line segment AC at C'. ΔAB'C' is the required triangle.
Justification

The construction can be justified by proving that

In ΔABC and ΔAB'C',

∠ABC = ∠AB'C' (Corresponding angles)

∠BAC = ∠B'AC' (Common)

∴ ΔABC ∼ ΔAB'C' (AA similarity criterion)

… (1)

In ΔAA2B and ΔAA3B',

∠A2AB = ∠A3AB' (Common)

∠AA2B = ∠AA3B' (Corresponding angles)

∴ ΔAA2B ∼ ΔAA3B' (AA similarity criterion)

On comparing equations (1) and (2), we obtain


This justifies the construction.

Q5 :

Draw a triangle ABC with side BC = 6 cm, AB = 5 cm and ∠ABC = 60°. Then construct a triangle whose sides

are of the corresponding sides of the triangle ABC.

Give the justification of the construction.

Answer :

A ΔA'BC' whose sides are of the corresponding sides of ΔABC can be drawn as follows.

Step 1

Draw a ΔABC with side BC = 6 cm, AB = 5 cm and ∠ABC = 60°.

Step 2

Draw a ray BX making an acute angle with BC on the opposite side of vertex A.

Step 3

Locate 4 points (as 4 is greater in 3 and 4), B1, B2, B3, B4, on line segment BX.

Step 4

Join B4C and draw a line through B3, parallel to B4C intersecting BC at C'.

Step 5

Draw a line through C' parallel to AC intersecting AB at A'. ΔA'BC' is the required triangle.

Justification

The construction can be justified by proving


In ΔA'BC' and ΔABC,

∠A'C'B = ∠ACB (Corresponding angles)

∠A'BC' = ∠ABC (Common)

∴ ΔA'BC' ∼ ΔABC (AA similarity criterion)

… (1)

In ΔBB3C' and ΔBB4C,

∠B3BC' = ∠B4BC (Common)

∠BB3C' = ∠BB4C (Corresponding angles)

∴ ΔBB3C' ∼ ΔBB4C (AA similarity criterion)

From equations (1) and (2), we obtain

This justifies the construction.

Q6 :

Draw a triangle ABC with side BC = 7 cm, ∠B = 45°, ∠A = 105°. Then, construct a triangle whose sides are
times the corresponding side of ΔABC. Give the justification of the construction.

Answer :

∠B = 45°, ∠A = 105°

Sum of all interior angles in a triangle is 180°.

∠A + ∠B + ∠C = 180°

105° + 45° + ∠C = 180°

∠C = 180° - 150°

∠C = 30°

The required triangle can be drawn as follows.


Step 1

Draw a ΔABC with side BC = 7 cm, ∠B = 45°, ∠C = 30°.

Step 2

Draw a ray BX making an acute angle with BC on the opposite side of vertex A.

Step 3

Locate 4 points (as 4 is greater in 4 and 3), B1, B2, B3, B4, on BX.

Step 4

Join B3C. Draw a line through B4 parallel to B3C intersecting extended BC at C'.

Step 5

Through C', draw a line parallel to AC intersecting extended line segment at C'. ΔA'BC' is the required triangle.

Justification

The construction can be justified by proving that

In ΔABC and ΔA'BC',

∠ABC = ∠A'BC' (Common)

∠ACB = ∠A'C'B (Corresponding angles)

∴ ΔABC ∼ ΔA'BC' (AA similarity criterion)

… (1)

In ΔBB3C and ΔBB4C',

∠B3BC = ∠B4BC' (Common)

∠BB3C = ∠BB4C' (Corresponding angles)

∴ ΔBB3C ∼ ΔBB4C' (AA similarity criterion)


On comparing equations (1) and (2), we obtain

This justifies the construction.

Q7 :

Draw a right triangle in which the sides (other than hypotenuse) are of lengths 4 cm and 3 cm. the construct

another triangle whose sides are times the corresponding sides of the given triangle. Give the justification
of the construction.

Answer :

It is given that sides other than hypotenuse are of lengths 4 cm and 3 cm. Clearly, these will be perpendicular to each
other.

The required triangle can be drawn as follows.

Step 1

Draw a line segment AB = 4 cm. Draw a ray SA making 90° with it.

Step 2

Draw an arc of 3 cm radius while taking A as its centre to intersect SA at C. Join BC. ΔABC is the required triangle.

Step 3

Draw a ray AX making an acute angle with AB, opposite to vertex C.

Step 4

Locate 5 points (as 5 is greater in 5 and 3), A1, A2, A3, A4, A5, on line segment AX such that AA1 = A1A2 = A2A3 = A3A4 =
A4A5.

Step 5

Join A3B. Draw a line through A5 parallel to A3B intersecting extended line segment AB at B'.

Step 6

Through B', draw a line parallel to BC intersecting extended line segment AC at C'. ΔAB'C' is the required triangle.
Justification

The construction can be justified by proving that

In ΔABC and ΔAB'C',

∠ABC = ∠AB'C' (Corresponding angles)

∠BAC = ∠B'AC' (Common)

∴ ΔABC ∼ ΔAB'C' (AA similarity criterion)

… (1)

In ΔAA3B and ΔAA5B',

∠A3AB = ∠A5AB' (Common)

∠AA3B = ∠AA5B' (Corresponding angles)

∴ ΔAA3B ∼ ΔAA5B' (AA similarity criterion)

On comparing equations (1) and (2), we obtain

This justifies the construction.


Exercise 11.2 : Solutions of Questions on Page Number : 221
Q1 :

Draw a circle of radius 6 cm. From a point 10 cm away from its centre, construct the pair of tangents to the
circle and measure their lengths. Give the justification of the construction.

Answer :

A pair of tangents to the given circle can be constructed as follows.

Step 1

Taking any point O of the given plane as centre, draw a circle of 6 cm radius. Locate a point P, 10 cm away from O.
Join OP.

Step 2

Bisect OP. Let M be the mid-point of PO.

Step 3

Taking M as centre and MO as radius, draw a circle.

Step 4

Let this circle intersect the previous circle at point Q and R.

Step 5

Join PQ and PR. PQ and PR are the required tangents.

The lengths of tangents PQ and PR are 8 cm each.

Justification

The construction can be justified by proving that PQ and PR are the tangents to the circle (whose centre is O and
radius is 6 cm). For this, join OQ and OR.
∠ PQO is an angle in the semi-circle. We know that angle in a semi-circle is a right angle.

∴ ∠ PQO = 90°

⇒ OQ ⊥ PQ

Since OQ is the radius of the circle, PQ has to be a tangent of the circle. Similarly, PR is a tangent of the circle.

Q2 :

Construct a tangent to a circle of radius 4 cm from a point on the concentric circle of radius 6 cm and
measure its length. Also verify the measurement by actual calculation. Give the justification of the
construction.

Answer :

Tangents on the given circle can be drawn as follows.

Step 1

Draw a circle of 4 cm radius with centre as O on the given plane.

Step 2

Draw a circle of 6 cm radius taking O as its centre. Locate a point P on this circle and join OP.

Step 3

Bisect OP. Let M be the mid-point of PO.

Step 4

Taking M as its centre and MO as its radius, draw a circle. Let it intersect the given circle at the points Q and R.

Step 5

Join PQ and PR. PQ and PR are the required tangents.


It can be observed that PQ and PR are of length 4.47 cm each.

In ΔPQO,

Since PQ is a tangent,

∠PQO = 90°

PO = 6 cm

QO = 4 cm

Applying Pythagoras theorem in ΔPQO, we obtain

PQ2 + QO2 = PQ2

PQ2 + (4)2 = (6)2

PQ2 + 16 = 36

PQ2 = 36 - 16

PQ2 = 20

PQ

PQ = 4.47 cm

Justification

The construction can be justified by proving that PQ and PR are the tangents to the circle (whose centre is O and
radius is 4 cm). For this, let us join OQ and OR.

∠PQO is an angle in the semi-circle. We know that angle in a semi-circle is a right angle.

∴ ∠PQO = 90°
⇒ OQ ⊥ PQ

Since OQ is the radius of the circle, PQ has to be a tangent of the circle. Similarly, PR is a tangent of the circle.

Q3 :

Draw a circle of radius 3 cm. Take two points P and Q on one of its extended diameter each at a distance of 7
cm from its centre. Draw tangents to the circle from these two points P and Q. Give the justification of the
construction.

Answer :

The tangent can be constructed on the given circle as follows.

Step 1

Taking any point O on the given plane as centre, draw a circle of 3 cm radius.

Step 2

Take one of its diameters, PQ, and extend it on both sides. Locate two points on this diameter such that OR = OS = 7
cm

Step 3

Bisect OR and OS. Let T and U be the mid-points of OR and OS respectively.

Step 4

Taking T and U as its centre and with TO and UO as radius, draw two circles. These two circles will intersect the
circle at point V, W, X, Y respectively. Join RV, RW, SX, and SY. These are the required tangents.

Justification

The construction can be justified by proving that RV, RW, SY, and SX are the tangents to the circle (whose centre is
O and radius is 3 cm). For this, join OV, OW, OX, and OY.
∠ RVO is an angle in the semi-circle. We know that angle in a semi-circle is a right angle.

∴ ∠ RVO = 90°

⇒ OV ⊥ RV

Since OV is the radius of the circle, RV has to be a tangent to the circle.


Similarly, it can be shown that RW, SX, and SY are the tangents of the circle.

Q4 :

Draw a pair of tangents to a circle of radius 5 cm which are inclined to each other at an angle of 60°. Give the
justification of the construction.

Answer :

The tangents can be constructed in the following manner:

Step 1

Draw a circle of radius 5 cm and with centre as O.

Step 2

Take a point A on the circumference of the circle and join OA. Draw a perpendicular to OA at point A.

Step 3

Draw a radius OB, making an angle of 120° (180° - 60°) with OA.

Step 4

Draw a perpendicular to OB at point B. Let both the perpendiculars intersect at point P. PA and PB are the required
tangents at an angle of 60°.
Justification

The construction can be justified by proving that ∠ APB = 60°

By our construction

∠ OAP = 90°

∠ OBP = 90°

And ∠ AOB = 120°

We know that the sum of all interior angles of a quadrilateral = 360°

∠ OAP + ∠ AOB + ∠ OBP + ∠ APB = 360°

90° + 120° + 90° + ∠ APB = 360°

∠ APB = 60°

This justifies the construction.

Q5 :

Draw a line segment AB of length 8 cm. Taking A as centre, draw a circle of radius 4 cm and taking B as
centre, draw another circle of radius 3 cm. Construct tangents to each circle from the centre of the other
circle. Give the justification of the construction.

Answer :

The tangents can be constructed on the given circles as follows.

Step 1

Draw a line segment AB of 8 cm. Taking A and B as centre, draw two circles of 4 cm and 3 cm radius.

Step 2

Bisect the line AB. Let the mid-point of AB be C. Taking C as centre, draw a circle of AC radius which will intersect
the circles at points P, Q, R, and S. Join BP, BQ, AS, and AR. These are the required tangents.
Justification

The construction can be justified by proving that AS and AR are the tangents of the circle (whose centre is B and
radius is 3 cm) and BP and BQ are the tangents of the circle (whose centre is A and radius is 4 cm). For this, join AP,
AQ, BS, and BR.

∠ ASB is an angle in the semi-circle. We know that an angle in a semi-circle is a right angle.

∴ ∠ ASB = 90°

⇒ BS ⊥ AS

Since BS is the radius of the circle, AS has to be a tangent of the circle. Similarly, AR, BP, and BQ are the tangents.

Q6 :

Draw a circle with the help of a bangle. Take a point outside the circle. Construct the pair of tangents from
this point to the circles. Give the justification of the construction.

Answer :

The required tangents can be constructed on the given circle as follows.

Step 1

Draw a circle with the help of a bangle.

Step 2

Take a point P outside this circle and take two chords QR and ST.

Step 3

Draw perpendicular bisectors of these chords. Let them intersect each other at point O.

Step 4
Join PO and bisect it. Let U be the mid-point of PO. Taking U as centre, draw a circle of radius OU, which will
intersect the circle at V and W. Join PV and PW.

PV and PW are the required tangents.

Justification

The construction can be justified by proving that PV and PW are the tangents to the circle. For this, first of all, it has
to be proved that O is the centre of the circle. Let us join OV and OW.

We know that perpendicular bisector of a chord passes through the centre. Therefore, the perpendicular bisector of
chords QR and ST pass through the centre. It is clear that the intersection point of these perpendicular bisectors is
the centre of the circle. ∠ PVO is an angle in the semi-circle. We know that an angle in a semi-circle is a right angle.

∴ ∠ PVO = 90°

⇒ OV ⊥ PV

Since OV is the radius of the circle, PV has to be a tangent of the circle. Similarly, PW is a tangent of the circle.

Q7 :

Let ABC be a right triangle in which AB = 6 cm, BC = 8 cm and ∠ B = 90°. BD is the perpendicular from B on
AC. The circle through B, C, and D is drawn. Construct the tangents from A to this circle. Give the
justification of the construction.

Answer :

Consider the following situation. If a circle is drawn through B, D, and C, BC will be its diameter as ∠ BDC is of
measure 90°. The centre E of this circle will be the mid-point of BC.
The required tangents can be constructed on the given circle as follows.

Step 1

Join AE and bisect it. Let F be the mid-point of AE.

Step 2

Taking F as centre and FE as its radius, draw a circle which will intersect the circle at point B and G. Join AG.

AB and AG are the required tangents.

Justification

The construction can be justified by proving that AG and AB are the tangents to the circle. For this, join EG.

∠ AGE is an angle in the semi-circle. We know that an angle in a semi-circle is a right angle.

∴ ∠ AGE = 90°

⇒ EG ⊥ AG

Since EG is the radius of the circle, AG has to be a tangent of the circle.


Already, ∠ B = 90°

⇒ AB ⊥ BE

Since BE is the radius of the circle, AB has to be a tangent of the circle.


NCERT Solutions for Class 10 Maths Unit 12
Areas Related to Circles Class 10
Unit 12 Areas Related to Circles Exercise 12.1, 12.2, 12.3 Solutions
Exercise 12.1 : Solutions of Questions on Page Number : 225
Q1 :

The radii of two circles are 19 cm and 9 cm respectively. Find the radius of the circle which has
circumference equal to the sum of the circumferences of the two circles.

Answer :

Radius (r1) of 1st circle = 19 cm

Radius (r2) or 2nd circle = 9 cm

Let the radius of 3rd circle be r.

Circumference of 1st circle = 2πr1 = 2π (19) = 38π

Circumference of 2nd circle = 2πr2 = 2π (9) = 18π

Circumference of 3rd circle = 2πr

Given that,

Circumference of 3rd circle = Circumference of 1st circle + Circumference of 2nd circle

2πr = 38π + 18π = 56π

Therefore, the radius of the circle which has circumference equal to the sum of the circumference of the given two
circles is 28 cm.

Q2 :

The radii of two circles are 8 cm and 6 cm respectively. Find the radius of the circle having area equal to the
sum of the areas of the two circles.

Answer :

Radius (r1) of 1st circle = 8 cm

Radius (r2) of 2nd circle = 6 cm

Let the radius of 3rd circle be r.

Area of 1st circle

Area of 2nd circle


Given that,

Area of 3rd circle = Area of 1st circle + Area of 2nd circle

However, the radius cannot be negative. Therefore, the radius of the circle having area equal to the sum of the areas
of the two circles is 10 cm.

Q3 :

Given figure depicts an archery target marked with its five scoring areas from the centre outwards as Gold,
Red, Blue, Black and White. The diameter of the region representing Gold score is 21 cm and each of the

other bands is 10.5 cm wide. Find the area of each of the five scoring regions.

Answer :

Radius (r1) of gold region (i.e., 1st circle)


Given that each circle is 10.5 cm wider than the previous circle.

Therefore, radius (r2) of 2nd circle = 10.5 + 10.5

21 cm

Radius (r3) of 3rd circle = 21 + 10.5

= 31.5 cm

Radius (r4) of 4th circle = 31.5 + 10.5

= 42 cm

Radius (r5) of 5th circle = 42 + 10.5

= 52.5 cm

Area of gold region = Area of 1st circle

Area of red region = Area of 2nd circle - Area of 1st circle

Area of blue region = Area of 3rd circle - Area of 2nd circle

Area of black region = Area of 4th circle - Area of 3rd circle

Area of white region = Area of 5th circle - Area of 4th circle

Therefore, areas of gold, red, blue, black, and white regions are 346.5 cm2, 1039.5 cm2, 1732.5 cm2, 2425.5 cm2, and
3118.5 cm2 respectively.
Q4 :

The wheels of a car are of diameter 80 cm each. How many complete revolutions does each wheel make in 10

minutes when the car is traveling at a speed of 66 km per hour?

Answer :

Diameter of the wheel of the car = 80 cm

Radius (r) of the wheel of the car = 40 cm

Circumference of wheel = 2πr

= 2π (40) = 80π cm

Speed of car = 66 km/hour

Distance travelled by the car in 10 minutes

= 110000 × 10 = 1100000 cm

Let the number of revolutions of the wheel of the car be n.

n × Distance travelled in 1 revolution (i.e., circumference)

= Distance travelled in 10 minutes

Therefore, each wheel of the car will make 4375 revolutions.

Q5 :

Tick the correct answer in the following and justify your choice: If the perimeter and the area of a circle are
numerically equal, then the radius of the circle is

(A) 2 units (B) π units (C) 4 units (D)7 units

Answer :

Let the radius of the circle be r.


Circumference of circle = 2πr

Area of circle = πr2

Given that, the circumference of the circle and the area of the circle are equal.

This implies 2πr = πr2

2=r

Therefore, the radius of the circle is 2 units.

Hence, the correct answer is A.

Exercise 12.2 : Solutions of Questions on Page Number : 230


Q1 :

Find the area of a sector of a circle with radius 6 cm if angle of the sector is 60°.

Answer :

Let OACB be a sector of the circle making 60° angle at centre O of the circle.

Area of sector of angle θ =

Area of sector OACB =

Therefore, the area of the sector of the circle making 60° at the centre of the circle is

Q2 :

Find the area of a quadrant of a circle whose circumference is 22 cm.


Answer :

Let the radius of the circle be r.

Circumference = 22 cm

2πr = 22

Quadrant of circle will subtend 90° angle at the centre of the circle.

Area of such quadrant of the circle

Q3 :

The length of the minute hand of a clock is 14 cm. Find the area swept by the minute hand in 5

minutes.

Answer :

We know that in 1 hour (i.e., 60 minutes), the minute hand rotates 360°.
In 5 minutes, minute hand will rotate =

Therefore, the area swept by the minute hand in 5 minutes will be the area of a sector of 30° in a circle of 14 cm
radius.

Area of sector of angle θ =

Area of sector of 30°

Therefore, the area swept by the minute hand in 5 minutes is

Q4 :

A chord of a circle of radius 10 cm subtends a right angle at the centre. Find the area of the corresponding:

(i) Minor segment

(ii) Major sector

[Use π = 3.14]

Answer :

Let AB be the chord of the circle subtending 90° angle at centre O of the circle.

Area of major sector OADB =


Area of minor sector OACB =

Area of ΔOAB =

= 50 cm2

Area of minor segment ACB = Area of minor sector OACB -

Area of ΔOAB = 78.5 - 50 = 28.5 cm2

Q5 :

In a circle of radius 21 cm, an arc subtends an angle of 60° at the centre. Find:

(i) The length of the arc

(ii) Area of the sector formed by the arc

(iii) Area of the segment forced by the corresponding chord

Answer :

Radius (r) of circle = 21 cm

Angle subtended by the given arc = 60°

Length of an arc of a sector of angle θ =


Length of arc ACB =

= 22 cm

Area of sector OACB =

In ΔOAB,

∠OAB = ∠OBA (As OA = OB)

∠OAB + ∠AOB + ∠OBA = 180°

2∠OAB + 60° = 180°

∠OAB = 60°

Therefore, ΔOAB is an equilateral triangle.

Area of ΔOAB =

Area of segment ACB = Area of sector OACB - Area of ΔOAB

Q6 :

A chord of a circle of radius 15 cm subtends an angle of 60° at the centre. Find the areas of the
corresponding minor and major segments of the circle.

[Use π = 3.14 and ]

Answer :
Radius (r) of circle = 15 cm

Area of sector OPRQ =

In ΔOPQ,

∠OPQ = ∠OQP (As OP = OQ)

∠OPQ + ∠OQP + ∠POQ = 180°

2 ∠OPQ = 120°

∠OPQ = 60°

ΔOPQ is an equilateral triangle.

Area of ΔOPQ =

Area of segment PRQ = Area of sector OPRQ - Area of ΔOPQ

= 117.75 - 97.3125

= 20.4375 cm2

Area of major segment PSQ = Area of circle - Area of segment PRQ


Q7 :

A chord of a circle of radius 12 cm subtends an angle of 120° at the centre. Find the area of the
corresponding segment of the circle.

[Use π = 3.14 and ]

Answer :

Let us draw a perpendicular OV on chord ST. It will bisect the chord ST.

SV = VT

In ΔOVS,

Area of ΔOST =

Area of sector OSUT =


Area of segment SUT = Area of sector OSUT - Area of ΔOST

= 150.72 - 62.28

= 88.44 cm2

Q8 :

A horse is tied to a peg at one corner of a square shaped grass field of side 15 m by means of a 5 m long
rope (see the given figure). Find

(i) The area of that part of the field in which the horse can graze.

(ii) The increase in the grazing area of the rope were 10 m long instead of 5 m.

[Use À = 3.14]

Answer :

From the figure, it can be observed that the horse can graze a sector of 90° in a circle of 5 m radius.

Area that can be grazed by horse = Area of sector OACB

Area that can be grazed by the horse when length of rope is 10 m long
Increase in grazing area = (78.5 - 19.625) m2

= 58.875 m2

Q9 :

A brooch is made with silver wire in the form of a circle with diameter 35 mm. The wire is also used in making
5 diameters which divide the circle into 10 equal sectors as shown in figure. Find.

(i) The total length of the silver wire required.

(ii) The area of each sector of the brooch

Answer :

Total length of wire required will be the length of 5 diameters and the circumference of the brooch.

Radius of circle =

Circumference of brooch = 2πr

= 110 mm

Length of wire required = 110 + 5 × 35

= 110 + 175 = 285 mm

It can be observed from the figure that each of 10 sectors of the circle is subtending 36° at the centre of the circle.
Therefore, area of each sector =

Q10 :

An umbrella has 8 ribs which are equally spaced (see figure). Assuming umbrella to be a flat circle of radius

45 cm, find the area between the two consecutive ribs of the umbrella.

Answer :

There are 8 ribs in an umbrella. The area between two consecutive ribs is subtending at the centre of
the assumed flat circle.

Area between two consecutive ribs of circle =


Q11 :

A car has two wipers which do not overlap. Each wiper has blade of length 25 cm sweeping through an angle

of 115°. Find the total area cleaned at each sweep of the blades.

Answer :

It can be observed from the figure that each blade of wiper will sweep an area of a sector of 115° in a circle of 25 cm
radius.

Area of such sector =

Area swept by 2 blades =

Q12 :

To warn ships for underwater rocks, a lighthouse spreads a red coloured light over a sector of angle 80° to a
distance of 16.5 km. Find the area of the sea over which the ships warned. [Use π = 3.14]

Answer :
It can be observed from the figure that the lighthouse spreads light across a

sector of 80° in a circle of 16.5 km radius.

Area of sector OACB =

Q13 :

A round table cover has six equal designs as shown in figure. If the radius of the cover is 28 cm, find the cost

of making the designs at the rate of Rs.0.35 per cm2. [Use ]

Answer :

It can be observed that these designs are segments of the circle.

Consider segment APB. Chord AB is a side of the hexagon. Each chord will substitute at the centre of
the circle.
In ΔOAB,

∠OAB = ∠OBA (As OA = OB)

∠AOB = 60°

∠OAB + ∠OBA + ∠AOB = 180°

2∠OAB = 180° - 60° = 120°

∠OAB = 60°

Therefore, ΔOAB is an equilateral triangle.

Area of ΔOAB =

= 333.2 cm2

Area of sector OAPB =

Area of segment APB = Area of sector OAPB - Area of ΔOAB

Cost of making 1 cm2 designs = Rs 0.35

Cost of making 464.76 cm2 designs = = Rs 162.68

Therefore, the cost of making such designs is Rs 162.68.

Q14 :

Tick the correct answer in the following:

Area of a sector of angle p (in degrees) of a circle with radius R is

(A) , (B) , (C) , (D)


Answer :

We know that area of sector of angle θ =

Area of sector of angle P =

Hence, (D) is the correct answer.

Q15 :

Tick the correct answer in the following:

Area of a sector of angle p (in degrees) of a circle with radius R is

(A) , (B) , (C) , (D)

Answer :

We know that area of sector of angle θ =

Area of sector of angle P =


Hence, (D) is the correct answer.

Exercise 12.3 : Solutions of Questions on Page Number : 235


Q1 :

Find the area of the shaded region in the given figure, if radii of the two concentric circles with centre O are 7

cm and 14 cm respectively and ∠AOC = 40°.

Answer :

Radius of inner circle = 7 cm

Radius of outer circle = 14 cm

Area of shaded region = Area of sector OAFC - Area of sector OBED

=40°360°×π(14)2 - 40°

Q2 :
Find the area of the shaded region in the given figure, if ABCD is a square of side 14 cm and APD and BPC

are semicircles.

Answer :

It can be observed from the figure that the radius of each semi-circle is 7 cm.

Area of each semi-circle =

Area of square ABCD = (Side)2 = (14)2 = 196 cm2

Area of the shaded region

= Area of square ABCD - Area of semi-circle APD - Area of semi-circle BPC

= 196 - 77 - 77 = 196 - 154 = 42 cm2

Q3 :
Find the area of the shaded region in the given figure, where a circular arc of radius 6 cm has been drawn

with vertex O of an equilateral triangle OAB of side 12 cm as centre.

Answer :

We know that each interior angle of an equilateral triangle is of measure 60°.

Area of sector OCDE

Area of

Area of circle = πr2

Area of shaded region = Area of ΔOAB + Area of circle - Area of sector OCDE
Q4 :

From each corner of a square of side 4 cm a quadrant of a circle of radius 1 cm is cut and also a circle of
diameter 2 cm is cut as shown in the given figure. Find the area of the remaining portion of the

square.

Answer :

Each quadrant is a sector of 90° in a circle of 1 cm radius.

Area of each quadrant


Area of square = (Side)2 = (4)2 = 16 cm2

Area of circle = πr2 = π (1)2

Area of the shaded region = Area of square - Area of circle - 4 × Area of quadrant

Q5 :

In a circular table cover of radius 32 cm, a design is formed leaving an equilateral triangle ABC in the middle

as shown in the given figure. Find the area of the design (Shaded region).

Answer :
Radius (r) of circle = 32 cm

AD is the median of ABC.

AD = 48 cm

In ΔABD,

AB2 = AD2 + BD2

Area of equilateral triangle,

Area of circle = πr2

Area of design = Area of circle - Area of ΔABC

Q6 :
In the given figure, ABCD is a square of side 14 cm. With centres A, B, C and D, four circles are drawn such
that each circle touches externally two of the remaining three circles. Find the area of the shaded

region.

Answer :

Area of each of the 4 sectors is equal to each other and is a sector of 90° in a circle of 7 cm radius.
Area of each sector

Area of square ABCD = (Side)2 = (14)2 = 196 cm2

Area of shaded portion = Area of square ABCD - 4 × Area of each sector

Therefore, the area of shaded portion is 42 cm2.

Q7 :

Thegivenfigure depicts a racing track whose left and right ends are semicircular.

The distance between the two inner parallel line segments is 60 m and they are each 106 m long. If the track
is 10 m wide, find:

(i) The distance around the track along its inner edge

(ii) The area of the track

Answer :

Distance around the track along its inner edge = AB + arc BEC + CD + arc DFA
Area of the track = (Area of GHIJ - Area of ABCD) + (Area of semi-circle HKI - Area of semi-circle BEC) + (Area of
semi-circle GLJ - Area of semi-circle

AFD)

Therefore, the area of the track is 4320 m2.

Q8 :

In the given figure, AB and CD are two diameters of a circle (with centre O) perpendicular to each other and

OD is the diameter of the smaller circle. If OA = 7 cm, find the area of the shaded region.
Answer :

Radius (r1) of larger circle = 7 cm

Radius (r2) of smaller circle

Area of smaller circle

Area of semi-circle AECFB of larger circle


Area of

Area of the shaded region

= Area of smaller circle + Area of semi-circle AECFB - Area of ΔABC

Q9 :

The area of an equilateral triangle ABC is 17320.5 cm2. With each vertex of the triangle as centre, a circle is
drawn with radius equal to half the length of the side of the triangle (See the given figure). Find the area of

shaded region. [Use π = 3.14 and ]

Answer :

Let the side of the equilateral triangle be a.

Area of equilateral triangle = 17320.5 cm2


Each sector is of measure 60°.

Area of sector ADEF

Area of shaded region = Area of equilateral triangle - 3 × Area of each sector

Q10 :

On a square handkerchief, nine circular designs each of radius 7 cm are made (see the given figure). Find the

area of the remaining portion of the handkerchief.


Answer :

From the figure, it can be observed that the side of the square is 42 cm.

Area of square = (Side)2 = (42)2 = 1764 cm2

Area of each circle = πr2

Area of 9 circles = 9 × 154 = 1386 cm2

Area of the remaining portion of the handkerchief = 1764 - 1386 = 378 cm2

Q11 :
In the given figure, OACB is a quadrant of circle with centre O and radius 3.5 cm. If OD = 2 cm, find the area
of the

(i) Quadrant OACB

(ii) Shaded region

Answer :

(i) Since OACB is a quadrant, it will subtend 90° angle at O.

Area of quadrant OACB

(ii) Area of ΔOBD


Area of the shaded region = Area of quadrant OACB - Area of ΔOBD

Q12 :

In the given figure, a square OABC is inscribed in a quadrant OPBQ. If OA = 20 cm, find the area of the
shaded region. [Use À = 3.14]

Answer :
In ΔOAB,

OB2 = OA2 + AB2

= (20)2 + (20)2

Radius (r) of circle

Area of quadrant OPBQ

Area of OABC = (Side)2 = (20)2 = 400 cm2

Area of shaded region = Area of quadrant OPBQ - Area of OABC

= (628 - 400) cm2

= 228 cm2

Q13 :

AB and CD are respectively arcs of two concentric circles of radii 21 cm and 7 cm and centre O (see the

given figure). If ∠AOB = 30°, find the area of the shaded region.
Answer :

Area of the shaded region = Area of sector OAEB - Area of sector OCFD

Q14 :
In the given figure, ABC is a quadrant of a circle of radius 14 cm and a semicircle is drawn with BC as

diameter. Find the area of the shaded region.

Answer :

As ABC is a quadrant of the circle, ∠BAC will be of measure 90 º.

In ΔABC,

BC2 = AC2 + AB2

= (14)2 + (14)2

Radius (r1) of semi-circle drawn on

Area of
Area of sector

= 154 - (154
- 98)

= 98 cm2

Q15 :

Calculate the area of the designed region in the given figure common between the two quadrants of circles of

radius 8 cm each.

Answer :
The designed area is the common region between two sectors BAEC and DAFC.

Area of sector

Area of ΔBAC

Area of the designed portion = 2 × (Area of segment AEC)

= 2 × (Area of sector BAEC - Area of ΔBAC)

Q16 :

Calculate the area of the designed region in the given figure common between the two quadrants of circles of

radius 8 cm each.
Answer :

The designed area is the common region between two sectors BAEC and DAFC.

Area of sector

Area of ΔBAC

Area of the designed portion = 2 × (Area of segment AEC)

= 2 × (Area of sector BAEC - Area of ΔBAC)


 

Surface Areas and Volumes 


Exercise 13.1   
 
 
 
Question 1: 

2 cubes each of volume 64 cm3 are joined end to end. Find the surface area of the 
resulting cuboids. 

Answer: 

Given that, 

Volume of cubes = 64 cm3 

(Edge) 3 = 64 

Edge = 4 cm 

If cubes are joined end to end, the dimensions of the resulting cuboid will be 4 cm, 4 cm, 
8 cm. 

Question 2: 
A vessel is in the form of a hollow hemisphere mounted by a hollow cylinder. The 
diameter of the hemisphere is 14 cm and the total height of the vessel is 13 cm. Find 

the inner surface area of the vessel.  

Answer: 

It can be observed that radius (r) of the cylindrical part and the hemispherical part is the 
same (i.e., 7 cm). 

Height of hemispherical part = Radius = 7 cm 

Height of cylindrical part (h) = 13 −7 = 6 cm 

Inner surface area of the vessel = CSA of cylindrical part + CSA of hemispherical part 

 
 
 
Question 3: 
A toy is in the form of a cone of radius 3.5 cm mounted on a hemisphere of same 
radius. The total height of the toy is 15.5 cm. Find the total surface area of the toy. 

Answer: 
 

It can be observed that the radius of the conical part and the hemispherical part is same 
(i.e., 3.5 cm). 

Height of hemispherical part = Radius (r) = 3.5 = cm 

Height of conical part (h) = 15.5 −3.5 = 12 cm 

Total surface area of toy = CSA of conical part + CSA of hemispherical part 

Question 4: 
A cubical block of side 7 cm is surmounted by a hemisphere. What is the greatest 

diameter the hemisphere can have? Find the surface area of the solid.  

Answer: 
 

From the figure, it can be observed that the greatest diameter possible for such 
hemisphere is equal to the cube’s edge, i.e., 7cm. 

Radius (r) of hemispherical part = = 3.5cm 

Total surface area of solid = Surface area of cubical part + CSA of hemispherical part 

− Area of base of hemispherical part 

= 6 (Edge)2 − = 6 (Edge)2 +  

Question 5: 
A hemispherical depression is cut out from one face of a cubical wooden block such 
that the diameter l of the hemisphere is equal to the edge of the cube. Determine the 
surface area of the remaining solid. 

Answer: 

Diameter of hemisphere = Edge of cube = l 


Radius of hemisphere =  

Total surface area of solid = Surface area of cubical part + CSA of hemispherical part 

− Area of base of hemispherical part 

= 6 (Edge)2 − = 6 (Edge)2 +  

Question 6: 
A medicine capsule is in the shape of cylinder with two hemispheres stuck to each of 
its ends (see the given figure). The length of the entire capsule is 14 mm and the 

diameter of the capsule is 5 mm. Find its surface area.  

Answer: 

It can be observed that 

Radius (r) of cylindrical part = Radius (r) of hemispherical part 

Length of cylindrical part (h) = Length of the entire capsule − 2 × r 


= 14 − 5 = 9 cm 

Surface area of capsule = 2×CSA of hemispherical part + CSA of cylindrical part 

 
 
Question 7: 
A tent is in the shape of a cylinder surmounted by a conical top. If the height and 
diameter of the cylindrical part are 2.1 m and 4 m respectively, and the slant height of 
the top is 2.8 m, find the area of the canvas used for making the tent. Also, find the 
cost of the canvas of the tent at the rate of Rs 500 per m2. (Note that the base of the 

tent will not be covered with canvas.)  

Answer: 

Given that, 

Height (h) of the cylindrical part = 2.1 m 

Diameter of the cylindrical part = 4 m 


Radius of the cylindrical part = 2 m 

Slant height (l) of conical part = 2.8 m 

Area of canvas used = CSA of conical part + CSA of cylindrical part 

Cost of 1 m2 canvas = Rs 500 

Cost of 44 m2 canvas = 44 × 500 = 22000 

Therefore, it will cost Rs 22000 for making such a tent. 

Question 8: 
From a solid cylinder whose height is 2.4 cm and diameter 1.4 cm, a conical cavity of 
the same height and same diameter is hollowed out. Find the total surface area of the 

remaining solid to the nearest cm2.  

Answer: 

Given that, 

Height (h) of the conical part = Height (h) of the cylindrical part = 2.4 cm 

Diameter of the cylindrical part = 1.4 cm 

Therefore, radius (r) of the cylindrical part = 0.7 cm 


 

Total surface area of the remaining solid will be 

= CSA of cylindrical part + CSA of conical part + Area of cylindrical base 

The total surface area of the remaining solid to the nearest cm2 is 18 cm2. 

 
Question 9: 
A wooden article was made by scooping out a hemisphere from each end of a solid 
cylinder, as shown in given figure. If the height of the cylinder is 10 cm, and its base is 

of radius 3.5 cm, find the total surface area of the article.  

Answer: 

Given that, 

Radius (r) of cylindrical part = Radius (r) of hemispherical part = 3.5 cm 

Height of cylindrical part (h) = 10 cm 


Surface area of article = CSA of cylindrical part + 2 × CSA of hemispherical part 

 
 

Exercise 13.2 
Question 1: 
A solid is in the shape of a cone standing on a hemisphere with both their radii being 
equal to 1 cm and the height of the cone is equal to its radius. Find the volume of the 
solid in terms of π. 

Answer: 

Given that, 

Height (h) of conical part = Radius(r) of conical part = 1 cm 

Radius(r) of hemispherical part = Radius of conical part (r) = 1 cm 

Volume of solid = Volume of conical part + Volume of hemispherical part 

Question 2: 
Rachel, an engineering student, was asked to make a model shaped like a cylinder with 
two cones attached at its two ends by using a thin aluminum sheet. The diameter of the 
model is 3 cm and its length is 12 cm. if each cone has a height of 2 cm, find the 
volume of air contained in the model that Rachel made. (Assume the outer and inner 

dimensions of the model to be nearly the same.)  

Answer: 
 

From the figure, it can be observed that 

Height (h1) of each conical part = 2 cm 

Height (h2) of cylindrical part = 12 − 2 × Height of conical part 

= 12 − 2 ×2 = 8 cm 

Radius (r) of cylindrical part = Radius of conical part =  

Volume of air present in the model = Volume of cylinder + 2 × Volume of cones 

Question 3: 
A gulab jamun, contains sugar syrup up to about 30% of its volume. Find approximately 
how much syrup would be found in 45 gulab jamuns, each shaped like a cylinder with 
two hemispherical ends with length 5 cm and diameter 2.8 cm (see the given figure). 
 

Answer: 

It can be observed that 

Radius (r) of cylindrical part = Radius (r) of hemispherical part =  

Length of each hemispherical part = Radius of hemispherical part = 1.4 cm 

Length (h) of cylindrical part = 5 − 2 × Length of hemispherical part 

= 5 − 2 × 1.4 = 2.2 cm 

Volume of one gulab jamun = Vol. of cylindrical part + 2 × Vol. of hemispherical part 

Volume of 45 gulab jamuns = = 1,127.25 cm3 

Volume of sugar syrup = 30% of volume 


 

Question 4: 
A pen stand made of wood is in the shape of a cuboid with four conical depressions to 
hold pens. The dimensions of the cuboids are 15 cm by 10 cm by 3.5 cm. The radius of 
each of the depressions is 0.5 cm and the depth is 1.4 cm. Find the volume of wood in 

the entire stand (see the following figure).  

Answer: 

Depth (h) of each conical depression = 1.4 cm 

Radius (r) of each conical depression = 0.5 cm 

Volume of wood = Volume of cuboid − 4 × Volume of cones 

 
 
Question 5: 
A vessel is in the form of an inverted cone. Its height is 8 cm and the radius of its top, 
which is open, is 5 cm. It is filled with water up to the brim. When lead shots, each of 
which is a sphere of radius 0.5 cm are dropped into the vessel, one-fourth of the water 
flows out. Find the number of lead shots dropped in the vessel. 

Answer: 

Height (h) of conical vessel = 8 cm 

Radius (r1) of conical vessel = 5 cm 

Radius (r2) of lead shots = 0.5 cm 

Let n number of lead shots were dropped in the vessel. 

Volume of water spilled = Volume of dropped lead shots 

Hence, the number of lead shots dropped in the vessel is 100. 

Question 6: 
A solid iron pole consists of a cylinder of height 220 cm and base diameter 24 cm, 
which is surmounted by another cylinder of height 60 cm and radius 8 cm. Find the 
mass of the pole, given that 1 cm3 of iron has approximately 8 g mass. [Use π = 3.14] 

Answer: 

From the figure, it can be observed that 

Height (h1) of larger cylinder = 220 cm 

Radius (r1) of larger cylinder = = 12 cm 

Height (h2) of smaller cylinder = 60 cm 

Radius (r2) of smaller cylinder = 8 cm 

Mass of 1 iron = 8 g 

Mass of 111532.8 iron = 111532.8 × 8 = 892262.4 g = 892.262 kg 

Question 7: 
A solid consisting of a right circular cone of height 120 cm and radius 60 cm standing 
on a hemisphere of radius 60 cm is placed upright in a right circular cylinder full of 
water such that it touches the bottom. Find the volume of water left in the cylinder, if 

the radius of the cylinder is 60 cm and its height is 180 cm.  

Answer: 

Radius (r) of hemispherical part = Radius (r) of conical part = 60 cm 

Height (h2) of conical part of solid = 120 cm 

Height (h1) of cylinder = 180 cm 

Radius (r) of cylinder = 60 cm 

Volume of water left = Volume of cylinder − Volume of solid 

Question 8: 
A spherical glass vessel has a cylindrical neck 8 cm long, 2 cm in diameter; the 
diameter o the spherical part is 8.5 cm. By measuring the amount of water it holds, a 
child finds its volume to be 345 cm3. Check whether she is correct, taking the above as 
the inside measurements, and π = 3.14. 
Answer: 

Height (h) of cylindrical part = 8 cm 

Radius (r2) of cylindrical part = cm 

Radius (r1) spherical part =  

Volume of vessel = Volume of sphere + Volume of cylinder 

Hence, she is wrong. 

 
 
 
 
 
 
 
 
 
 
 
 
 
 
 
 

Exercise 13.3   
 
 
 
Question 1: 
A metallic sphere of radius 4.2 cm is melted and recast into the shape of a cylinder of 
radius 6 cm. Find the height of the cylinder. 

Answer: 

Radius (r1) of hemisphere = 4.2 cm 

Radius (r2) of cylinder = 6 cm 

Let the height of the cylinder be h. 

The object formed by recasting the hemisphere will be the same in volume. 

Volume of sphere = Volume of cylinder 


 

Hence, the height of the cylinder so formed will be 2.74 cm. 

Question 2: 
Metallic spheres of radii 6 cm, 8 cm, and 10 cm, respectively, are melted to form a 
single solid sphere. Find the radius of the resulting sphere. 

Answer: 

Radius (r1) of 1st sphere = 6 cm 

Radius (r2) of 2nd sphere = 8 cm 

Radius (r3) of 3rd sphere = 10 cm 

Let the radius of the resulting sphere be r. 

The object formed by recasting these spheres will be same in volume as the sum of the 
volumes of these spheres. 

Volume of 3 spheres = Volume of resulting sphere 

Therefore, the radius of the sphere so formed will be 12 cm. 

Question 3: 
A 20 m deep well with diameter 7 m is dug and the earth from digging is evenly spread 

out to form a platform 22 m by 14 m. Find the height of the platform.  


Answer: 

The shape of the well will be cylindrical. 

Depth (h) of well = 20 m 

Radius (r) of circular end of well =  

Area of platform = Length × Breadth = 22 × 14 m2 

Let height of the platform = H 

Volume of soil dug from the well will be equal to the volume of soil scattered on the 
platform. 

Volume of soil from well = Volume of soil used to make such platform 

Therefore, the height of such platform will be 2.5 m. 

Question 4: 
A well of diameter 3 m is dug 14 m deep. The earth taken out of it has been spread 
evenly all around it in the shape of a circular ring of width 4 m to form an embankment. 
Find the height of the embankment. 

Answer: 

The shape of the well will be cylindrical. 

Depth (h1) of well = 14 m 

Radius (r1) of the circular end of well =  

Width of embankment = 4 m 

From the figure, it can be observed that our embankment will be in a cylindrical shape 

having outer radius (r2) as and inner radius (r1) as . 

Let the height of embankment be h2. 

Volume of soil dug from well = Volume of earth used to form embankment 

 
 

Therefore, the height of the embankment will be 1.125 m. 

Question 5: 
A container shaped like a right circular cylinder having diameter 12 cm and height 15 
cm is full of ice cream. The ice cream is to be filled into cones of height 12 cm and 
diameter 6 cm, having a hemispherical shape on the top. Find the number of such 
cones which can be filled with ice cream. 

Answer: 

Height (h1) of cylindrical container = 15 cm 

Radius (r1) of circular end of container =  

Radius (r2) of circular end of ice-cream cone =  

Height (h2) of conical part of ice-cream cone = 12 cm 

Let n ice-cream cones be filled with ice-cream of the container. 

Volume of ice-cream in cylinder = n × (Volume of 1 ice-cream cone + Volume of 


hemispherical shape on the top) 

Therefore, 10 ice-cream cones can be filled with the ice-cream in the container. 

Question 6: 
How many silver coins, 1.75 cm in diameter and of thickness 2 mm, must be melted to 

form a cuboid of dimensions ?  

Answer: 

Coins are cylindrical in shape. 

Height (h1) of cylindrical coins = 2 mm = 0.2 cm 

Radius (r) of circular end of coins =  

Let n coins be melted to form the required cuboids. 

Volume of n coins = Volume of cuboids 

Therefore, the number of coins melted to form such a cuboid is 400. 

Question 7: 
A cylindrical bucket, 32 cm high and with radius of base 18 cm, is filled with sand. This 
bucket is emptied on the ground and a conical heap of sand is formed. If the height of 
the conical heap is 24 cm. Find the radius and slant height of the heap. 

Answer: 
 

Height (h1) of cylindrical bucket = 32 cm 

Radius (r1) of circular end of bucket = 18 cm 

Height (h2) of conical heap = 24 cm 

Let the radius of the circular end of conical heap be r2. 

The volume of sand in the cylindrical bucket will be equal to the volume of sand in the 
conical heap. 

Volume of sand in the cylindrical bucket = Volume of sand in conical heap 

r2 = = 36 cm 

Slant height =  
Therefore, the radius and slant height of the conical heap are 36 cm and
respectively. 

Question 8: 
Water in canal, 6 m wide and 1.5 m deep, is flowing with a speed of 10 km/h. how much 
area will it irrigate in 30 minutes, if 8 cm of standing water is needed? 

Answer: 

Consider an area of cross-section of canal as ABCD. 

Area of cross-section = 6 × 1.5 = 9 m2 

Speed of water = 10 km/h =  

Volume of water that flows in 1 minute from canal = =1500 m3 

Volume of water that flows in 30 minutes from canal = 30 × 1500 = 45000 m3 
 

Let the irrigated area be A. Volume of water irrigating the required area will be equal to 
the volume of water that flowed in 30 minutes from the canal. 

Vol. of water flowing in 30 minutes from canal = Vol. of water irrigating the reqd. area 

A = 562500 m2 

Therefore, area irrigated in 30 minutes is 562500 m2. 

Question 9: 
A farmer connects a pipe of internal diameter 20 cm form a canal into a cylindrical tank 
in her field, which is 10 m in diameter and 2 m deep. If water flows through the pipe at 
the rate of 3 km/h, in how much time will the tank be filled? 

Answer: 

Consider an area of cross-section of pipe as shown in the figure. 

Radius (r1) of circular end of pipe =  


Area of cross-section =  

Speed of water = 3 km/h =  

Volume of water that flows in 1 minute from pipe = 50 × = 0.5π m3 

Volume of water that flows in t minutes from pipe = t × 0.5π m3 

Radius (r2) of circular end of cylindrical tank = m 

Depth (h2) of cylindrical tank = 2 m 

Let the tank be filled completely in t minutes. 

Volume of water filled in tank in t minutes is equal to the volume of water flowed in t 
minutes from the pipe. 

Volume of water that flows in t minutes from pipe = Volume of water in tank 

t × 0.5π = π ×(r2)2 ×h2 

t × 0.5 = 52 ×2 

t = 100 

Therefore, the cylindrical tank will be filled in 100 minutes. 

 
 

Exercise 13.4 
 

Question 1: 
A drinking glass is in the shape of a frustum of a cone of height 14 cm. The diameters 

of its two circular ends are 4 cm and 2 cm. Find the capacity of the glass.  

Answer: 

Radius (r1) of  

Radius (r2) of  

Capacity of glass = Volume of frustum of cone 


 

Therefore, the capacity of the glass is . 

Question 2: 
The slant height of a frustum of a cone is 4 cm and the perimeters (circumference) of 
its circular ends are 18 cm and 6 cm. find the curved surface area of the frustum. 

Answer: 

Perimeter of upper circular end of frustum = 18 cm 

2Ï€r1 =18 

Perimeter of lower end of frustum = 6 cm 

2Ï€r2 = 6 

 
Slant height (l) of frustum = 4 cm 

CSA of frustum = Ï€ (r1 + r2) l 

Therefore, the curved surface area of the frustum is 48 cm2. 

Question 3: 
A fez, the cap used by the Turks, is shaped like the frustum of a cone (see the figure 
given below). If its radius on the open side is 10 cm, radius at the upper base is 4 cm 

and its slant height is 15 cm, find the area of material use for making it.  

Answer: 

Radius (r2) at upper circular end = 4 cm 

Radius (r1) at lower circular end = 10 cm 

Slant height (l) of frustum = 15 cm 


Area of material used for making the fez = CSA of frustum + Area of upper circular end 

= π (10 + 4) 15 + π (4)2 

= π (14) 15 + 16 π 

Therefore, the area of material used for making it is . 

Question 4: 
A container, opened from the top and made up of a metal sheet, is in the form of a 
frustum of a cone of height 16 cm with radii of its lower and upper ends as 8 cm and 20 
cm respectively. Find the cost of the milk which can completely fill the container, at the 
rate of Rs.20 per litre. Also find the cost of metal sheet used to make the container, if it 
costs Rs.8 per 100 cm2. [Take π = 3.14] 

Answer: 

Radius (r1) of upper end of container = 20 cm 

Radius (r2) of lower end of container = 8 cm 

Height (h) of container = 16 cm 


Slant height (l) of frustum =  

Capacity of container = Volume of frustum 

Cost of 1 litre milk = Rs 20 

Cost of 10.45 litre milk = 10.45 × 20 

= Rs 209 

Area of metal sheet used to make the container 

= π (20 + 8) 20 + π (8)2 

= 560 π + 64 π = 624 π cm2 

Cost of 100 cm2 metal sheet = Rs 8 


 

Therefore, the cost of the milk which can completely fill the container is 

Rs 209 and the cost of metal sheet used to make the container is Rs 156.75. 

 
Question 5: 
A metallic right circular cone 20 cm high and whose vertical angle is 60° is cut into two 
parts at the middle of its height by a plane parallel to its base. If the frustum so 

obtained is drawn into a wire of diameter cm, find the length of the wire. 

Answer: 

In ΔAEG, 

In ΔABD, 
 

Radius (r1) of upper end of frustum = cm 

Radius (r2) of lower end of container =  

Height (h) of container = 10 cm 

Volume of frustum  

Radius (r) of wire =  

Let the length of wire be l. 

Volume of wire = Area of cross-section × Length 

= (πr2) (l) 

Volume of frustum = Volume of wire 


 

 
 

 
 
 
 
 
 
 
 
 
 
 
 
 
 
 
 
 
 
Exercise 13.5 
 
Question 1: 
A copper wire, 3 mm in diameter, is wound about a cylinder whose length is 12 cm, and 
diameter 10 cm, so as to cover the curved surface of the cylinder. Find the length and 
mass of the wire, assuming the density of copper to be 8.88 g per cm3. 

Answer: 

It can be observed that 1 round of wire will cover 3 mm height of cylinder. 

Length of wire required in 1 round = Circumference of base of cylinder 

= 2πr = 2π × 5 = 10π 

Length of wire in 40 rounds = 40 × 10π 

= 1257.14 cm = 12.57 m 

Radius of wire  
Volume of wire = Area of cross-section of wire × Length of wire 

= π(0.15)2 × 1257.14 

= 88.898 cm3 

Mass = Volume × Density 

= 88.898 × 8.88 

= 789.41 gm 

Question 2: 
A right triangle whose sides are 3 cm and 4 cm (other than hypotenuse) is made to 
revolve about its hypotenuse. Find the volume and surface area of the double cone so 
formed. (Choose value of π as found appropriate.) 

Answer: 

The double cone so formed by revolving this right-angled triangle ABC about its 
hypotenuse is shown in the figure. 

Hypotenuse  

= 5 cm 
Area of ΔABC  

Volume of double cone = Volume of cone 1 + Volume of cone 2 

= 30.14 cm3 

Surface area of double cone = Surface area of cone 1 + Surface area of cone 2 

= πrl1 + πrl2 

= 52.75 cm2 

Question 3: 

A cistern, internally measuring 150 cm × 120 cm × 110 cm, has 129600 cm3 of water in 
it. Porous bricks are placed in the water until the cistern is full to the brim. Each brick 
absorbs one-seventeenth of its own volume of water. How many bricks can be put in 
without overflowing the water, each brick being 22.5 cm × 7.5 cm × 6.5 cm? 

Answer: 

Volume of cistern = 150 × 120 × 110 

= 1980000 cm3 
Volume to be filled in cistern = 1980000 − 129600 

= 1850400 cm3 

Let n numbers of porous bricks were placed in the cistern. 

Volume of n bricks = n × 22.5 × 7.5 × 6.5 

= 1096.875n 

As each brick absorbs one-seventeenth of its volume, therefore, volume absorbed by 

these bricks  

n = 1792.41 

Therefore, 1792 bricks were placed in the cistern. 

Question 4: 
In one fortnight of a given month, there was a rainfall of 10 cm in a river valley. If the 
area of the valley is 7280 km2, show that the total rainfall was approximately equivalent 
to the addition to the normal water of three rivers each 1072 km long, 75 m wide and 3 
m deep. 

Answer: 

Area of the valley = 7280 km2

If there was a rainfall of 10 cm in the valley then amount of rainfall in the valley = Area of
the valley × 10 cm

2
Amount of rainfall in the valley = 7280 km × 10 cm

=7280×1000m2×10100m=7280×105m3=7.28×108m3 

Length of each river, l = 1072 km = 1072 × 1000 m = 1072000 m


Breadth of each river, b = 75 m

Depth of each river, h = 3 m

Volume of each river = l × b × h

= 1072000 × 75 × 3 m3

= 2.412 × 108 m3

Volume of three such rivers = 3 × Volume of each river

= 3 × 2.412 × 108 m3

= 7.236 × 108 m3

Thus, the total rainfall is approximately same as the volume of the three rivers. 

Question 5: 
An oil funnel made of tin sheet consists of a 10 cm long cylindrical portion attached to 
a frustum of a cone. If the total height is 22 cm, diameter of the cylindrical portion is 8 
cm and the diameter of the top of the funnel is 18 cm, find the area of the tin sheet 
required to make the funnel (see the given figure). 

Answer: 

Radius (r1) of upper circular end of frustum part  


Radius (r2) of lower circular end of frustum part = Radius of circular end of cylindrical 

part 

Height (h1) of frustum part = 22 − 10 = 12 cm 

Height (h2) of cylindrical part = 10 cm 

Slant height (l) of frustum part  

Area of tin sheet required = CSA of frustum part + CSA of cylindrical part 

Question 6: 
Derive the formula for the curved surface area and total surface area of the frustum of 
cone. 

Answer: 
 

Let ABC be a cone. A frustum DECB is cut by a plane parallel to its base. Let r1 and r2 be 
the radii of the ends of the frustum of the cone and h be the height of the frustum of the 
cone. 

In ΔABG and ΔADF, DF||BG 

∴ ΔABG ∼ ΔADF 

CSA of frustum DECB = CSA of cone ABC − CSA cone ADE 


 

CSA of frustum =  

Question 7: 
Derive the formula for the volume of the frustum of a cone. 

Answer: 

Let ABC be a cone. A frustum DECB is cut by a plane parallel to its base. 
Let r1 and r2 be the radii of the ends of the frustum of the cone and h be the height of the 
frustum of the cone. 

In ΔABG and ΔADF, DF||BG 

∴ ΔABG ∼ ΔADF 

Volume of frustum of cone = Volume of cone ABC − Volume of cone ADE 

 
 

 
NCERT Solutions for Class 10 Maths Unit 14
Statistics Class 10
Unit 14 Statistics Exercise 14.1, 14.2, 14.3, 14.4 Solutions

Exercise 14.1 : Solutions of Questions on Page Number : 270


Q1 :

A survey was conducted by a group of students as a part of their environment awareness programme, in
which they collected the following data regarding the number of plants in 20 houses in a locality. Find the
mean number of plants per house.

Number of plants 0-2 2-4 4-6 6-8 8 - 10 10 - 12 12 - 14

Number of houses 1 2 1 5 6 2 3
Which method did you use for finding the mean, and why?

Answer :

To find the class mark (xi) for each interval, the following relation is used.

Class mark (xi) =

xi andfixi can be calculated as follows.

Number of plants Number of houses x i fx


i i

(f )i

0-2 1 1 1×1=1
2-4 2 3 2×3=6
4-6 1 5 1×5=5
6-8 5 7 5 × 7 = 35
8 - 10 6 9 6 × 9 = 54
10 - 12 2 11 2 ×11 = 22
12 - 14 3 13 3 × 13 = 39
Total 20 162
From the table, it can be observed that
Mean,

Therefore, mean number of plants per house is 8.1.

Here, direct method has been used as the values of class marks (xi) and fi are small.

Q2 :

Consider the following distribution of daily wages of 50 worker of a factory.

Daily wages (in Rs) 100 - 120 120 - 140 140 - 1 60 160 - 180 180 - 200

Number of workers 12 14 8 6 10
Find the mean daily wages of the workers of the factory by using an appropriate method.

Answer :

To find the class mark for each interval, the following relation is used.

Class size (h) of this data = 20

Taking 150 as assured mean (a), di, ui, and fiui can be calculated as follows.

Daily wages
Number of workers (f ) i x i d = x - 150
i i fu
i i
(in Rs)
100 - 120 12 110 - 40 -2 - 24
120 - 140 14 130 - 20 -1 - 14
140 - 160 8 150 0 0 0
160 - 180 6 170 20 1 6
180 - 200 10 190 40 2 20
Total 50 - 12
From the table, it can be observed that
Therefore, the mean daily wage of the workers of the factory is Rs 145.20.

Q3 :

The following distribution shows the daily pocket allowance of children of a locality. The mean pocket
allowance is Rs.18. Find the missing frequency f.

Daily pocket allowance (in Rs)


11 - 13 13 - 15 15 - 17 17 - 19 19 - 21 21 - 23 23 - 25

Number of workers 7 6 9 13 f 5 4

Answer :

To find the class mark (xi) for each interval, the following relation is used.

Given that, mean pocket allowance,

Taking 18 as assured mean (a), di and fidi are calculated as follows.

Daily pocket allowance Number of children


Class mark x i d = x - 18
i i fd
i i
(in Rs) f i

11 - 13 7 12 -6 - 42
13 - 15 6 14 -4 - 24
15 - 17 9 16 -2 - 18
17 - 19 13 18 0 0
19 - 21 f 20 2 2f
21 - 23 5 22 4 20
23 - 25 4 24 6 24
Total 2f - 40

From the table, we obtain

Hence, the missing frequency, f, is 20.

Q4 :

Thirty women were examined in a hospital by a doctor and the number of heart beats per minute were
recorded and summarized as follows. Fine the mean heart beats per minute for these women, choosing a
suitable method.

Number of heart beats per minute


65 - 68 68 - 71 71 - 74 74 - 77 77 - 80 80 - 83 83 - 86

Number of women 2 4 3 8 7 4 2

Answer :

To find the class mark of each interval (xi), the following relation is used.

Class size, h, of this data = 3

Taking 75.5 as assumed mean (a), di, ui, fiui are calculated as follows.
Number of women
Number of heart beats per minute x i d = x - 75.5
i i fu
i i
f i

65 - 68 2 66.5 -9 -3 -6
68 - 71 4 69.5 -6 -2 -8
71 - 74 3 72.5 -3 -1 -3
74 - 77 8 75.5 0 0 0
77 - 80 7 78.5 3 1 7
80 - 83 4 81.5 6 2 8
83 - 86 2 84.5 9 3 6
Total 30 4
From the table, we obtain

Therefore, mean hear beats per minute for these women are 75.9 beats per minute.

Q5 :

In a retail market, fruit vendors were selling mangoes kept in packing boxes. These boxes contained varying
number of mangoes. The following was the distribution of mangoes according to the number of boxes.

Number of mangoes 50 - 52 53 - 55 56 - 58 59 - 61 62 - 64

Number of boxes 15 110 135 115 25


Find the mean number of mangoes kept in a packing box. Which method of finding the mean did you
choose?

Answer :

Number of mangoes Number of boxes f i


50 - 52 15
53 - 55 110
56 - 58 135
59 - 61 115
62 - 64 25
It can be observed that class intervals are not continuous. There is a gap of 1 between two class intervals. Therefore,

has to be added to the upper class limit and has to be subtracted from the lower class limit of each interval.

Class mark (xi) can be obtained by using the following relation.

Class size (h) of this data = 3

Taking 57 as assumed mean (a), di, ui, fiui are calculated as follows.

Class interval f i x i d = x - 57
i i fu
i i

49.5 - 52.5 15 51 -6 -2 - 30
52.5 - 55.5 110 54 -3 -1 - 110
55.5 - 58.5 135 57 0 0 0
58.5 - 61.5 115 60 3 1 115
61.5 - 64.5 25 63 6 2 50
Total 400 25
It can be observed that
Mean number of mangoes kept in a packing box is 57.19.

Step deviation method is used here as the values of fi, di are big and also, there is a common

Q6 :

The table below shows the daily expenditure on food of 25 households in a locality.

Daily expenditure (in Rs) 100 - 150 150 - 200 200 - 250 250 - 300 300 - 350

Number of households 4 5 12 2 2
Find the mean daily expenditure on food by a suitable method.

Answer :

To find the class mark (xi) for each interval, the following relation is used.

Class size = 50

Taking 225 as assumed mean (a), di, ui, fiui are calculated as follows.

Daily expenditure (in Rs) f i x i d = x - 225


i i fu
i i

100 - 150 4 125 - 100 -2 -8


150 - 200 5 175 - 50 -1 -5
200 - 250 12 225 0 0 0
250 - 300 2 275 50 1 2
300 - 350 2 325 100 2 4
Total 25 -7
From the table, we obtain
Therefore, mean daily expenditure on food is Rs 211.

Q7 :

To find out the concentration of SO2 in the air (in parts per million, i.e., ppm), the data was collected for 30
localities in a certain city and is presented below:

concentration of SO (in ppm)


2 Frequency
0.00 - 0.04 4
0.04 - 0.08 9
0.08 - 0.12 9
0.12 - 0.16 2
0.16 - 0.20 4
0.20 - 0.24 2
Find the mean concentration of SO2 in the air.

Answer :

To find the class marks for each interval, the following relation is used.

Class size of this data = 0.04

Taking 0.14 as assumed mean (a), di, ui,fiui are calculated as follows.

Concentration of SO (in ppm) 2 Frequency Class mark fu


i i
d = x - 0.14
i i

f i x i

0.00 - 0.04 4 0.02 - 0.12 -3 - 12


0.04 - 0.08 9 0.06 - 0.08 -2 - 18
0.08 - 0.12 9 0.10 - 0.04 -1 -9
0.12 - 0.16 2 0.14 0 0 0
0.16 - 0.20 4 0.18 0.04 1 4
0.20 - 0.24 2 0.22 0.08 2 4
Total 30 - 31
From the table, we obtain

Therefore, mean concentration of SO2 in the air is 0.099 ppm.

Q8 :

A class teacher has the following absentee record of 40 students of a class for the whole term. Find the mean
number of days a student was absent.

Number of days 0-6 6 - 10 10 - 14 14 - 20 20 - 28 28 - 38 38 - 40

Number of students 11 10 7 4 4 3 1

Answer :

To find the class mark of each interval, the following relation is used.

Taking 17 as assumed mean (a), di and fidi are calculated as follows.

Number of days Number of students x i d = x - 17


i i fd
i i

f i

0-6 11 3 - 14 - 154
6 - 10 10 8 -9 - 90
10 - 14 7 12 -5 - 35
14 - 20 4 17 0 0
20 - 28 4 24 7 28
28 - 38 3 33 16 48
38 - 40 1 39 22 22
Total 40 - 181
From the table, we obtain

Therefore, the mean number of days is 12.48 days for which a student was absent.

Q9 :

The following table gives the literacy rate (in percentage) of 35 cities. Find the mean literacy rate.

Literacy rate (in %) 45 - 55 55 - 65 65 - 75 75 - 85 85 - 95

Number of cities 3 10 11 8 3

Answer :

To find the class marks, the following relation is used.

Class size (h) for this data = 10

Taking 70 as assumed mean (a), di, ui, and fiui are calculated as follows.

Literacy rate (in %) Number of cities x i fu i i


d = x - 70
i i

f i
45 - 55 3 50 - 20 -2 -6
55 - 65 10 60 - 10 -1 - 10
65 - 75 11 70 0 0 0
75 - 85 8 80 10 1 8
85 - 95 3 90 20 2 6
Total 35 -2
From the table, we obtain

Therefore, mean literacy rate is 69.43%.

Exercise 14.2 : Solutions of Questions on Page Number : 275


Q1 :

The following table shows the ages of the patients admitted in a hospital during a year:

age (in years) 5 - 15 15 - 25 25 - 35 35 - 45 45 - 55 55 - 65

Number of patients 6 11 21 23 14 5
Find the mode and the mean of the data given above. Compare and interpret the two measures of central
tendency.

Answer :

To find the class marks (xi), the following relation is used.


Taking 30 as assumed mean (a), di and fidiare calculated as follows.

Age (in years) Number of patients Class mark d = x - 30


i i fd
i i

f i x i

5 - 15 6 10 - 20 - 120
15 - 25 11 20 - 10 - 110
25 - 35 21 30 0 0
35 - 45 23 40 10 230
45 - 55 14 50 20 280
55 - 65 5 60 30 150
Total 80 430
From the table, we obtain

Mean of this data is 35.38. It represents that on an average, the age of a patient admitted to hospital was 35.38
years.

It can be observed that the maximum class frequency is 23 belonging to class interval 35 - 45.

Modal class = 35 - 45

Lower limit (l) of modal class = 35

Frequency (f1) of modal class = 23

Class size (h) = 10

Frequency (f0) of class preceding the modal class = 21

Frequency (f2) of class succeeding the modal class = 14

Mode =
Mode is 36.8. It represents that the age of maximum number of patients admitted in hospital was 36.8 years.

Q2 :

The following data gives the information on the observed lifetimes (in hours) of 225 electrical components:

Lifetimes (in hours) 0 - 20 20 - 40 40 - 60 60 - 80 80 - 100 100 - 120

Frequency 10 35 52 61 38 29
Determine the modal lifetimes of the components.

Answer :

From the data given above, it can be observed that the maximum class frequency is 61, belonging to class interval 60
- 80.

Therefore, modal class = 60 - 80

Lower class limit (l) of modal class = 60

Frequency (f1) of modal class = 61

Frequency (f0) of class preceding the modal class = 52

Frequency (f2) of class succeeding the modal class = 38

Class size (h) = 20


Therefore, modal lifetime of electrical components is 65.625 hours.

Q3 :

The following data gives the distribution of total monthly household expenditure of 200 families of a village.
Find the modal monthly expenditure of the families. Also, find the mean monthly expenditure.

Expenditure (in Rs) Number of families


1000 - 1500 24
1500 - 2000 40
2000 - 2500 33
2500 - 3000 28
3000 - 3500 30
3500 - 4000 22
4000 - 4500 16
4500 - 5000 7

Answer :

It can be observed from the given data that the maximum class frequency is 40, belonging to 1500 - 2000 intervals.

Therefore, modal class = 1500 - 2000

Lower limit (l) of modal class = 1500

Frequency (f1) of modal class = 40

Frequency (f0) of class preceding modal class = 24

Frequency (f2) of class succeeding modal class = 33

Class size (h) = 500


Therefore, modal monthly expenditure was Rs 1847.83.

To find the class mark, the following relation is used.

Class size (h) of the given data = 500

Taking 2750 as assumed mean (a), di, ui, and fiuiare calculated as follows.

Expenditure (in Rs) Number of families x i d = x - 2750


i i fu
i i

f i

1000 - 1500 24 1250 - 1500 -3 - 72


1500 - 2000 40 1750 - 1000 -2 - 80
2000 - 2500 33 2250 - 500 -1 - 33
2500 - 3000 28 2750 0 0 0
3000 - 3500 30 3250 500 1 30
3500 - 4000 22 3750 1000 2 44
4000 - 4500 16 4250 1500 3 48
4500 - 5000 7 4750 2000 4 28
Total 200 - 35
From the table, we obta

Q4 :
The following distribution gives the state-wise teacher-student ratio in higher secondary schools of India.
Find the mode and mean of this data. Interpret the two measures.

Number of students per teacher Number of states/U.T


15 - 20 3
20 - 25 8
25 - 30 9
30 - 35 10
35 - 40 3
40 - 45 0
45 - 50 0
50 - 55 2

Answer :

It can be observed from the given data that the maximum class frequency is 10 belonging to class interval 30 - 35.

Therefore, modal class = 30 - 35

Class size (h) = 5

Lower limit (l) of modal class = 30

Frequency (f1) of modal class = 10

Frequency (f0) of class preceding modal class = 9

Frequency (f2) of class succeeding modal class = 3

It represents that most of the states/U.T have a teacher-student ratio as 30.6.

To find the class marks, the following relation is used.


Taking 32.5 as assumed mean (a), di, ui, and fiui are calculated as follows.

Number of students per teacher Number of states/U.T x i d = x - 32.5


i i fu
i i

(f )
i

15 - 20 3 17.5 - 15 -3 -9
20 - 25 8 22.5 - 10 -2 - 16
25 - 30 9 27.5 -5 -1 -9
30 - 35 10 32.5 0 0 0
35 - 40 3 37.5 5 1 3
40 - 45 0 42.5 10 2 0
45 - 50 0 47.5 15 3 0
50 - 55 2 52.5 20 4 8
Total 35 - 23

Therefore, mean of the data is 29.2.

It repre

Q5 :

The given distribution shows the number of runs scored by some top batsmen of the world in one-day
international cricket matches.

Runs scored Number of batsmen


3000 - 4000 4
4000 - 5000 18
5000 - 6000 9
6000 - 7000 7
7000 - 8000 6
8000 - 9000 3
9000 - 10000 1
10000 - 11000 1
Find the mode of the data.

Answer :

From the given data, it can be observed that the maximum class frequency is 18, belonging to class interval 4000 -
5000.

Therefore, modal class = 4000 - 5000

Lower limit (l) of modal class = 4000

Frequency (f1) of modal class = 18

Frequency (f0) of class preceding modal class = 4

Frequency (f2) of class succeeding modal class = 9

Class size (h) = 1000

Therefore, mode of the given data is 4608.7 runs.

Q6 :

A student noted the number of cars passing through a spot on a road for 100 periods each of 3 minutes and
summarised it in the table given below. Find the mode of the data:

Number of cars 0 - 10 10 - 20 20 - 30 30 - 40 40 - 50 50 - 60 60 - 70 70 - 80

Frequency 7 14 13 12 20 11 15 8

Answer :

From the given data, it can be observed that the maximum class frequency is 20, belonging to 40 - 50 class intervals.
Therefore, modal class = 40 - 50

Lower limit (l) of modal class = 40

Frequency (f1) of modal class = 20

Frequency (f0) of class preceding modal class = 12

Frequency (f2) of class succeeding modal class = 11

Class size = 10

Therefore, mode of this data is 44.7 cars.

Exercise 14.3 : Solutions of Questions on Page Number : 287


Q1 :

The following frequency distribution gives the monthly consumption of electricity of 68 consumers of a
locality. Find the median, mean and mode of the data and compare them.

Monthly consumption (in units) Number of consumers


65 - 85 4
85 - 105 5
105 - 125 13
125 - 145 20
145 - 165 14
165 - 185 8
185 - 205 4

Answer :
To find the class marks, the following relation is used.

Taking 135 as assumed mean (a), di, ui,fiui are calculated according to step deviation method as follows.

Monthly consumption (in Number of consumers x class


i d= x -
i i

units) (f ) i mark 135

65 - 85 4 75 - 60 -3 - 12
85 - 105 5 95 - 40 -2 - 10
105 - 125 13 115 - 20 -1 - 13
125 - 145 20 135 0 0 0
145 - 165 14 155 20 1 14
165 - 185 8 175 40 2 16
185 - 205 4 195 60 3 12
Total 68 7
From the table, we obtain

From the table, it can be observed that the maximum class frequency is 20, belonging to class interval 125 - 145.

Modal class = 125 - 145

Lower limit (l) of modal class = 125

Class size (h) = 20

Frequency (f1) of modal class = 20

Frequency (f0) of class preceding modal class = 13

Frequency (f2) of class succeeding the modal cla


Q2 :

If the median of the distribution is given below is 28.5, find the values of x and y.

Class interval Frequency


0 - 10 5
10 - 20 x
20 - 30 20
30 - 40 15
40 - 50 y
50 - 60 5
Total 60

Answer :

The cumulative frequency for the given data is calculated as follows.

Class interval Frequency Cumulative frequency


0 - 10 5 5
10 - 20 x 5+ x
20 - 30 20 25 + x
30 - 40 15 40 + x
40 - 50 y 40+ x + y
50 - 60 5 45 + x + y
Total (n) 60
From the table, it can be observed that n = 60

45 + x + y = 60

x + y = 15 (1)

Median of the data is given as 28.5 which lies in interval 20 - 30.

Therefore, median class = 20 - 30

Lower limit (l) of median class = 20

Cumulative frequency (cf) of class preceding the median class = 5 + x


Frequency (f) of median class = 20

Class size (h) = 10

From equation (1),

8 + y = 15

y=7

Hence, the values of x and y are 8 and 7 respectively.

Q3 :

A life insurance agent found the following data for distribution of ages of 100 policy holders. Calculate the
median age, if policies are given only to persons having age 18 years onwards but less than 60 year.

Age (in years) Number of policy holders


Below 20 2
Below 25 6
Below 30 24
Below 35 45
Below 40 78
Below 45 89
Below 50 92
Below 55 98
Below 60 100
Answer :

Here, class width is not the same. There is no requirement of adjusting the frequencies according to class intervals.
The given frequency table is of less than type represented with upper class limits. The policies were given only to
persons with age 18 years onwards but less than 60 years. Therefore, class intervals with their respective cumulative
frequency can be defined as below.

Age (in years) Number of policy holders (f ) i Cumulative frequency (cf)


18 - 20 2 2
20 - 25 6-2=4 6
25 - 30 24 - 6 = 18 24
30 - 35 45 - 24 = 21 45
35 - 40 78 - 45 = 33 78
40 - 45 89 - 78 = 11 89
45 - 50 92 - 89 = 3 92
50 - 55 98 - 92 = 6 98
55 - 60 100 - 98 = 2 100
Total (n)
From the table, it can be observed that n = 100.

Cumulative frequency (cf) just greater than is 78, belonging to interval 35 - 40.

Therefore, median class = 35 - 40

Lower limit (l) of median class = 35

Class size (h) = 5

Frequency (f) of median class = 33

Cumulative frequency (cf) of class preceding median class = 45


Therefore, median age is 35.76 years.

Q4 :

The lengths of 40 leaves of a plant are measured correct to the nearest millimeter, and the data obtained is
represented in the following table:

Length (in mm) Number or leaves f i

118 - 126 3
127 - 135 5
136 - 144 9
145 - 153 12
154 - 162 5
163 - 171 4
172 - 180 2
Find the median length of the leaves.

(Hint: The data needs to be converted to continuous classes for finding the median, since the formula
assumes continuous classes. The classes then change to 117.5 - 126.5, 126.5 - 135.5… 171.5 - 180.5)

Answer :

The given data does not have continuous class intervals. It can be observed that the difference between two class

intervals is 1. Therefore, has to be added and subtracted to upper class limits and lower class limits
respectively.

Continuous class intervals with respective cumulative frequencies can be represented as follows.

Length (in mm) Number or leaves f i Cumulative frequency


117.5 - 126.5 3 3
126.5 - 135.5 5 3+5=8
135.5 - 144.5 9 8 + 9 = 17
144.5 - 153.5 12 17 + 12 = 29
153.5 - 162.5 5 29 + 5 = 34
162.5 - 171.5 4 34 + 4 = 38
171.5 - 180.5 2 38 + 2 = 40

From the table, it can be observed that the cumulative frequency just greater than is 29,
belonging to class interval 144.5 - 153.5.

Median class = 144.5 - 153.5

Lower limit (l) of median class = 144.5

Class size (h) = 9

Frequency (f) of median class = 12

Cumulative frequency (cf) of class preceding median class = 17

Median

Therefore, median length of leaves is 146.75 mm.

Q5 :

Find the following table gives the distribution of the life time of 400 neon lamps:

Life time (in hours) Number of lamps


1500 - 2000 14
2000 - 2500 56
2500 - 3000 60
3000 - 3500 86
3500 - 4000 74
4000 - 4500 62
4500 - 5000 48
Find the median life time of a lamp.

Answer :

Thecumulative frequencies with their respective class intervals are as follows.

Life time Number of lamps (f ) i Cumulative frequency


1500 - 2000 14 14
2000 - 2500 56 14 + 56 = 70
2500 - 3000 60 70 + 60 = 130
3000 - 3500 86 130 + 86 = 216
3500 - 4000 74 216 + 74 = 290
4000 - 4500 62 290 + 62 = 352
4500 - 5000 48 352 + 48 = 400
Total (n) 400

It can be observed that the cumulative frequency just greater than is 216, belonging to class
interval 3000 - 3500.

Median class = 3000 - 3500

Lower limit (l) of median class = 3000

Frequency (f) of median class = 86

Cumulative frequency (cf) of class preceding median class = 130

Class size (h) = 500

Median
= 3406.976

Therefore, median life time of lamps is 3406.98 hours.

Q6 :

100 surnames were randomly picked up from a local telephone directory and the frequency distribution of
the number of letters in the English alphabets in the surnames was obtained as follows:

Number of letters 1-4 4-7 7 - 10 10 - 13 13 - 16 16 - 19

Number of surnames 6 30 40 6 4 4
Determine the median number of letters in the surnames. Find the mean number of letters in the surnames?
Also, find the modal size of the surnames.

Answer :

The cumulative frequencies with their respective class intervals are as follows.

Number of letters Frequency (f ) i Cumulative frequency


1-4 6 6
4-7 30 30 + 6 = 36
7 - 10 40 36 + 40 = 76
10 - 13 16 76 + 16 = 92
13 - 16 4 92 + 4 = 96
16 - 19 4 96 + 4 = 100
Total (n) 100

It can be observed that the cumulative frequency just greater than is 76, belonging to class
interval 7 - 10.

Median class = 7 - 10

Lower limit (l) of median class = 7

Cumulative frequency (cf) of class preceding median class = 36

Frequency (f) of median class = 40


Class size (h) = 3

Median

= 8.05

To find the class marks of the given class intervals, the following relation is used.

Taking 11.5 as assumed mean (a), di, ui, and fiui are calculated according to step deviation method as follows.

Number of letters Number of surnames x i d = x - 11.5


i i fui i

f i

1-4 6 2.5 -9 -3 - 18
4-7 30 5.5 -6 -2 - 60
7 - 10 40 8.5 -3 -1 - 40

10 - 13 16

Q7 :

The distribution below gives the weights of 30 students of a class. Find the median weight of the students.

Weight (in kg) 40 - 45 45 - 50 50 - 55 55 - 60 60 - 65 65 - 70 70 - 75

Number of students 2 3 8 6 6 3 2

Answer :

The cumulative frequencies with their respective class intervals are as follows.

Weight (in kg) Frequency (fi) Cumulative frequency


40 - 45 2 2
45 - 50 3 2+3=5
50 - 55 8 5 + 8 = 13
55 - 60 6 13 + 6 = 19
60 - 65 6 19 + 6 = 25
65 - 70 3 25 + 3 = 28
70 - 75 2 28 + 2 = 30
Total (n) 30

Cumulative frequency just greater than is 19, belonging to class interval 55 - 60.

Median class = 55 - 60

Lower limit (l) of median class = 55

Frequency (f) of median class = 6

Cumulative frequency (cf) of median class = 13

Class size (h) = 5

Median

= 56.67

Therefore, median weight is 56.67 kg.

Exercise 14.4 : Solutions of Questions on Page Number : 293


Q1 :

The following distribution gives the daily income of 50 workers of a factory.

Daily income (in Rs) 100 - 120 120 - 140 140 - 160 160 - 180 180 - 200

Number of workers 12 14 8 6 10
Convert the distribution above to a less than type cumulative frequency distribution, and draw its ogive.

Answer :
The frequency distribution table of less than type is as follows.

Daily income (in Rs) Cumulative frequency


(upper class limits)
Less than 120 12
Less than 140 12 + 14 = 26
Less than 160 26 + 8 = 34
Less than 180 34 + 6 = 40
Less than 200 40 + 10 = 50
Taking upper class limits of class intervals on x-axis and their respective frequencies on y-axis, its ogive can be
drawn as follows.

Q2 :

During the medical check-up of 35 students of a class, their weights were recorded as follows:

Weight (in kg) Number of students


Less than 38 0
Less than 40 3
Less than 42 5
Less than 44 9
Less than 46 14
Less than 48 28
Less than 50 32
Less than 52 35
Draw a less than type ogive for the given data. Hence obtain the median weight from the graph verify the
result by using the formula.

Answer :

The given cumulative frequency distributions of less than type are

Weight (in kg) Number of students


upper class limits (cumulative frequency)
Less than 38 0
Less than 40 3
Less than 42 5
Less than 44 9
Less than 46 14
Less than 48 28
Less than 50 32
Less than 52 35
Taking upper class limits on x-axis and their respective cumulative frequencies on y-axis, its ogive can be drawn as
follows.

Here, n = 35

So, = 17.5

Mark the point A whose ordinate is 17.5 and its x-coordinate is 46.5. Therefore, median of this data is 46.5.
It can be observed that the difference between two consecutive upper class limits is 2. The class marks with their
respective frequencies are obtained as below.

Weight (in kg) Frequency (f) Cumulative frequency


Less than 38 0 0
38 - 40 3-0=3 3
40 - 42 5-3=2 5
42 - 44 9-5=4 9
44 - 46 14 - 9 = 5 14
46 - 48 28 - 14 = 14 28
48 - 50 32 - 28 = 4 32
50 - 52 35 - 32 = 3 35
Total (n) 35

The cumulative frequency just greater than is 28, belonging to class interval 46 - 48.

Median class = 46 - 48

Lower class limit (l) of median class = 46

Frequency (f) of median class = 14

Cumulative frequency (cf) of class preceding median class = 14

Class size (h) = 2


Q3 :

The following table gives production yield per hectare of wheat of 100 farms of a village.

Production yield (in kg/ha) 50 - 55 55 - 60 60 - 65 65 - 70 70 - 75 75 - 80

Number of farms 2 8 12 24 38 16
Change the distribution to a more than type distribution and draw ogive.

Answer :

The cumulative frequency distribution of more than type can be obtained as follows.

Production yield Cumulative frequency


(lower class limits)
more than or equal to 50 100
more than or equal to 55 100 - 2 = 98
more than or equal to 60 98 - 8 = 90
more than or equal to 65 90 - 12 = 78
more than or equal to 70 78 - 24 = 54
more than or equal to 75 54 - 38 = 16
Taking the lower class limits on x-axis and their respective cumulative frequencies on y-axis, its ogive can be
obtained as follows.
Probability 
Exercise 15.1
 
Question 1: 
Complete the following statements: 
(i) Probability of an event E + Probability of the event ‘not E’ = _______. 
(ii) The probability of an event that cannot happen is _________. Such as event is called 
_________. 
(iii) The probability of an event that is certain to happen is _________. Such as event is 
called ________. 
(iv) The sum of the probabilities of all the elementary events of an experiment is 
_________. 
(v) The probability of an event is greater than or equal to _______ and less than or equal 
to _______. 

Answer: 

(i) 1 

(ii) 0, impossible event 

(iii) 1, sure event or certain event 

(iv) 1 

(v) 0, 1 

Question 2: 
Which of the following experiments have equally likely outcomes? Explain. 
(i) A driver attempts to start a car. The car starts or does not start. 
(ii) A player attempts to shoot a basketball. She/he shoots or misses the shot. 
(iii) A trial is made to answer a true-false question. The answer is right or wrong. 
(iv) A baby is born. It is a boy or a girl. 

Answer: 
(i) It is not an equally likely event, as it depends on various factors such as whether the 
car will start or not. And factors for both the conditions are not the same. 

(ii) It is not an equally likely event, as it depends on the player’s ability and there is no 
information given about that. 

(iii) It is an equally likely event. 

(iv) It is an equally likely event. 

Question 3: 
Why is tossing a coin considered to be a fair way of deciding which team should get the 
ball at the beginning of a football game? 

Answer: 

When we toss a coin, the possible outcomes are only two, head or tail, which are equally 
likely outcomes. Therefore, the result of an individual toss is completely unpredictable. 

 
Question 4: 
Which of the following cannot be the probability of an event? 

Answer: 

Probability of an event (E) is always greater than or equal to 0. Also, it is always less than 
or equal to one. This implies that the probability of an event cannot be negative or 
greater than 1. Therefore, out of these alternatives, −1.5 cannot be a probability of an 
event. 

Hence, (B) 

 
Question 5: 
If P(E) = 0.05, what is the probability of ‘not E’? 

Answer: 

We know that, 
 

Therefore, the probability of ‘not E’ is 0.95. 

 
Question 6: 
A bag contains lemon flavoured candies only. Malini takes out one candy without 
looking into the bag. What is the probability that she takes out 
(i) an orange flavoured candy? 
(ii) a lemon flavoured candy? 

Answer: 

(i) The bag contains lemon flavoured candies only. It does not contain any orange 
flavoured candies. This implies that every time, she will take out only lemon flavoured 
candies. Therefore, event that Malini will take out an orange flavoured candy is an 
impossible event. 

Hence, P (an orange flavoured candy) = 0 

(ii)As the bag has lemon flavoured candies, Malini will take out only lemon flavoured 
candies. Therefore, event that Malini will take out a lemon flavoured candy is a sure 
event. 

P (a lemon flavoured candy) = 1 

 
Question 7: 
It is given that in a group of 3 students, the probability of 2 students not having the 
same birthday is 0.992. What is the probability that the 2 students have the same 
birthday? 

Answer: 

Probability that two students are not having same birthday P ( ) = 0.992 

Probability that two students are having same birthday P (E) = 1 − P ( ) 


= 1 − 0.992 

= 0.008 

 
Question 8: 
A bag contains 3 red balls and 5 black balls. A ball is drawn at random from the bag. 
What is the probability that the ball drawn is (i) red? (ii) not red? 

Answer: 

(i) Total number of balls in the bag = 8 

(ii) Probability of not getting red ball 

= 1 − Probability of getting a red ball 

 
Question 9: 
A box contains 5 red marbles, 8 white marbles and 4 green marbles. One marble is 
taken out of the box at random. What is the probability that the marble taken out will be 
(i) red? (ii) white? (iii) not green? 

Answer: 

Total number of marbles = 5 + 8 + 4 

= 17 

(i)Number of red marbles = 5 


 

(ii)Number of white marbles = 8 

(iii)Number of green marbles = 4 

Probability of not getting a green marble  

Question 10: 
A piggy bank contains hundred 50 p coins, fifty Rs 1 coins, twenty Rs 2 coins and ten 
Rs 5 coins. If it is equally likely that one of the coins will fall out when the bank is 
turned upside down, what is the probability that the coin 
(i) Will be a 50 p coin? 
(ii) Will not be a Rs.5 coin? 

Answer: 

Total number of coins in a piggy bank = 100 + 50 + 20 + 10 

= 180 

(i) Number of 50 p coins = 100 


 

(ii) Number of Rs 5 coins = 10 

Probability of not getting a Rs 5 coin  

 
Question 11: 
Gopi buys a fish from a shop for his aquarium. The shopkeeper takes out one fish at 
random from a tank containing 5 male fish and 8 female fish (see the given figure). 
What is the probability that the fish taken out is a male fish? 

Answer: 

Total number of fishes in a tank 

= Number of male fishes + Number of female fishes 

= 5 + 8 = 13 
 

 
Question 12: 
A game of chance consists of spinning an arrow which comes to rest pointing at one of 
the numbers 1, 2, 3, 4, 5, 6, 7, 8 (see the given figure), and these are equally likely 
outcomes. What is the probability that it will point at 
(i) 8? 
(ii) an odd number? 
(iii) a number greater than 2? 
(iv) a number less than 9? 

Answer: 

Total number of possible outcomes = 8 

(i)  

(ii) Total number of odd numbers on spinner = 4 

(iii) The numbers greater than 2 are 3, 4, 5, 6, 7, and 8. 

Therefore, total numbers greater than 2 = 6 


 

(iv) The numbers less than 9 are 1, 2, 3, 4, 6, 7, and 8. 

Therefore, total numbers less than 9 = 8 

Probability of getting a number less than 9  

 
Question 13: 
A die is thrown once. Find the probability of getting 
(i) a prime number; 
(ii) a number lying between 2 and 6; 
(iii) an odd number. 

Answer: 

The possible outcomes when a dice is thrown = {1, 2, 3, 4, 5, 6} 

Number of possible outcomes of a dice = 6 

(i) Prime numbers on a dice are 2, 3, and 5. 

Total prime numbers on a dice = 3 

Probability of getting a prime number =  

(ii) Numbers lying between 2 and 6 = 3, 4, 5 

Total numbers lying between 2 and 6 = 3 

Probability of getting a number lying between 2 and 6  

(iii) Odd numbers on a dice = 1, 3, and 5 


Total odd numbers on a dice = 3 

Probability of getting an odd number  

 
Question 14: 
One card is drawn from a well-shuffled deck of 52 cards. Find the probability of getting 
(i) a king of red colour 
(ii) a face card 
(iii) a red face card 
(iv) the jack of hearts 
(v) a spade 
(vi) the queen of diamonds 

Answer: 

Total number of cards in a well-shuffled deck = 52 

(i) Total number of kings of red colour = 2 

P (getting a king of red colour)  

(ii) Total number of face cards = 12 

P (getting a face card)  

(iii) Total number of red face cards = 6 


P (getting a red face card)  

(iv) Total number of Jack of hearts = 1 

P (getting a Jack of hearts)  

(v) Total number of spade cards = 13 

P (getting a spade card)  

(vi) Total number of queen of diamonds = 1 

P (getting a queen of diamond)  

 
Question 15: 
Five cards−−the ten, jack, queen, king and ace of diamonds, are well-shuffled with their 
face downwards. One card is then picked up at random. 
(i) What is the probability that the card is the queen? 
(ii) If the queen is drawn and put aside, what is the probability that the second card 
picked up is (a) an ace? (b) a queen? 

Answer: 
(i) Total number of cards = 5 

Total number of queens = 1 

P (getting a queen)  

(ii) When the queen is drawn and put aside, the total number of remaining cards will be 4. 

(a) Total number of aces = 1 

P (getting an ace)  

(b) As queen is already drawn, therefore, the number of queens 

will be 0. 

P (getting a queen) = 0 

 
Question 16: 
12 defective pens are accidentally mixed with 132 good ones. It is not possible to just 
look at a pen and tell whether or not it is defective. One pen is taken out at random 
from this lot. Determine the probability that the pen taken out is a good one. 

Answer: 

Total number of pens = 12 + 132 = 144 

Total number of good pens = 132 

P (getting a good pen)  


 

 
Question 17: 
(i) A lot of 20 bulbs contain 4 defective ones. One bulb is drawn at random from the lot. 
What is the probability that this bulb is defective? 
(ii) Suppose the bulb drawn in (i) is not defective and is not replaced. Now one bulb is 
drawn at random from the rest. What is the probability that this bulb is not defective? 

Answer: 

(i) Total number of bulbs = 20 

Total number of defective bulbs = 4 

P (getting a defective bulb)  

(ii) Remaining total number of bulbs = 19 

Remaining total number of non-defective bulbs = 16 − 1 = 15 

P (getting a not defective bulb)  

 
Question 18: 
A box contains 90 discs which are numbered from 1 to 90. If one disc is drawn at 
random from the box, find the probability that it bears 
(i) a two-digit number 
(ii) a perfect square number 
(iii) a number divisible by 5. 

Answer: 
Total number of discs = 90 

(i) Total number of two-digit numbers between 1 and 90 = 81 

P (getting a two-digit number)  

(ii) Perfect squares between 1 and 90 are 1, 4, 9, 16, 25, 36, 49, 64, and 81. Therefore, 
total number of perfect squares between 1 and 90 is 9. 

P (getting a perfect square)  

(iii) Numbers that are between 1 and 90 and divisible by 5 are 5, 10, 15, 20, 25, 30, 35, 40, 
45, 50, 55, 60, 65, 70, 75, 80, 85, and 90. Therefore, total numbers divisible by 5 = 18 

Probability of getting a number divisible by 5  

Question 19: 
A child has a die whose six faces shows the letters as given below: 

 
The die is thrown once. What is the probability of getting (i) A? (ii) D? 

Answer: 

Total number of possible outcomes on the dice = 6 

(i) Total number of faces having A on it = 2 

P (getting A)  

(ii) Total number of faces having D on it = 1 

P (getting D)  
Question 20: 
Suppose you drop a die at random on the rectangular region shown in the given figure. 
What is the probability that it will land inside the circle with diameter 1 m? 

Answer: 

Area of rectangle = l × b = 3 × 2 = 6 m2 

Area of circle (of diameter 1 m)  

P (die will land inside the circle)  

Question 21: 
A lot consists of 144 ball pens of which 20 are defective and the others are good. Nuri 
will buy a pen if it is good, but will not buy if it is defective. The shopkeeper draws one 
pen at random and gives it to her. What is the probability that 
(i) She will buy it? 
(ii) She will not buy it? 

Answer: 

Total number of pens = 144 

Total number of defective pens = 20 

Total number of good pens = 144 − 20 = 124 

(i) Probability of getting a good pen  


P (Nuri buys a pen)  

(ii) P (Nuri will not buy a pen)  

Question 22: 
Two dice, one blue and one grey, are thrown at the same time. 
(i) Write down all the possible outcomes and complete the following table: 

Event: 
1 1
2  3 4 5 6 7 8  9 12 
Sum of two dice  0  1 

Probability                 
   

(ii) A student argues that ‘there are 11 possible outcomes 2, 3, 4, 5, 6, 7, 8, 9, 10, 11 

and 12. Therefore, each of them has a probability . Do you agree with this argument? 

Answer: 

(i) It can be observed that, 

To get the sum as 2, possible outcomes = (1, 1) 

To get the sum as 3, possible outcomes = (2, 1) and (1, 2) 

To get the sum as 4, possible outcomes = (3, 1), (1, 3), (2, 2) 

To get the sum as 5, possible outcomes = (4, 1), (1, 4), (2, 3), (3, 2) 

To get the sum as 6, possible outcomes = (5, 1), (1, 5), (2, 4), (4, 2), 

(3, 3) 

To get the sum as 7, possible outcomes = (6, 1), (1, 6), (2, 5), (5, 2), 

(3, 4), (4, 3) 


To get the sum as 8, possible outcomes = (6, 2), (2, 6), (3, 5), (5, 3), 

(4, 4) 

To get the sum as 9, possible outcomes = (3, 6), (6, 3), (4, 5), (5, 4) 

To get the sum as 10, possible outcomes = (4, 6), (6, 4), (5, 5) 

To get the sum as 11, possible outcomes = (5, 6), (6, 5) 

To get the sum as 12, possible outcomes = (6, 6) 

Event: 
1 1 1
2  3  4  5  6  7  8  9 
0  1  2 
Sum of two dice 

Probability 

(ii)Probability of each of these sums will not be as these sums are not equally likely. 

Question 23: 
A game consists of tossing a one rupee coin 3 times and noting its outcome each time. 
Hanif wins if all the tosses give the same result i.e., three heads or three tails, and 
loses otherwise. Calculate the probability that Hanif will lose the game. 

Answer: 

The possible outcomes are 

{HHH, TTT, HHT, HTH, THH, TTH, THT, HTT} 

Number of total possible outcomes = 8 

Number of favourable outcomes = 2 {i.e., TTT and HHH} 


P (Hanif will win the game)  

P (Hanif will lose the game)  

Question 24: 
A die is thrown twice. What is the probability that 
(i) 5 will not come up either time? 
(ii) 5 will come up at least once? 
[Hint: Throwing a die twice and throwing two dice simultaneously are treated as the 
same experiment]. 

Answer: 

Total number of outcomes = 6 × 6 

= 36 

(i)Total number of outcomes when 5 comes up on either time are (5, 1), (5, 2), (5, 3), (5, 
4), (5, 5), (5, 6), (1, 5), (2, 5), (3, 5), (4, 5), (6, 5) 

Hence, total number of favourable cases = 11 

P (5 will come up either time)  

P (5 will not come up either time)  

(ii)Total number of cases, when 5 can come at least once = 11 

P (5 will come at least once)  

 
Question 25: 
Which of the following arguments are correct and which are not correct? Give reasons 
for your answer. 
(i) If two coins are tossed simultaneously there are three possible outcomes−−two 
heads, two tails or one of each. Therefore, for each of these outcomes, the probability 

is . 
(ii) If a die is thrown, there are two possible outcomes−−an odd number or an even 

number. Therefore, the probability of getting an odd number is . 

Answer: 

(i) Incorrect 

When two coins are tossed, the possible outcomes are (H, H), (H, T), (T, H), and (T, T). It 
can be observed that there can be one of each in two possible ways − (H, T), (T, H). 

Therefore, the probability of getting two heads is , the probability of getting two tails is

, and the probability of getting one of each is . 

It can be observed that for each outcome, the probability is not . 

(ii) Correct 

When a dice is thrown, the possible outcomes are 1, 2, 3, 4, 5, and 6. Out of these, 1, 3, 5 
are odd and 2, 4, 6 are even numbers. 

Therefore, the probability of getting an odd number is . 

 
 
 
 
 
 
 
Exercise 15.2 
Question 1: 
Two customers Shyam and Ekta are visiting a particular shop in the same week 
(Tuesday to Saturday). Each is equally likely to visit the shop on any day as on another 
day. What is the probability that both will visit the shop on 
(i) the same day? (ii) consecutive days? (iii) different days? 

Answer: 

There are a total of 5 days. Shyam can go to the shop in 5 ways and Ekta can go to the 
shop in 5 ways. 

Therefore, total number of outcomes = 5 × 5 = 25 

(i) They can reach on the same day in 5 ways. 

i.e., (t, t), (w, w), (th, th), (f, f), (s, s) 

P (both will reach on same day)  

(ii) They can reach on consecutive days in these 8 ways – (t, w), (w, th), (th, f), (f, s), (w, t), 
(th, w), (f, th), (s, f). 

Therefore, P (both will reach on consecutive days)  

(iii) P (both will reach on same day) [(From (i)] 

P (both will reach on different days)  

Question 2: 
A die is numbered in such a way that its faces show the number 1, 2, 2, 3, 3, 6. It is 
thrown two times and the total score in two throws is noted. Complete the following 
table which gives a few values of the total score on the two throws: 
 
What is the probability that the total score is 
(i) even? (ii) 6? (iii) at least 6? 

Answer: 

+  1 2 2 3 3 6 

1  2 3 3 4 4 7 

2  3 4 4 5 5 8 

2  3 4 4 5 5 8 

3  4 5 5 6 6 9 

3  4 5 5 6 6 9 

6  7 8 8 9 9 12 

Total number of possible outcomes when two dice are thrown = 6 × 6 = 36 

(i) Total times when the sum is even = 18 

P (getting an even number)  


(ii) Total times when the sum is 6 = 4 

P (getting sum as 6)  

(iii) Total times when the sum is at least 6 (i.e., greater than 5) = 15 

P (getting sum at least 6)  

Question 3: 
A bag contains 5 red balls and some blue balls. If the probability of drawing a blue ball 
is double that of a red ball, determine the number of blue balls in the bag. 

Answer: 

Let the number of blue balls be x. 

Number of red balls = 5 

Total number of balls = x + 5 

P (getting a red ball)  

P (getting a blue ball)  

Given that, 

 
However, the number of balls cannot be negative. 

Hence, number of blue balls = 10 

Question 4: 
A box contains 12 balls out of which x are black. If one ball is drawn at random from the 
box, what is the probability that it will be a black ball? 
If 6 more black balls are put in the box, the probability of drawing a black ball is now 
double of what it was before. Find x. 

Answer: 

Total number of balls = 12 

Total number of black balls = x 

P (getting a black ball) =  

If 6 more black balls are put in the box, then 

Total number of balls = 12 + 6 = 18 

Total number of black balls = x + 6 

P (getting a black ball now)  

According to the condition given in the question, 

Question 5: 
A jar contains 24 marbles, some are green and others are blue. If a marble is drawn at 

random from the jar, the probability that it is green is . Find the number of blue balls 
in the jar. 

Answer: 

Total number of marbles = 24 

Let the total number of green marbles be x. 

Then, total number of blue marbles = 24 − x 

P (getting a given marble)  

According to the condition given in the question, 

Therefore, total number of green marbles in the jar = 16 

Hence, total number of blue marbles = 24 − x = 24 − 16 = 8 

You might also like